Stack Exchange Network

Stack Exchange network consists of 183 Q&A communities including Stack Overflow , the largest, most trusted online community for developers to learn, share their knowledge, and build their careers.

Q&A for work

Connect and share knowledge within a single location that is structured and easy to search.

Determining the problem type - Vehicle Routing - Task assignment

I want to figure out what problem I'm dealing with so I can move in the right direction of research. I'm still at the very beginning of formulating the problem. What I've done so far is figure out just enough to understand how little I know about optimization problems. So for me it is also about the bascis.

The problem is probably a combination of several.

The minimization goal: I want to minimize the total time $t$ , or the costs, for the process.

  • There is one vehicle.
  • The vehicle has limited capacity.
  • The vehicle moves from point / node to another to complete a task. So I think that nodes are equivalent to tasks in this problem?
  • There is a distance between nodes. This is part of the costs, since this means travel time.
  • A task $T_n$ takes a certain amount of time $t_T$ , e.g. $t_{T_n}=3\text{minutes}$ .
  • Material is consumed per task.
  • The vehicle starts fully loaded and can recharge the material at any station.
  • There are at least two stations.

Why I am not making progress in my research: The complexity comes from the fact that a task has to happen twice in one place. Let's call them two types of tasks, "prepare" and "finish".

  • Two task types. Both have to be done per location.
  • There must be a certain amount of time between "prepare" $T_{pn}$ and "finish" $T_{fn}$ , e.g. 10 minutes.
  • However, only a certain amount of time may elapse, not more, e.g. $t_{max} = 30\text{minutes}$ .

I had the feeling that this is about time windows, but what I find difficult is the fact that the task must be done twice in the same place or same node, but then with the different time windows. That means there are dependencies and they specify the starting points. So the start of $T_{f1}$ will be determined by $T_{p1}$ . At the same time I do not really care when a time window starts to begin with $T_{p1}$ , work just has to be done. It is OK, when $T_{p5}$ is done before $T_{p1}$ , this is flexible. And also, when $T_{p1..7}$ have been done, $T_{f1..7}$ can be done afterwards sequentially but do not have to. Tasks just need to be fulfilled within the time frame that is made up by $T_{pn}$ .

So I think the only hard dependency is the point of time $tp$ after $T_{pn}$ has been done:

So, $T_{fn}^{tp}$ lies within $[ ( T_{pn}^{tp} + 10\text{minutes} ) , ( T_{pn}^{tp} + 30\text{minutes} ) ]$

ps. Sorry, but I am not very sure about how to express all of this in the correct mathematical way w.r.t. optimization problems, this is something I would like to catch on in the near future, as soon as I know in which direction I should head now.

So by what I know so far this is some kind of vehicle routing problem VRP with time windows TW. But TW is not very strict, so is this more about scheduling?

And when I would start to implement a solution, what tools could I use for that?

  • vehicle-routing

Pascal's user avatar

  • 1 $\begingroup$ So you want to find a single route to serve all tasks in the shortest amount of time? Then you have a resource constrained shortest path problem. That is often solved by a labeling algorithm, which is a dynamic programming approach. $\endgroup$ –  gmn Commented Sep 23, 2022 at 9:03
  • $\begingroup$ Why don't you just aggregate those two tasks as one? $\endgroup$ –  overboxed Commented Sep 24, 2022 at 0:40

3 Answers 3

I think that the keyword you are looking for is "temporal constraints". Here are a few articles related to this topic:

  • "Combined vehicle routing and scheduling with temporal precedence and synchronization constraints" (Bredström et Rönnqvist, 2008) DOI
  • "The vehicle routing problem with time windows and temporal dependencies" (Dohn et al., 2011) DOI
  • "A constraint-programming based decomposition method for the Generalised Workforce Scheduling and Routing Problem (GWSRP)" (Bourreau et al., 2020) DOI

fontanf's user avatar

  • 1 $\begingroup$ Adding to that: "An exact solution method for a rich helicopter flight scheduling problem arising in offshore oil and gas logistics" (Nafstad et al., 2021) handles a minimum time requirement between two visits in a route in a labeling algorithm. sciencedirect.com/science/article/pii/S0305054820302756 $\endgroup$ –  gmn Commented Sep 23, 2022 at 12:09

If you want to view the problem as a VRP, it might help to clone all the nodes. Rather than having a single node $N_j$ for task $j,$ you have node $N_j^p$ for the preparation portion of the task and node $N_j^f$ for the finishing portion of the task. You will also end up cloning arcs, so that arc $(N_i, N_j)$ becomes $(N_i^h,N_j^k)$ for all combinations of $h,k\in \lbrace p,f\rbrace.$ (You may also want to include arcs $N_j^p \rightarrow N_j^f$ with distance/travel time 0 to allow the vehicle to stick around after a prep task until the time window for the finish task opens.) Preventing a visit to the finish copy of a node before the visit to the prep copy is handled by the time window constraints.

prubin's user avatar

You have a vehicle routing problem (VRP) here. Because you only have one vehicle, the problem can be reduced to a traveling salesman problem (TSP). Every Vehicle routing is an assignment problem. Basically, VRP is about the combination of assignments and knapsack. You assign a vehicle for each node and a knapsack for the commodity carried for each vehicle. From your description, besides Time Windows there are two keywords that you might perhaps want do some little digging into, which are dependent VRP and Multi-Trip VRP.

For the tools, you might want to consider approximate technique (meta or heuristics) because VRP is NP-hard.

overboxed's user avatar

  • $\begingroup$ Additional keyword: Multi-depot, Split-delivery $\endgroup$ –  overboxed Commented Sep 23, 2022 at 15:51

Your Answer

Sign up or log in, post as a guest.

Required, but never shown

By clicking “Post Your Answer”, you agree to our terms of service and acknowledge you have read our privacy policy .

Not the answer you're looking for? Browse other questions tagged scheduling vehicle-routing or ask your own question .

  • Featured on Meta
  • Site maintenance - Mon, Sept 16 2024, 21:00 UTC to Tue, Sept 17 2024, 2:00...
  • User activation: Learnings and opportunities
  • Join Stack Overflow’s CEO and me for the first Stack IRL Community Event in...

Hot Network Questions

  • How many minutes the Apollo astronauts had to wait from splashdown until the first frogmen touched the CM?
  • In QGIS 3, changing borders of shared polygon shapefile features
  • Definition of annuity
  • What would the natural diet of Bigfoot be?
  • How to reply to a revise and resubmit review, saying is all good?
  • How to prove that the Greek cross tiles the plane?
  • What would be an appropriate translation of Solitude?
  • Looking for a short story on chess, maybe published in Playboy decades ago?
  • Browse a web page through SSH? (Need to access router web interface remotely, but only have SSH access to a different device on LAN)
  • How to best characterize the doctrine deriving from Palsgraf?
  • The meaning of an implication in an existential quantifier
  • How did people know that the war against the mimics was over?
  • Is it safe to use the dnd 3.5 skill system in pathfinder 1e?
  • For a bike with "Forged alloy crank with 44T compact disc chainring", can the chainring be removed?
  • Difference between 2 version numbers from `adb --version`
  • What prevents indoor climbing gyms from making a v18 boulder even if one hasn't been found outside?
  • siunitx dollar per hour broken going from SI to qty
  • Creating good tabularx
  • How to prevent a bash script from running repeatedly at the start of the terminal
  • How can a microcontroller (such as an Arduino Uno) that requires 7-21V input voltage be powered via USB-B which can only run 5V?
  • Subject verb agreement - I as well as he is/am the culprit
  • "There is a bra for every ket, but there is not a ket for every bra"
  • How can I send instance attributes from Geometry Nodes to Shading Editor?
  • Why would the absence of Chalmers' 'consciousness' make p-zombie world 'inconceivable'?

vehicle routing assignment problem

IEEE Account

  • Change Username/Password
  • Update Address

Purchase Details

  • Payment Options
  • Order History
  • View Purchased Documents

Profile Information

  • Communications Preferences
  • Profession and Education
  • Technical Interests
  • US & Canada: +1 800 678 4333
  • Worldwide: +1 732 981 0060
  • Contact & Support
  • About IEEE Xplore
  • Accessibility
  • Terms of Use
  • Nondiscrimination Policy
  • Privacy & Opting Out of Cookies

A not-for-profit organization, IEEE is the world's largest technical professional organization dedicated to advancing technology for the benefit of humanity. © Copyright 2024 IEEE - All rights reserved. Use of this web site signifies your agreement to the terms and conditions.

Genetic Algorithms with Neural Cost Predictor for Solving Hierarchical Vehicle Routing Problems

When vehicle routing decisions are intertwined with higher-level decisions, the resulting optimization problems pose significant challenges for computation. Examples are the multi-depot vehicle routing problem (MDVRP), where customers are assigned to depots before delivery, and the capacitated location routing problem (CLRP), where the locations of depots should be determined first. A simple and straightforward approach for such hierarchical problems would be to separate the higher-level decisions from the complicated vehicle routing decisions. For each higher-level decision candidate, we may evaluate the underlying vehicle routing problems to assess the candidate. As this approach requires solving vehicle routing problems multiple times, it has been regarded as impractical in most cases. We propose a novel deep-learning-based approach called Genetic Algorithm with Neural Cost Predictor (GANCP) to tackle the challenge and simplify algorithm developments. For each higher-level decision candidate, we predict the objective function values of the underlying vehicle routing problems using a pre-trained graph neural network without actually solving the routing problems. In particular, our proposed neural network learns the objective values of the HGS-CVRP open-source package that solves capacitated vehicle routing problems. Our numerical experiments show that this simplified approach is effective and efficient in generating high-quality solutions for both MDVRP and CLRP and has the potential to expedite algorithm developments for complicated hierarchical problems. We provide computational results evaluated in the standard benchmark instances used in the literature.

Funding: This research was funded by the Korean government (MSIT) through the National Research Foundation of Korea (NRF) grant RS-2023-00259550.

Key words: multi-depot vehicle routing problem, genetic algorithm, deep learning, cost prediction

1 Introduction

Vehicle Routing Problems (VRPs) have garnered significant attention over the past few decades due to their crucial role in last-mile delivery logistics. CapGemini Research Institute ( 2019 ) reports that last-mile delivery accounts for 41% of the supply chain expenditures. As a direct extension of the Traveling Salesman Problem (TSP), the Capacitated Vehicle Routing Problem (CVRP) forms a fundamental class of VRPs and serves as the basis for addressing many last-mile delivery challenges. In many real-life routing scenarios, the VRP under consideration often includes additional constraints, parameters, or assumptions. Such problems are referred to as VRP variants or rich VRPs.

Among the many VRP variants, we focus on hierarchical VRPs (HVRPs), where the higher-level decisions impact the lower-level decisions. Examples include two widely used VRP variants called the Multi-Depot VRP (MDVRP) and the Capacitated Location Routing Problem (CLRP). In essence, the MDVRP involves the higher-level task of clustering customers with order-fulfillment centers, from which the respective demands are fulfilled. Clustering decomposes the original problem into multiple CVRPs. Evidently, the MDVRP is at least as difficult as the CVRP, and thus an 𝒩 ⁢ 𝒫 𝒩 𝒫 \mathcal{NP} caligraphic_N caligraphic_P -Hard problem. In MDVRPs, the higher-level decisions involve assigning each customer to a depot, whereas the lower-level decisions focus on routing vehicles from each depot to its assigned customers. As a generalization of the MDVRP, the CLRP requires additional higher-level decisions that determine which depots should be opened from the available candidate locations.

The main challenge in obtaining real-time solutions to combinatorial optimization (CO) problems lies in their computational complexity. Although heuristics tackle this issue for relatively small and medium-sized problems, the search space for good solutions grows exponentially with the problem size. Many logistics problems in the practical settings encompass several hundred or even thousands of customers (Qi et al., 2012 ) . The MDVRP is a fundamental optimization problem in many real-world transportation problems, including parcel delivery, waste collection, mobile healthcare routing, and field service management. Some of these routing scenarios involve solving large networks comprising thousands of locations and need to be tackled on a daily basis, such as waste collection and e-commerce order fulfillment (Arnold et al., 2019 ) . In waste collection, numerous trucks dispatched from various waste disposal plants (depots) are tasked with collecting waste from tens of thousands of customers. In 2022, a prominent e-commerce firm Amazon Logistics processed 4.79 billion U.S. delivery orders (Capital One Shopping Research, 2019 ) , equating to approximately 9,116 orders per minute. These examples underscore the magnitude and complexity of large-scale vehicle routing faced by real-world businesses and the need for fast operational decisions.

As another example of an HVRP, consider grocery deliveries by an organization that operates a chain of grocery stores or hypermarkets across an urban area. If a customer’s delivery demands can be satisfied from multiple stores, the company needs to decide which store fulfills the customer demand more profitably. With several incoming orders from different geographical locations and a limited number of delivery agents available at each store, the organization needs to determine the optimal assignment of each customer to a store so that the overall transportation cost is minimized. It is evident that allocating one or more customers to their nearest store, disregarding other factors such as the geographical distribution of demands and the delivery capacities, may lead to suboptimal or even infeasible solutions. Presently, a prevalent method to address such large-scale HVRPs involves a cluster first and route second approach. However, employing a fixed strategy for higher-level decisions may result in increased operational costs.

The objective function value of an MDVRP with customer-depot assignments is calculated by adding the optimal costs of all decomposed CVRPs. For problems that can be decomposed into “simpler” subproblems, a convenient solution method is to find the optimal decomposition. However, recall that an MDVRP decomposition leads to multiple CVRPs that are still 𝒩 ⁢ 𝒫 𝒩 𝒫 \mathcal{NP} caligraphic_N caligraphic_P -Hard. To evaluate a decomposition, the only requirement is the optimal routing cost of each subproblem without the need for routing decisions. In this research, we train a deep neural network (NN) model to predict the optimal cost of each CVRP instance arising from the decomposition of an HVRP, without actually solving the CVRPs. The predicted cost can be leveraged as an approximate measure of the decomposition strategy’s effectiveness. With the help of a Genetic Algorithm (GA), we improve the clustering of customers with depots. The fitness cost of an assignment is calculated from the sum of subproblems’ cost predictions and the distinctiveness of the decomposition compared to other assignments. We refer to this approach as the Genetic Algorithm with Neural Cost Predictor ( GANCP ).

\text{GANCP}^{+} GANCP start_POSTSUPERSCRIPT + end_POSTSUPERSCRIPT . We compare the test results with the performance of an open-source generic VRP solver, the Vehicle Routing Open-source Optimization Machine (VROOM) (Coupey et al., 2023 ) . In many VRP variants, VROOM is known to produce high-quality solutions that are close to the best-known solutions reported in the literature. We analyze the performance of the cost prediction model using three different train data generation procedures. We also showcase the transferability of our heuristic to instances that are different from the trained distribution and size.

\text{GANCP}^{+} GANCP start_POSTSUPERSCRIPT + end_POSTSUPERSCRIPT has the potential to tackle many challenging hierarchical VRPs that arise in the real world for the following reasons. First, the trained NN can predict the cost of each candidate quickly. The biggest challenge in the decomposition approach for hierarchical VRPs is the time to solve subproblems. In our approach, we use the neural network to quickly generate a cost prediction without solving the problem. Second, our proposed NN can learn the cost values from standard VRP solvers for a wide range of problem sizes. Fundamental building blocks in hierarchical VRPs are well-studied problems such as CVRP, for which several outstanding VRP solvers exist. Therefore, we can train the NN for each standard VRP class and use it for various HVRPs that use the same VRP class of problems as the subproblems. This can significantly reduce algorithm development time, as we can focus on handling non-standard modeling components within the GA framework without considering the routing solutions. Third, our proposed NN model can provide a robust prediction capability for the same VRP class within different problem contexts. We demonstrate the transferability of our trained NN model from the MDVRP to the CLRP by experimenting with multiple benchmark datasets.

Our goal in this paper is, therefore, to show that the proposed approach can simplify algorithm development processes for complicated hierarchical vehicle routing problems and produce quality solutions in a short amount of time rather than to develop an algorithm that can produce the best solutions and outperform all existing algorithms. We will focus on illustrating this main point throughout the paper and will use MDVRP and CLRP as examples.

The remainder of this paper is organized as follows. In Section 2 , we discuss the relevant literature on the CVRP methodologies involving reinforcement learning (RL), machine learning (ML), heuristics, and cost estimation techniques, followed by a brief review of related literature on MDVRP and CLRP. Section 3 describes the HVRPs in general and focuses on the decomposition of the MDVRP to VRPs based on the mathematical formulation. The neural network prediction model and training details are discussed in Section 4 . Section 5 explains our MDVRP solution methodology that incorporates the NN model architecture and the heuristics in detail. We discuss and evaluate the performance of the NN model using three different training procedures in Section 6 . We further demonstrate the effectiveness of our proposed heuristic on multiple MDVRP instances, including out-of-distribution and out-of-range instances. The paper finally concludes with a discussion of relevant and related research directions in optimization using supervised learning in Section 7 . In addition, we demonstrate the transferability of our approach with a simple extension of the MDVRP methodology for the CLRP in Section A.1 and examine the results of the CLRP experiments in Section A.2 .

2 Literature Review

First, we review the relevant literature on VRPs that use deep RL as a solution methodology. Subsequently, we explore the supervised learning approaches applied to VRPs. We further discuss pertinent approximation techniques utilized in the vehicle routing literature for optimal cost estimations. This is followed by a review of relevant research on solution methods for the MDVRP and the CLRP. Finally, we highlight our contributions to the existing research gap in this domain.

2.1 Deep Reinforcement Learning Approaches for VRPs

Over the past few years, deep reinforcement learning has been demonstrating notable success in solving combinatorial optimization problems, especially the TSP and the CVRP. Bello et al. ( 2017 ) introduces an effective deep RL framework to train and solve the TSP using pointer networks (Vinyals et al., 2015 ) . The problem is modeled as a Markov Decision Process and uses the pointer network for encoding the inputs, which are then learned using policy gradients. Nazari et al. ( 2018 ) extends this work for VRPs without a Recurrent Neural Network for the input encoding. Unlike Natural Language Processing, the order of the input sequence—the customer locations—can be treated independently in the VRP. While the inclusion of a pointer encoder network does not negatively impact the final accuracy, it delays learning due to added model complexity. Kool et al. ( 2019 ) yields better performance for CVRPs using an architecture for solution construction with multiple multi-head attention layers, proposed in Vaswani et al. ( 2017 ) , to capture the node dependencies, and using the REINFORCE algorithm to train the model. With consideration of symmetries in the solutions of a CO problem, Kwon et al. ( 2020 ) designs the Policy Optimization with Multiple Optima (POMO) network with a modified REINFORCE algorithm to explore the solution space more effectively. This improves the results in terms of both performance gap and inference time compared to the existing architectures. Recent studies have also addressed different VRP variants using deep RL frameworks (Bogyrbayeva et al., 2023 ; Park et al., 2023 ) . All of these approaches are end-to-end solution approaches that rely solely on neural networks to solve the problem.

Traditional end-to-end RL frameworks for CO solution construction have limitations concerning the scope and size of the problems they can effectively address. Another active research focus is on the use of RL as an enhancement heuristic for problem partitions or when an initial solution is known. Lu et al. ( 2020 ) utilize an RL agent as a heuristic controller to apply intra-route and inter-route improvement operators to an initial randomly generated solution and obtain high-quality CVRP solutions. In a similar study, Wu et al. ( 2021 ) proposes an RL model that employs a single improvement operator instead of multiple operations in combination. The authors compare operators such as 2-opt, node swap, and relocation and illustrate that 2-opt yields better results. Zong et al. ( 2022 ) proposes a Rewriting-by-Generating framework for solving large-scale VRPs hierarchically. Here, the Rewriter agent partitions the customers into independent regions, and the Generator agent solves the routing problem in each region. This method outperforms the HGS-CVRP solver by Vidal ( 2022 ) for large-scale problems, where the hyperparameters are fine-tuned for small to medium-size CVRP problems. However, RL requires higher computational time for training and inference compared to supervised machine learning and faces challenges in exploration due to the iterative nature of reinforcement learning with the environment.

2.2 Supervised Learning Approaches for VRPs

Multiple supervised learning frameworks exist for solving the VRP through the integration of exact methods, heuristics, or predictions of subproblem costs. The integration of ML with exact optimization techniques proves considerable promise and is currently a subject of active research. For example, Furian et al. ( 2021 ) solves VRPs using a branch-and-price method where ML models are used to predict the decision variables and branching scores. Another interesting study is Morabit et al. ( 2021 ) , where a Graph Neural Network (GNN) acts as a column selector to accelerate column generation for the VRP with time windows. Kim et al. ( 2024 ) further demonstrates that GNN can be leveraged to solve the separation problems in cutting plane methods, particularly for large-scale instances.

A few recent studies utilize supervised machine learning to learn potential cost improvements for a subproblem and iteratively improve the solution. Li et al. ( 2021 ) proposes a method that iteratively improves the solution to large-scale VRPs by learning to select subproblems and estimating the corresponding subsolution costs using a transformer architecture. Using a combination of the subproblem selector and cost estimation, the problem can be partitioned into smaller problems that are easily solved. Kim et al. ( 2023 ) presents a neural cross-exchange operator to solve multiple VRP variants based on cost decrement predictions and demonstrates its computational benefits against the traditional operator. In a recent study, Varol et al. ( 2024 ) proposed a neural network tour length estimator for a very large-scale Generalized TSP, incorporating instance-specific features to enhance estimation accuracy. The authors employ a similar approach to ours, integrating the neural network with a genetic algorithm in the solution framework. However, their estimator performs well only on specific problem sizes, in contrast to our NN model. The results of these studies showcase the efficacy of supervised learning in addressing large-scale routing problems. This stream of research motivates our proposed methodology.

2.3 Optimal Cost Estimations of VRPs

Numerous studies aim to accurately estimate optimal routing costs based on the topological distribution of customers, demands, and vehicle capacities. These studies typically utilize two methodologies: Continuous Approximation (CA) and Regression. CA, as an alternative to discrete models, employs smooth density functions to represent the network and is particularly effective for larger instances. CA techniques find application in estimating optimal transportation costs across various VRP variants, including emission minimization VRP (Saberi and Verbas, 2012 ) , school bus routing (Ellegood et al., 2015 ) , hub location routing problems (Ghaffarinasab et al., 2018 ) , dynamic multiple TSP (Garn, 2021 ) , and same-day delivery (Banerjee et al., 2022 ; Stroh et al., 2022 ) . A detailed review of the CA approaches used in freight distribution is available in Franceschetti et al. ( 2017 ) .

Among analytical expressions, the Daganzo-approximation (Robust et al., 1990 ) is widely recognized for estimating the optimal CVRP distance given by following the formula:

(1)

where A 𝐴 A italic_A represents the bounded area within which N 𝑁 N italic_N number of customers are distributed, k 𝑘 k italic_k is an area shape constant, and C 𝐶 C italic_C is the maximum number of customers a vehicle can serve. Figliozzi ( 2008 ) improves the accuracy of cost approximation using linear regression to calculate the optimal cost based on a few critical instance features as follows:

(2)

CA and regression models require careful adaptation for each problem type with the recognition of critical instance features. Further, they do not generalize well when confronted with varying problem sizes and data distributions. We employ a graph neural network to capture both the topological and demand features of the customers. This approach eliminates the need for focused feature engineering across different data types, enabling highly accurate predictions of optimal costs across varying problem sizes and data distributions.

2.4 Optimization Approaches for MDVRP and CLRP

Multiple studies use exact methods to solve the MDVRP, such as Laporte et al. ( 1988 ) , Contardo and Martinelli ( 2014 ) , and Baldacci and Mingozzi ( 2009 ) . Similarly, for the study of the CLRP using exact methods, one may refer to Baldacci et al. ( 2011 ) , Belenguer et al. ( 2011 ) and Contardo et al. ( 2014 ) . As a consequence of the 𝒩 ⁢ 𝒫 𝒩 𝒫 \mathcal{NP} caligraphic_N caligraphic_P -hardness of these problems, efficient heuristics are critical to the development of industrial routing solutions.

Cordeau et al. ( 1997 ) proposes a notable Tabu Search (TS) heuristic to solve the MDVRP, the Periodic VRP, and the Periodic TSP. A few years later, genetic algorithms emerged as one of the most effective heuristics for vehicle routing. Vidal et al. ( 2012 ) proposes a fast and efficient hybrid genetic algorithm that solves the MDVRP and the Periodic VRP. The algorithm differentiates neighboring solutions using a diversity factor in the fitness cost. In addition, a pool of infeasible solutions is preserved for diversification and better search space. Vidal et al. ( 2014 ) further improves the results where the depot, vehicle, and first customer visited on each route are optimally determined using a dynamic programming methodology. Sadati et al. ( 2021 ) presents a Variable Neighborhood Search algorithm that solves the MDVRP variants with a tabu-shaking mechanism to improve the search trajectory. End-to-end RL frameworks for MDVRP are presented in Zou et al. ( 2022 ) based on an improved transformer model and in Zhang et al. ( 2023 ) using Graph Attention Networks.

One of the most effective existing algorithms for solving the large-scale CLRP is by Schneider and Löffler ( 2019 ) , which explores multiple depot configurations using a Tree-Based Search Algorithm (TBSA). For a given configuration, an MDVRP is solved at the routing phase using a granular tabu search. The paper also discusses different variants of the algorithm with trade-offs in computational time and solution quality. More recently, Akpunar and Akpinar ( 2021 ) proposed a hybrid adaptive large neighborhood search method that demonstrates improved benchmark performance in a few CLRP benchmark instances.

\text{GANCP}^{+} GANCP start_POSTSUPERSCRIPT + end_POSTSUPERSCRIPT heuristic is empowered with a supervised learning model for cost estimation, which requires less training time and is, more importantly, applicable to distinct and large problem sizes, producing near-optimal results. To the best of our knowledge, this paper is the first to address solving large-scale hierarchical VRPs using a deep learning architecture to evaluate the subproblem decompositions.

3 Hierarchical Vehicle Routing Problems

Hierarchical vehicle routing problems (HVRPs) involve decisions made at multiple levels, often executed sequentially, to obtain an optimal routing solution. In a large or complex HVRP, decisions made at higher levels can decompose the problem into smaller subproblems. A good solution strategy involves identifying effective higher-level decisions that optimize the objective function while considering the original problem constraints. Higher-level decisions influence subsequent decisions, solution quality, and the optimal objective cost of the original problem. Lower-level decisions include detailed vehicle routing and schedule plans to complete deliveries or services designated to each vehicle.

As the HVRPs require decisions in addition to and including vehicle routing, it is computationally challenging to solve such problems in many real-life scenarios. This paper examines two prevalent HVRP variants: the MDVRP and the CLRP, with the latter analyzed in the Appendix. Due to the complexity of the HVRPs, multiple techniques and their combination are often used to solve large instances. A straightforward approach is to focus on the method to find effective higher-level decisions and solve the corresponding subproblems using an existing efficient algorithm or a proven solver. The higher-level decisions decompose the MDVRPs and the CLRPs into subsequent CVRPs, where the total solution cost of the subproblems determines the quality of the higher-level decisions. Since the CVRPs are well-researched, we focus on a heuristic method to find the higher-level decisions for the HVRPs.

In the remainder of this section, we first discuss the arc-based flow formulation of the MDVRP to ensure mathematical clarity. Following this, we demonstrate how employing a decomposition approach yields CVRP subproblems that reduce the complexity of the original problem. Later, we provide a discussion regarding the generalizability of the hierarchical decomposition approach for solving HVRPs.

3.1 Multi-Depot Vehicle Routing Problem

Consider a complete and undirected graph 𝒢 = ( 𝒱 , ℰ ) 𝒢 𝒱 ℰ \mathcal{G}=(\mathcal{V},\mathcal{E}) caligraphic_G = ( caligraphic_V , caligraphic_E ) , where 𝒱 𝒱 \mathcal{V} caligraphic_V represents the set of all nodes and ℰ ℰ \mathcal{E} caligraphic_E denotes the set of all edges connecting nodes in 𝒱 𝒱 \mathcal{V} caligraphic_V . Let 𝒟 ⊂ 𝒱 𝒟 𝒱 \mathcal{D}\subset\mathcal{V} caligraphic_D ⊂ caligraphic_V be the set of depots, and 𝒞 ⊂ 𝒱 𝒞 𝒱 \mathcal{C}\subset\mathcal{V} caligraphic_C ⊂ caligraphic_V be the set of customers, such that 𝒱 = 𝒟 ∪ 𝒞 𝒱 𝒟 𝒞 \mathcal{V}=\mathcal{D}\cup\mathcal{C} caligraphic_V = caligraphic_D ∪ caligraphic_C . Furthermore, let 𝒦 d subscript 𝒦 𝑑 \mathcal{K}_{d} caligraphic_K start_POSTSUBSCRIPT italic_d end_POSTSUBSCRIPT be the set of vehicles available for order-fulfillment at depot d ∈ 𝒟 𝑑 𝒟 d\in\mathcal{D} italic_d ∈ caligraphic_D , and 𝒦 = ∑ d ∈ 𝒟 𝒦 d 𝒦 subscript 𝑑 𝒟 subscript 𝒦 𝑑 \mathcal{K}=\sum_{d\in\mathcal{D}}\mathcal{K}_{d} caligraphic_K = ∑ start_POSTSUBSCRIPT italic_d ∈ caligraphic_D end_POSTSUBSCRIPT caligraphic_K start_POSTSUBSCRIPT italic_d end_POSTSUBSCRIPT is the set of all vehicles. Each vehicle k ∈ 𝒦 𝑘 𝒦 k\in\mathcal{K} italic_k ∈ caligraphic_K has a capacity Q k subscript 𝑄 𝑘 Q_{k} italic_Q start_POSTSUBSCRIPT italic_k end_POSTSUBSCRIPT , and customer i ∈ 𝒞 𝑖 𝒞 i\in\mathcal{C} italic_i ∈ caligraphic_C has a positive demand q i subscript 𝑞 𝑖 q_{i} italic_q start_POSTSUBSCRIPT italic_i end_POSTSUBSCRIPT such that q i ≤ Q min subscript 𝑞 𝑖 subscript 𝑄 min q_{i}\leq Q_{\text{min}} italic_q start_POSTSUBSCRIPT italic_i end_POSTSUBSCRIPT ≤ italic_Q start_POSTSUBSCRIPT min end_POSTSUBSCRIPT , where Q min subscript 𝑄 min Q_{\text{min}} italic_Q start_POSTSUBSCRIPT min end_POSTSUBSCRIPT is the minimum vehicle capacity among the fleet of vehicles. Note that a vehicle that starts from a depot must return to the same depot after completing its route.

We examine the mathematical model for MDVRP, originally proposed by Golden et al. ( 1977 ) and later improved by Ramos et al. ( 2020 ) through the distinct partitioning of vehicles among depots. Let the binary integer decision variable x i ⁢ j ⁢ k subscript 𝑥 𝑖 𝑗 𝑘 x_{ijk} italic_x start_POSTSUBSCRIPT italic_i italic_j italic_k end_POSTSUBSCRIPT equal 1 if vehicle k 𝑘 k italic_k traverses the edge ( i , j ) 𝑖 𝑗 (i,j) ( italic_i , italic_j ) in a feasible solution, and 0 otherwise. The traversal cost for the edge ( i , j ) 𝑖 𝑗 (i,j) ( italic_i , italic_j ) is denoted as c i ⁢ j subscript 𝑐 𝑖 𝑗 c_{ij} italic_c start_POSTSUBSCRIPT italic_i italic_j end_POSTSUBSCRIPT . The problem is then formulated as follows:

(3)
s.t. (4)
(5)
(6)
(7)
(8)
(9)
(10)
(11)

The objective function ( 3 ) minimizes the total routing cost for vehicles across all depots. Constraints ( 4 ) ensures that each customer is visited exactly once in the optimal solution. Flow conservation is enforced using constraints ( 5 ), while constraints ( 6 ) ensure that the total demand fulfilled by a vehicle does not exceed its capacity. Additionally, constraints ( 7 ) indicate that each vehicle belonging to a depot is utilized for routing at most once. Subtour elimination constraints are presented in ( 8 ). Constraints ( 9 ) prevent direct tours between depots, and constraints ( 10 ) distinctly partition the vehicles between depots. Our MDVRP formulation captures the fundamental aspects of vehicle routing with multiple depots and excludes additional constraints such as service times, depot capacities, and vehicle opening costs.

Next, we examine the decomposition of the MDVRP model in ( 3 )–( 11 ) into subproblems. As evident from the formulation, vehicles are distinctly categorized based on their assigned depots, and inter-depot routes are strictly prohibited. Thus, once customers are assigned to the depots, the problem decomposes into | 𝒟 | 𝒟 |\mathcal{D}| | caligraphic_D | distinct CVRP subproblems. Let 𝒞 P = { 𝒞 1 , 𝒞 2 , … , 𝒞 | 𝒟 | } superscript 𝒞 𝑃 subscript 𝒞 1 subscript 𝒞 2 … subscript 𝒞 𝒟 \mathcal{C}^{P}=\{\mathcal{C}_{1},\mathcal{C}_{2},\dots,\mathcal{C}_{|\mathcal% {D}|}\} caligraphic_C start_POSTSUPERSCRIPT italic_P end_POSTSUPERSCRIPT = { caligraphic_C start_POSTSUBSCRIPT 1 end_POSTSUBSCRIPT , caligraphic_C start_POSTSUBSCRIPT 2 end_POSTSUBSCRIPT , … , caligraphic_C start_POSTSUBSCRIPT | caligraphic_D | end_POSTSUBSCRIPT } be a partition of 𝒞 𝒞 \mathcal{C} caligraphic_C such that 𝒞 = 𝒞 1 ∪ 𝒞 2 ∪ ⋯ ∪ 𝒞 | 𝒟 | 𝒞 subscript 𝒞 1 subscript 𝒞 2 ⋯ subscript 𝒞 𝒟 \mathcal{C}=\mathcal{C}_{1}\cup\mathcal{C}_{2}\cup\cdots\cup\mathcal{C}_{|% \mathcal{D}|} caligraphic_C = caligraphic_C start_POSTSUBSCRIPT 1 end_POSTSUBSCRIPT ∪ caligraphic_C start_POSTSUBSCRIPT 2 end_POSTSUBSCRIPT ∪ ⋯ ∪ caligraphic_C start_POSTSUBSCRIPT | caligraphic_D | end_POSTSUBSCRIPT and 𝒞 i ∩ 𝒞 j = ∅ subscript 𝒞 𝑖 subscript 𝒞 𝑗 \mathcal{C}_{i}\cap\mathcal{C}_{j}=\emptyset caligraphic_C start_POSTSUBSCRIPT italic_i end_POSTSUBSCRIPT ∩ caligraphic_C start_POSTSUBSCRIPT italic_j end_POSTSUBSCRIPT = ∅ for 1 ≤ i < j ≤ | 𝒟 | 1 𝑖 𝑗 𝒟 1\leq i<j\leq|\mathcal{D}| 1 ≤ italic_i < italic_j ≤ | caligraphic_D | . Additionally, we assume that each depot serves at least one customer, i.e., | 𝒞 d | ≥ 1 , ∀ d ∈ 𝒟 formulae-sequence subscript 𝒞 𝑑 1 for-all 𝑑 𝒟 |\mathcal{C}_{d}|\geq 1,\forall d\in\mathcal{D} | caligraphic_C start_POSTSUBSCRIPT italic_d end_POSTSUBSCRIPT | ≥ 1 , ∀ italic_d ∈ caligraphic_D . The partition 𝒞 P superscript 𝒞 𝑃 \mathcal{C}^{P} caligraphic_C start_POSTSUPERSCRIPT italic_P end_POSTSUPERSCRIPT represents the distinct assignment of | 𝒞 | 𝒞 |\mathcal{C}| | caligraphic_C | customers among | 𝒟 | 𝒟 |\mathcal{D}| | caligraphic_D | depots. We assume that the partition is ordered so that 𝒞 d subscript 𝒞 𝑑 \mathcal{C}_{d} caligraphic_C start_POSTSUBSCRIPT italic_d end_POSTSUBSCRIPT is served by depot d 𝑑 d italic_d .

Let 𝒱 d = { d } ∪ 𝒞 d , ∀ d ∈ 𝒟 formulae-sequence subscript 𝒱 𝑑 𝑑 subscript 𝒞 𝑑 for-all 𝑑 𝒟 \mathcal{V}_{d}=\{d\}\cup\mathcal{C}_{d},\forall d\in\mathcal{D} caligraphic_V start_POSTSUBSCRIPT italic_d end_POSTSUBSCRIPT = { italic_d } ∪ caligraphic_C start_POSTSUBSCRIPT italic_d end_POSTSUBSCRIPT , ∀ italic_d ∈ caligraphic_D . With a partition 𝒞 P superscript 𝒞 𝑃 \mathcal{C}^{P} caligraphic_C start_POSTSUPERSCRIPT italic_P end_POSTSUPERSCRIPT , the original problem reduces to | 𝒟 | 𝒟 |\mathcal{D}| | caligraphic_D | CVRPs, and a subproblem d ∈ 𝒟 𝑑 𝒟 d\in\mathcal{D} italic_d ∈ caligraphic_D is represented as follows:

(12)
s.t. (13)
(14)
(15)
(16)
(17)
(18)

The subproblem formulation ( 12 )–( 18 ) for depot d 𝑑 d italic_d , obtained after decomposition, represents the well-known CVRP three-index flow formulation. By partitioning customers across different depots, the complexity of the original problem was significantly reduced due to the decrease in the number of feasible solutions. The quality of a decomposition strategy can be estimated based on the sum of the optimal routing costs of its subproblems. However, examining multiple partitions by solving all subproblems further increases computational difficulty. In this study, we introduce an approach to finding good higher-level decisions using a simple heuristic and machine learning. Note that a random partition 𝒞 P superscript 𝒞 𝑃 \mathcal{C}^{P} caligraphic_C start_POSTSUPERSCRIPT italic_P end_POSTSUPERSCRIPT may violate the vehicle capacity and fleet number constraints for one or more depots. Therefore, efforts must be made to carefully avoid any such infeasibility when possible. For instance, an undesirable partition occurs when the total demand of customers assigned to a depot surpasses the total vehicle capacity available at that depot. However, detecting feasibility based on higher-level decisions can be challenging and may only become apparent during subproblem resolution. A similar decomposition methodology can be constructed for the CLRP, which incorporates additional factors such as depot capacities, depot costs, and vehicle opening costs to evaluate the decomposition, along with the total cost of subproblems.

3.2 General Discussion

Although this is a direct decomposition approach, there has been limited research on related methodologies due to the challenge of evaluating multiple decomposition strategies by addressing several subproblems. Our solution to this issue involves training a supervised learning network to predict the optimal routing cost of a CVRP based on the input graph structure. Since the neural network is trained only to predict the near-optimal cost rather than finding the actual routing solution, we benefit from significant computational advantages. Consequently, to evaluate higher-level decisions, we need P ⁢ ( 𝐱 ) 𝑃 𝐱 P(\mathbf{x}) italic_P ( bold_x ) and do not directly require 𝐱 ∈ 𝐗 𝐱 𝐗 \mathbf{x}\in\mathbf{X} bold_x ∈ bold_X . The next section discusses the neural network architecture used to approximate the optimal cost of a subproblem.

4 Learning to predict CVRP Costs

Each route must start and end at the depot.

Each customer must belong to exactly one route.

The total demand of customers assigned to a route must not exceed the vehicle capacity Q 𝑄 Q italic_Q .

We assume homogeneity in the vehicle capacities and do not impose an upper bound on the number of vehicles. In the following section, we describe our cost prediction approach for CVRPs using a graph neural network (GNN). Using supervised learning, GNN is trained to predict the optimal CVRP cost estimated by the HGS-CVRP solver. An overview of the prediction model is presented in Figure 1 .

4.1 Cost Prediction using Graph Neural Network

Refer to caption

KNN Graph Construction

We transform an input CVRP instance 𝒢 𝒢 \mathcal{G} caligraphic_G into a graph-structured data 𝒢 knn subscript 𝒢 knn \mathcal{G}_{\text{knn}} caligraphic_G start_POSTSUBSCRIPT knn end_POSTSUBSCRIPT to better represent the node-level and edge-level features for effectively training the GNN. Here, 𝒢 knn subscript 𝒢 knn \mathcal{G}_{\text{knn}} caligraphic_G start_POSTSUBSCRIPT knn end_POSTSUBSCRIPT is a graph transformation of 𝒢 𝒢 \mathcal{G} caligraphic_G constructed using the k-nearest neighbor (KNN) method. During the KNN graph construction, each node is connected to its k 𝑘 k italic_k nearest neighbors based on the Euclidean distances. The node features 𝐔 𝐔 \mathbf{U} bold_U consists of two components: normalized node coordinates and normalized demand. The normalized node coordinates of 𝒢 knn subscript 𝒢 knn \mathcal{G}_{\text{knn}} caligraphic_G start_POSTSUBSCRIPT knn end_POSTSUBSCRIPT are computed by translating the node coordinates to ensure they are positive. This is achieved by adding the absolute value of the minimum coordinate values and then dividing them by the maximum coordinate value. The normalized demand is defined as the ratio of the demand q i subscript 𝑞 𝑖 q_{i} italic_q start_POSTSUBSCRIPT italic_i end_POSTSUBSCRIPT to the vehicle capacity Q 𝑄 Q italic_Q .

Graph Neural Network Architecture

The cost prediction GNN f 𝜽 subscript 𝑓 𝜽 f_{\bm{\theta}} italic_f start_POSTSUBSCRIPT bold_italic_θ end_POSTSUBSCRIPT with parameters 𝜽 𝜽 {\bm{\theta}} bold_italic_θ predicts the optimal cost of 𝒢 𝒢 \mathcal{G} caligraphic_G as v ^ ^ 𝑣 \hat{v} over^ start_ARG italic_v end_ARG . Our GNN architecture comprises the encoder, which consists of a stack of self-attention blocks (Vaswani et al., 2017 ) , and the decoder, a linear layer followed by the graph readout module. f 𝜽 subscript 𝑓 𝜽 f_{{\bm{\theta}}} italic_f start_POSTSUBSCRIPT bold_italic_θ end_POSTSUBSCRIPT predicts the cost as follows:

where Encoder ⁢ ( ⋅ ) Encoder ⋅ \texttt{Encoder}(\cdot) Encoder ( ⋅ ) and Decoder ⁢ ( ⋅ ) Decoder ⋅ \texttt{Decoder}(\cdot) Decoder ( ⋅ ) are described in the following paragraphs.

𝑁 1 ℎ \mathbf{U^{\prime}}\in\mathbb{R}^{(N+1)\times h} bold_U start_POSTSUPERSCRIPT ′ end_POSTSUPERSCRIPT ∈ blackboard_R start_POSTSUPERSCRIPT ( italic_N + 1 ) × italic_h end_POSTSUPERSCRIPT , where h ℎ h italic_h denotes the hidden node feature dimension. This transformation is achieved through a series of operations. First, a linear embedding layer is applied to match the input node feature dimension d ~ ~ 𝑑 \tilde{d} over~ start_ARG italic_d end_ARG to h ℎ h italic_h . Then, multiple self-attention blocks are employed to learn the attention scores with respect to the extracted node features. Each self-attention block consists of two steps: MHA ⁢ ( ⋅ ) MHA ⋅ \texttt{MHA}(\cdot) MHA ( ⋅ ) and FeedForward ⁢ ( ⋅ ) FeedForward ⋅ \texttt{FeedForward}(\cdot) FeedForward ( ⋅ ) . The Multi-Head Attention (MHA) mechanism divides the input into multiple heads and computes the attention for each head simultaneously. Each head may prioritize different aspects of the input data, leading to focused learning and improved accuracy.

The use of MHA in the graph encoder exhibits multiple benefits over other architectures such as Multilayer Perceptron (MLP). Firstly, MHA handles an arbitrary number of input nodes within a single model. In contrast, traditional networks such as MLP are incapable of handling variable-length data and this limits the generalizability of learning to predict a general VRP instance. Another significant advantage of MHA is its representational capability for complex graph data. By dynamically adjusting the strength of relationships among nodes based on their features, MHA can disregard the correlations between the nodes that do not impact the CVRP cost prediction. On the contrary, the prediction accuracy of conventional GNN architectures like Graph Convolutional Networks and Graph Attention Networks relies on the design of the input graph structure. The dynamic edge weight allocation of MHA mitigates the errors arising from suboptimal graph designs. Furthermore, MHA offers higher computational efficiency with matrix multiplication on modern computing frameworks compared to the message-passing frameworks in traditional GNNs, leading to faster training and inference.

If head i subscript head 𝑖 \text{head}_{i} head start_POSTSUBSCRIPT italic_i end_POSTSUBSCRIPT denotes the attention applied to 𝐔 𝐔 \mathbf{U} bold_U for i ∈ { 1 , 2 , … , H } 𝑖 1 2 … 𝐻 i\in\{1,2,\dots,H\} italic_i ∈ { 1 , 2 , … , italic_H } , and for a learnable parameter 𝐖 𝐎 ∈ ℝ H × h superscript 𝐖 𝐎 superscript ℝ 𝐻 ℎ \mathbf{W^{O}}\in\mathbb{R}^{H\times h} bold_W start_POSTSUPERSCRIPT bold_O end_POSTSUPERSCRIPT ∈ blackboard_R start_POSTSUPERSCRIPT italic_H × italic_h end_POSTSUPERSCRIPT , the MHA ⁢ ( ⋅ ) MHA ⋅ \texttt{MHA}(\cdot) MHA ( ⋅ ) step is defined as follows:

where the Attention ⁢ ( ⋅ ) Attention ⋅ \texttt{Attention}(\cdot) Attention ( ⋅ ) transformation is applied to each node u i subscript 𝑢 𝑖 u_{i} italic_u start_POSTSUBSCRIPT italic_i end_POSTSUBSCRIPT in 𝐔 𝐔 \mathbf{U} bold_U to compute the updated node feature u i ′ subscript superscript 𝑢 ′ 𝑖 u^{\prime}_{i} italic_u start_POSTSUPERSCRIPT ′ end_POSTSUPERSCRIPT start_POSTSUBSCRIPT italic_i end_POSTSUBSCRIPT , given by

where 𝐖 𝐐 superscript 𝐖 𝐐 \mathbf{W^{Q}} bold_W start_POSTSUPERSCRIPT bold_Q end_POSTSUPERSCRIPT , 𝐖 𝐊 superscript 𝐖 𝐊 \mathbf{W^{K}} bold_W start_POSTSUPERSCRIPT bold_K end_POSTSUPERSCRIPT , and 𝐖 𝐕 ∈ ℝ h × h superscript 𝐖 𝐕 superscript ℝ ℎ ℎ \mathbf{W^{V}}\in\mathbb{R}^{h\times h} bold_W start_POSTSUPERSCRIPT bold_V end_POSTSUPERSCRIPT ∈ blackboard_R start_POSTSUPERSCRIPT italic_h × italic_h end_POSTSUPERSCRIPT are the learnable parameters, 𝒩 ⁢ ( i ) 𝒩 𝑖 \mathcal{N}(i) caligraphic_N ( italic_i ) represents the set of neighbors of node i 𝑖 i italic_i , and softmax denotes the softmax function applied over j ∈ 𝒩 ⁢ ( i ) 𝑗 𝒩 𝑖 j\in\mathcal{N}(i) italic_j ∈ caligraphic_N ( italic_i ) . After the Multi-Head Attention (MHA) step, the updated node features 𝐔 ′ superscript 𝐔 ′ \mathbf{U^{\prime}} bold_U start_POSTSUPERSCRIPT ′ end_POSTSUPERSCRIPT are obtained. The attention mechanism is applied to the neighbors of each node, and the attention weights are computed based on the similarity between the node features. This approach allows the encoder to capture the local relationships between the nodes.

After accounting for the local relationships between the nodes, Encoder ⁢ ( ⋅ ) Encoder ⋅ \texttt{Encoder}(\cdot) Encoder ( ⋅ ) applies FeedForward ⁢ ( ⋅ ) FeedForward ⋅ \texttt{FeedForward}(\cdot) FeedForward ( ⋅ ) to process nodes independently. The FeedForward ⁢ ( ⋅ ) FeedForward ⋅ \texttt{FeedForward}(\cdot) FeedForward ( ⋅ ) operator is a multilayer perceptron model applied to each node feature. The updated node features 𝐔 ′ superscript 𝐔 ′ \mathbf{U^{\prime}} bold_U start_POSTSUPERSCRIPT ′ end_POSTSUPERSCRIPT are computed using the normalization layers LayerNorm ⁢ ( ⋅ ) LayerNorm ⋅ \texttt{LayerNorm}(\cdot) LayerNorm ( ⋅ ) (Ba et al., 2016 ) and residual connections (He et al., 2016 ) , which are applied to enhance the trainability of the deep models. To summarize, Encoder ⁢ ( ⋅ ) Encoder ⋅ \texttt{Encoder}(\cdot) Encoder ( ⋅ ) consists of the following sequential steps:

The Decoder ⁢ ( ⋅ ) Decoder ⋅ \texttt{Decoder}(\cdot) Decoder ( ⋅ ) operator subsequently transforms the latent vectors 𝐔 ′ superscript 𝐔 ′ \mathbf{U^{\prime}} bold_U start_POSTSUPERSCRIPT ′ end_POSTSUPERSCRIPT into the predicted optimal cost v ^ ^ 𝑣 \hat{v} over^ start_ARG italic_v end_ARG . This transformation involves a linear layer to project the latent vectors into a lower-dimensional space, followed by a graph readout module to aggregate the graph information. Decoder ⁢ ( ⋅ ) Decoder ⋅ \texttt{Decoder}(\cdot) Decoder ( ⋅ ) is defined as follows:

where 𝒱 𝒱 \mathcal{V} caligraphic_V represents the set of nodes in 𝒢 knn subscript 𝒢 knn \mathcal{G}_{\text{knn}} caligraphic_G start_POSTSUBSCRIPT knn end_POSTSUBSCRIPT , 𝟏 | 𝒱 | = ( 1 , 1 , … , 1 ) ∈ ℝ 1 × | 𝒱 | subscript 1 𝒱 1 1 … 1 superscript ℝ 1 𝒱 \mathbf{1}_{|\mathcal{V}|}=(1,1,\dots,1)\in\mathbb{R}^{1\times|\mathcal{V}|} bold_1 start_POSTSUBSCRIPT | caligraphic_V | end_POSTSUBSCRIPT = ( 1 , 1 , … , 1 ) ∈ blackboard_R start_POSTSUPERSCRIPT 1 × | caligraphic_V | end_POSTSUPERSCRIPT , and 𝐖 ∈ ℝ h × 1 𝐖 superscript ℝ ℎ 1 \mathbf{W}\in\mathbb{R}^{h\times 1} bold_W ∈ blackboard_R start_POSTSUPERSCRIPT italic_h × 1 end_POSTSUPERSCRIPT and 𝐛 ∈ ℝ | 𝒱 | × 1 𝐛 superscript ℝ 𝒱 1 \mathbf{b}\in\mathbb{R}^{|\mathcal{V}|\times 1} bold_b ∈ blackboard_R start_POSTSUPERSCRIPT | caligraphic_V | × 1 end_POSTSUPERSCRIPT are the scalar projection parameters. The graph readout module calculates a weighted average of the transformed latent vectors to produce a scalar output. This approach allows the decoder to capture the global relationships between the nodes.

4.2 Training

Existing CVRP benchmark datasets are insufficient in quantity to train this model, and thus, based on the problem requirements, CVRP instances need to be generated and solved to optimality (or near-optimality) to form the training data. An effective and widely used heuristic method for this problem is the Hybrid Genetic Search (HGS) for CVRP by Vidal ( 2022 ) , which has an open-source implementation. HGS-CVRP solver remains a top-performing metaheuristic in terms of methodological simplicity, solution quality, and convergence speed. We utilize a key feature of this solver by setting a time limit to compute the near-optimal routing costs for the train data, especially for larger instances.

Due to input data normalization, the final cost from average pooling needs to be rescaled by a corresponding factor to obtain the final cost prediction of the CVRP instance. We use the simple yet effective mean squared error as our loss function ( 19 ).

(19)

5 Solution Methodology for MDVRP

In this section, we propose our solution methodology to solve large-scale MDVRPs that involve the decomposition of the original problem into different CVRP subproblems. This approach aims to determine the optimal decomposition of the MDVRP into subsequent subproblems, where each CVRP instance corresponds to a depot of the main problem. Using an effective decomposition strategy, the desired CVRP subproblems can be obtained and solved using any existing method or solver. As CVRPs have been extensively studied, we do not attempt to develop a novel method for solving them. Instead, we utilize a well-established and validated heuristic solver to arrive at the final solution. An evaluation of all possible decomposition strategies could be computationally intractable. Therefore, we first select a set of random customer assignment strategies and improve them iteratively with a simplified genetic algorithm. Note that evaluating each MDVRP decomposition in the GA with an existing VRP solver is computationally expensive, and it defeats our goal of solving MDVRPs more effectively than the existing methods in the literature. With a supervised deep learning model, we predict the optimal routing cost of a CVRP instance almost instantly without actually solving it.

Refer to caption

\text{GANCP}^{+} GANCP start_POSTSUPERSCRIPT + end_POSTSUPERSCRIPT heuristic, such as the genetic algorithm, fitness estimation, and obtaining a final routing solution.

5.1 Genetic Algorithm for MDVRP decomposition

As an evolutionary approach, GAs represent possible solutions as a set of chromosomes that evolve towards optimality through a process of parent selection, crossover, and mutation over multiple generations. To apply GA, each chromosome is encoded as a string of data. We represent each individual as an N 𝑁 N italic_N -length chromosome of depot allocations that corresponds to customers i ∈ { 1 , 2 , … , N } 𝑖 1 2 … 𝑁 i\in\{1,2,\dots,N\} italic_i ∈ { 1 , 2 , … , italic_N } , as depicted in Figure 3 . The customer indices with the same depot allocation d 𝑑 d italic_d are selected to form the CVRP with depot d 𝑑 d italic_d .

Algorithm 1 gives a comprehensive overview of our method, and it begins with the generation of an initial population that consists of multiple MDVRP decompositions. Random assignments of customers to depots generate a diverse initial population, and it ensures that the population starts at a high level of diversity, leading to a larger solution search space over time. Besides this rule, we also initialize two or three potential solutions based on the input graph structure. Our study found that a combination of targeted and random assignments in the initial population results in faster algorithm convergence. One effective targeted assignment strategy is the nearest depot assignment (NDA), where the customers are allocated to their nearest depot. The second strategy is to allocate each customer to the closest neighbor’s nearest depot. These initial assignment rules have been discussed in de Oliveira et al. ( 2016 ) . Additionally, we initialize a chromosome where customers are allocated to their second nearest depot for problems with more than two depots. Targeted assignments serve as an effective initial solution to problems involving uniform customer distribution in a region. Metaheuristics can improve such initial solutions and achieve near-optimal results.

During the solution search, the genetic algorithm may encounter infeasible candidates. While all the infeasible candidates cannot be determined during the cost prediction phase of our heuristic, we can detect candidates with a high degree of infeasibility. For example, a VRP is evidently infeasible if the total demand exceeds the total vehicle capacity. Such individuals are repaired by selecting a random customer from the infeasible CVRP and reassigning them to a depot that can fulfill the corresponding demand, as detailed in Algorithm 2 .

The next generation of the population is propagated using multiple iterations. At each iteration, two distinct parents are selected from the current solution pool based on fitness scores using a K 𝐾 K italic_K tournament selection method. As larger tournament sets select better chromosomes but require more computation and may result in premature convergence, we use the binary tournament selection procedure. To generate children, uniform crossover operations are performed over the selected parents by randomly selecting offspring genes from the corresponding genes of the parents. Our simple chromosome representation requires minimal information about the routing solution, so more sophisticated crossover operations are not necessary. The population size is maintained in the range [ P L , P H ] subscript 𝑃 𝐿 subscript 𝑃 𝐻 [P_{L},P_{H}] [ italic_P start_POSTSUBSCRIPT italic_L end_POSTSUBSCRIPT , italic_P start_POSTSUBSCRIPT italic_H end_POSTSUBSCRIPT ] . Mutations are performed once crossover operations have generated P L subscript 𝑃 𝐿 P_{L} italic_P start_POSTSUBSCRIPT italic_L end_POSTSUBSCRIPT chromosomes for the next generation.

Mutations include the random reset of a customer’s depot called FLIP , and the SWAP operation in which two selected customers’ depots are interchanged. Based on their fitness score, chromosomes in the first tertile are selected for mutation. Approximately 5 % percent 5 5\% 5 % of all customers undergo a mutation, resulting in either a FLIP or SWAP operation with a probability of p m subscript 𝑝 𝑚 p_{m} italic_p start_POSTSUBSCRIPT italic_m end_POSTSUBSCRIPT or 1 − p m 1 subscript 𝑝 𝑚 1-p_{m} 1 - italic_p start_POSTSUBSCRIPT italic_m end_POSTSUBSCRIPT , respectively. Additional targeted mutations are also carried out over 5 % percent 5 5\% 5 % of the child population using a binary tournament selection process. As part of targeted mutations, 10 % percent 10 10\% 10 % of the customers are randomly selected, and the depot allocations of these individuals are copied from one of the elite individuals within the initial population of targeted assignments.

5.2 NN Estimation and Fitness function

A candidate solution generated by the GA should be evaluated using a measure of fitness to guide the population toward the direction of minimum cost. In the MDVRP, the solution cost is simply the sum of the optimal routing costs for its CVRP subproblems. These costs are predicted using the NN prediction model trained with parameters 𝜽 𝜽 {\bm{\theta}} bold_italic_θ . Although our objective is to minimize the routing cost of the main problem, the application of the MDVRP cost directly as a measure of fitness may lead to premature convergence. With a score for the diversity of a solution in the population, in addition to the predicted cost, which measures solution quality, the GA expands its search space and often overcomes local optima. Hence, the fitness of an individual can be determined using the predicted MDVRP cost ( 21 ) and the diversity factor ( 20 ). The diversity factor of a chromosome is measured as the mean Hamming distance of the individual from the rest of the population, as shown in ( 20 ). The fitness score fit ⁢ ( I ) fit 𝐼 \text{fit}(I) fit ( italic_I ) ( 22 ) for a chromosome I 𝐼 I italic_I is calculated as the weighted sum of normalized solution cost C ~ ~ 𝐶 \tilde{C} over~ start_ARG italic_C end_ARG , normalized diversity factor δ ~ ~ 𝛿 \tilde{\delta} over~ start_ARG italic_δ end_ARG , and a penalty for the extent of approximate vehicle capacity violations. Note that l d subscript 𝑙 𝑑 l_{d} italic_l start_POSTSUBSCRIPT italic_d end_POSTSUBSCRIPT , m d subscript 𝑚 𝑑 m_{d} italic_m start_POSTSUBSCRIPT italic_d end_POSTSUBSCRIPT , and Q 𝑄 Q italic_Q represent the total demand, the number of available vehicles, and homogeneous vehicle capacity at the depot d 𝑑 d italic_d , respectively.

(20)
(21)

(22)

Newly generated chromosomes from crossovers and mutations are evaluated in batches to find the MDVRP cost predictions, and duplicate solutions are eliminated before the batch processing. When the CVRP subproblems from multiple chromosomes are processed together with the NN model, the computational time reduces significantly. Following this, fitness scores are calculated for the new generation. According to the fitness scores, a group of P H subscript 𝑃 𝐻 P_{H} italic_P start_POSTSUBSCRIPT italic_H end_POSTSUBSCRIPT individuals are selected and represent the next generation. To prevent wide divergences from our objective, the top 1 % percent 1 1\% 1 % of individuals from each generation is preserved for the subsequent generation. This process is repeated until there is no improvement over multiple generations or up to the desired number of generations.

5.3 Determining the final solution by HGS-CVRP

\text{GANCP}^{+} GANCP start_POSTSUPERSCRIPT + end_POSTSUPERSCRIPT .

6 Computational Study for MDVRP

\text{GANCP}^{+} GANCP start_POSTSUPERSCRIPT + end_POSTSUPERSCRIPT is adapted to minimize the number of Python calls by employing batch estimation of VRP costs. Our code is publicly available at https://github.com/abhaysobhanan/GANCP .

6.1 NN Training

Refer to caption

A common approach for training the proposed NN on CVRP data is to randomly distribute customers across a grid. However, we demonstrate that this method is inadequate for accurately predicting the subproblems arising from optimal MDVRP decompositions. To address this, we conduct three distinct training sessions on the proposed NN, each utilizing a different data generation technique, referred to as a phase. In Phase I, the VRP instances are generated following the rules of Uchoa et al. ( 2017 ) with randomly placed depots and customers, where coordinates are integers from a grid of size [ 0 , 1000 ] × [ 0 , 1000 ] 0 1000 0 1000 [0,1000]\times[0,1000] [ 0 , 1000 ] × [ 0 , 1000 ] and demands are integers in the range [ 1 , 100 ] 1 100 [1,100] [ 1 , 100 ] . Phase I follows the common approach discussed above and is effective for estimating the cost of randomly allocating customers to depots. The learning curves for both the training and validation data are illustrated in Figure 4 . While this trained model provides good cost estimates for the initial MDVRP solutions, the prediction error increases significantly as the GA propagates toward optimality. Figure 5 depicts the testing accuracy of Phase-I with an average mean absolute percentage error (MAPE) of 1.06 % percent 1.06 1.06\% 1.06 % , and shows that our NN model outperforms ( 1 ) and ( 2 ) significantly. We choose C = 6.52 𝐶 6.52 C=6.52 italic_C = 6.52 in ( 1 ) to minimize the average error across the test data. Similarly, the regression parameters in ( 2 ) are obtained using the same train data in Phase I.

In Phase II, the VRP subproblem instances are generated from random MDVRP instances to incorporate the near-optimal cases seen during our methodology. 80 % percent 80 80\% 80 % of the training data is generated using the two targeted assignment rules, out of which in 70 % percent 70 70\% 70 % of the instances, exceptions are made to the targeted assignments by randomly reallocating up to 10 % percent 10 10\% 10 % of the customers. The remaining 20 % percent 20 20\% 20 % of the instances follow Phase I data generation. Phase II involves targeted train data based on a partial understanding of our real data distribution. As a result of comparing multiple assignments, MDVRP solutions are better represented, and cost prediction errors are reduced towards optimal decompositions.

In the final training phase, i.e., Phase III, we generate train/validation data in four steps using random MDVRP instances in conjunction with GANCP . In the first step, the NN prediction model is initialized with randomized parameters 𝜽 𝜽 {\bm{\theta}} bold_italic_θ . For an input CVRP, this untrained model is incapable of a fair cost prediction and provides any real number as the output based on 𝜽 𝜽 {\bm{\theta}} bold_italic_θ . Therefore, the final solutions from GANCP are random assignments. Multiple MDVRP instances are utilized to generate CVRP subproblems. After each step, the VRP subproblems generated based on the existing 𝜽 𝜽 {\bm{\theta}} bold_italic_θ are evaluated with the HGS-CVRP solver. This serves as the train data for the NN model, and the parameter values are updated. In the next step, the improved NN model parameters are used in GANCP with random MDVRP instances to generate the new VRP decompositions. This process of NN parameter estimation using partial train data is repeated until the NN model is trained in the fourth step. For each MDVRP evaluation, the top 2 2 2 2 solutions using GANCP are selected to produce the corresponding CVRP subproblems. Each step contributes to the generation of 1 / 4 1 4 1/4 1 / 4 -th of the total train/validation data. At each step, the generated data is used for training in combination with data from previous steps, if applicable. This enhances the capability of the NN model to evaluate the CVRP instances arising from various assignment patterns observed during our heuristic.

# Nodes (%)
) ) Phase I Phase II Phase III
[50,100] 27.01 11.09 1.99 3.58 3.15
[101,150] 22.22 11.08 1.40 2.65 2.61
[151,200] 17.09 11.44 1.16 2.25 2.42
[201,250] 13.13 11.94 0.99 2.00 2.16
[251,300] 11.79 11.75 0.88 1.90 2.04
[301,350] 11.92 11.97 0.83 1.82 2.01
[351,400] 13.49 12.23 0.77 1.85 1.97
[401,450] 15.54 12.15 0.72 1.81 1.96
[451,500] 19.25 12.18 0.72 1.88 1.92

6.2 Test instances

\text{GANCP}^{+} GANCP start_POSTSUPERSCRIPT + end_POSTSUPERSCRIPT methodology, 150 150 150 150 MDVRP Test (T) instances are generated in the range N ∈ [ 100 , 1500 ] 𝑁 100 1500 N\in[100,1500] italic_N ∈ [ 100 , 1500 ] and D ∈ [ 2 , 10 ] 𝐷 2 10 D\in[2,10] italic_D ∈ [ 2 , 10 ] . The number of depots for each problem is chosen so that the approximate CVRP subproblem size falls within the range of trained data, i.e., N D ∈ [ 50 , 500 ] 𝑁 𝐷 50 500 \frac{N}{D}\in[50,500] divide start_ARG italic_N end_ARG start_ARG italic_D end_ARG ∈ [ 50 , 500 ] . Instances are generated based on random customer and depot positioning in a grid of size [ 0 , 1000 ] × [ 0 , 1000 ] 0 1000 0 1000 [0,1000]\times[0,1000] [ 0 , 1000 ] × [ 0 , 1000 ] , while demands are integers in the range of [ 1 , 100 ] 1 100 [1,100] [ 1 , 100 ] . The NN models from each phase undergo testing using 20,000 random CVRP instances, and the results are summarized in Table  1 for different problem sizes. The performance gap g H = Predicted Cost − HGS Cost HGS Cost × 100 subscript 𝑔 𝐻 Predicted Cost HGS Cost HGS Cost 100 g_{H}=\frac{\text{Predicted Cost}-\text{HGS Cost}}{\text{HGS Cost}}\times 100 italic_g start_POSTSUBSCRIPT italic_H end_POSTSUBSCRIPT = divide start_ARG Predicted Cost - HGS Cost end_ARG start_ARG HGS Cost end_ARG × 100 is calculated in comparison to the HGS-CVRP solver, and the absolute values | g H | subscript 𝑔 𝐻 |g_{H}| | italic_g start_POSTSUBSCRIPT italic_H end_POSTSUBSCRIPT | are reported to consider both overestimation and underestimation.

\text{GANCP}^{+} GANCP start_POSTSUPERSCRIPT + end_POSTSUPERSCRIPT obtains a better quality solution compared to the benchmark solver. The details regarding the experiments on T instances and the benchmark solver will be discussed subsequently. While Phase I demonstrates strong performance for random CVRP data, the training falls short in incorporating VRPs that arise from the near-optimal MDVRP decompositions. Given that Phase III excels for our MDVRP heuristic, the remaining experiments will focus on the NN model formed using the Phase III parameters, unless stated otherwise. Note that in the event of limited training time availability, Phase II can be used to obtain satisfactory results.

Train Data Average Performance Gap (%)
CVRP T Instances

Phase I 1.06 0.50 -0.49
Phase II 2.21 -0.09 -0.47
Phase III 2.26 -0.58 -0.89

Algorithm Settings

The computational requirements of the HGS-CVRP solver increase as the size of the problem increases. The time limits for the HGS-CVRP solver based on approximate subproblem size, shown in Table 3 , are used to ensure better routing solutions for the top candidates obtained from GANCP . Additionally, the number of CVRP instances that can be processed by the NN model in a batch depends on the memory limitations of the computer. On the specified computer, the batch processing is executed in units as specified in Table 3 to avoid memory overflows.

Approximate NN batch size HGS time
subproblem size (sec)
512 0.1
256 1.0
128 2.0
64 2.0
32 2.0

Benchmark Algorithm

\text{GANCP}^{+} GANCP start_POSTSUPERSCRIPT + end_POSTSUPERSCRIPT performs relatively poorly for the out-of-distribution Cordeau et al. ( 1997 ) instances, with a maximum gap of 4.61 % percent 4.61 4.61\% 4.61 % . The best-known results are also compared with an open-source solver VROOM (Coupey et al., 2023 ) on the same machine running Ubuntu 22.04.1, and the results indicate that VROOM is a competitive solver for the MDVRP instances and yields high-quality solutions within a short time frame. To ensure that the comparison between the benchmark MDVRP solver and the use of our NN model on a GPU is fair, we run VROOM using 16 CPU threads on the same machine for all future experiments. Through the use of multiple CPU threads, we improve the benchmark solver’s computational speed.

\text{GANCP}^{+} GANCP start_POSTSUPERSCRIPT + end_POSTSUPERSCRIPT consistently outperformed NDA with a gap of -1.77%. In the K-Means-10 approach, customers are grouped into D 𝐷 D italic_D clusters based on randomized centroids. After forming the clusters, each one is assigned distinctly to a depot based on the Euclidean distance between the centroids and the depots. This process is repeated 10 times, and the best cost and total time are reported.

NDA VROOM GANCP

Instance Best Cost Gap Cost Time Gap NN Cost Time Cost Time Gap
Known (%) (sec) (%) (sec) (sec) (%)
p01-n50-d4 576.87 609.24 5.61 576.87 0.15 0.00 697.00 1.64 586.39 3.64 1.65
p02-n50-d4 473.53 507.01 7.07 476.66 0.08 0.66 593.56 1.61 495.38 3.61 4.61
p03-n75-d5 641.19 681.72 6.32 647.89 0.28 1.04 844.65 2.23 662.14 4.73 3.27
p04-n100-d2 1001.04 1016.64 1.56 1013.50 0.52 1.24 1121.68 1.16 1013.37 2.16 1.23
p05-n100-d2 750.03 774.33 3.24 759.05 0.52 1.20 876.46 1.24 763.90 2.24 1.85
p06-n100-d3 876.50 890.35 1.58 884.67 0.55 0.93 1026.63 1.60 885.92 3.11 1.07
p07-n100-d4 881.97 - - 901.82 0.48 2.25 1102.56 1.24 916.65 3.24 3.93

\text{GANCP}^{+} GANCP start_POSTSUPERSCRIPT + end_POSTSUPERSCRIPT is 11.4 times faster than VROOM, with an observed performance gap of − 0.39 % percent 0.39 -0.39\% - 0.39 % . With a shorter time limit, VROOM often produces the same solutions as the full run but fails to improve the earlier solutions. NDA yields good quality solutions for the T instances, sometimes even better than VROOM. This is because VROOM, like most solvers, is tailored for medium-sized problems. This is evident from Table 4 and the smallest problem size considered in Table 5 , where the NDA exhibits relatively poor performance. Our GANCP+ framework outperforms NDA without imposing a significant computational time requirement, as compared to VROOM, even for large instances that follow a uniform distribution and seldom exhibit infeasibility issues with the NDA. Note that employing a random clustering approach such as K-Means-10 may result in low-quality solutions.

# Nodes NDA K-Means-10 VROOM Best of 10 runs
Cost Cost Time Cost Time

(sec) (sec) (%) (%) (%)
[100,200] 29675.75 31643.21 3.42 29498.94 1.66 29321.03 -1.20 -7.34 -0.60
[201,300] 41068.73 48338.13 8.45 41080.90 4.58 40651.23 -1.02 -15.90 -1.05
[301,400] 48829.56 57748.89 11.99 48987.85 9.85 48434.45 -0.81 -16.13 -1.13
[401,500] 58144.47 78762.95 14.01 58326.76 22.28 57719.28 -0.73 -26.72 -1.04
[501,600] 75715.25 102142.93 26.12 75692.72 48.52 75129.11 -0.77 -26.45 -0.74
[601,700] 78428.69 119417.80 32.54 78687.05 102.69 77930.80 -0.63 -34.74 -0.96
[701,800] 101372.52 138626.28 51.48 101334.56 155.25 100516.96 -0.84 -27.49 -0.81
[801,900] 101151.53 140426.73 50.61 101510.61 228.24 100544.05 -0.60 -28.40 -0.95
[901,1000] 117989.13 166262.61 59.84 118042.43 330.17 117118.12 -0.74 -29.56 -0.78
[1001,1100] 125657.77 181945.18 77.62 125837.19 453.42 124800.93 -0.68 -31.41 -0.82
[1101,1200] 123575.23 182073.25 75.59 124239.53 551.04 122851.90 -0.59 -32.53 -1.12
[1201,1300] 141202.37 212741.86 81.78 141516.74 752.71 140339.98 -0.61 -34.03 -0.83
[1301,1400] 155926.93 238823.50 82.95 155932.46 923.81 155101.03 -0.53 -35.06 -0.53
[1401,1500] 167590.06 228078.03 96.12 167286.25 1110.83 166641.89 -0.57 -26.94 -0.39
# Nodes VROOM- Average of 10 runs
Cost Time GANCP Time

Time
(sec) (sec) (sec) (%) (%) (%)
[100,200] 29498.94 1.63 29117.09 1.65 29506.57 2.90 -0.57 -6.75 0.03
[201,300] 41080.90 4.24 40341.34 3.10 40806.03 7.15 -0.64 -15.58 -0.67
[301,400] 48987.85 9.14 48376.34 5.51 48604.29 11.24 -0.46 -15.84 -0.78
[401,500] 58347.64 15.88 57233.29 8.75 57929.15 15.42 -0.37 -26.45 -0.68
[501,600] 74519.29 25.28 74756.34 10.61 75345.94 23.19 -0.49 -26.23 -0.46
[601,700] 78745.10 34.38 78006.76 14.64 78170.21 30.29 -0.33 -34.54 -0.66
[701,800] 101364.25 47.82 99638.54 16.96 100795.98 41.97 -0.57 -27.29 -0.53
[801,900] 101510.61 52.40 100531.98 21.01 100784.26 45.56 -0.36 -28.23 -0.72
[901,1000] 118045.20 63.88 116908.49 24.98 117343.41 54.00 -0.55 -29.42 -0.59
[1001,1100] 125837.19 80.52 124396.17 29.29 125074.06 67.32 -0.46 -31.26 -0.61
[1101,1200] 124255.41 87.06 122553.92 36.35 123121.97 73.37 -0.37 -32.38 -0.90
[1201,1300] 141564.49 101.18 140751.63 42.02 140683.88 82.05 -0.37 -33.87 -0.59
[1301,1400] 155932.46 108.11 156527.15 47.33 155526.77 87.82 -0.26 -34.88 -0.26
[1401,1500] 167286.57 119.85 166412.10 50.08 166956.54 97.05 -0.38 -26.80 -0.20

\text{GANCP}^{+} GANCP start_POSTSUPERSCRIPT + end_POSTSUPERSCRIPT can be enhanced by directly incorporating the NDA solution, if feasible, as a top solution of GANCP. In the subsequent sections, we demonstrate the robustness of our method against data variations, in comparison to VROOM. Additionally, we showcase the transferability of our trained NN model to the CLRP in the appendix.

6.3 Out-of-range Data

We further evaluate the predictive accuracy of our method for MDVRP instances with approximate subproblem sizes that fall outside the range of the trained data. A summary of the results from 50 randomly generated Out-of-range (O) instances with N D ∉ [ 50 , 500 ] 𝑁 𝐷 50 500 \frac{N}{D}\notin[50,500] divide start_ARG italic_N end_ARG start_ARG italic_D end_ARG ∉ [ 50 , 500 ] is presented in Table 7 . The results indicate that our method outperforms VROOM for problems with N ∈ [ 101 , 250 ] 𝑁 101 250 N\in[101,250] italic_N ∈ [ 101 , 250 ] . This is likely due to the model’s exposure to some out-of-range VRP instances during the stepwise training procedure and its ability to learn from them. Our method fails to outperform VROOM for relatively smaller and larger problems in the O instances dataset. Nevertheless, the results can be considered satisfactory due to the relatively small positive gap.

# Nodes VROOM Average of 10 runs Best of 10 runs
Cost Time GANCP

Time

(sec) (sec) (%) (%)
[50,100] 16371.24 0.23 16328.06 16513.03 2.15 0.94 16467.98 0.66
[101,150] 22040.55 0.80 22021.44 22027.67 3.65 -0.05 21930.31 -0.49
[151,200] 25161.54 1.74 24994.17 25268.67 4.92 0.48 25108.81 -0.18
[201,250] 35356.99 4.21 35150.75 35246.59 6.55 -0.33 34999.57 -1.03
[1000,1500] 222183.17 330.73 220043.57 223942.65 56.88 0.77 223747.96 0.68

6.4 Out-of-distribution data

The robustness of our method in association with the trained NN model is tested for multiple combinations of coordinates and demand distributions. Uchoa et al. ( 2017 ) uses similar customer positioning and demand distributions to generate a class of VRP benchmark instances. The positioning of customers is determined using three different rules, namely Random (R), Clustered (C), and Random-Clustered (RC), as described in Solomon ( 1987 ) VRPTW instances. In C, a few randomly positioned customers s ∈ UD ⁢ [ 3 , 8 ] 𝑠 UD 3 8 s\in\text{UD}[3,8] italic_s ∈ UD [ 3 , 8 ] act as cluster seeds to attract N − s 𝑁 𝑠 N-s italic_N - italic_s customers with an exponential decay. In RC, half of the customers are randomly positioned, and the remaining are clustered as in C. Two different types of demand distributions such as Unitary (U), where each demand is equal to 1, and Quadrant-Dependent (Q), where demand belongs to UD ⁢ [ 1 , 50 ] UD 1 50 \text{UD}[1,50] UD [ 1 , 50 ] for even quadrant and UD ⁢ [ 51 , 100 ] UD 51 100 \text{UD}[51,100] UD [ 51 , 100 ] for odd quadrant customers, respectively, are considered for evaluation. Our T instances were generated using R customer positioning within the grid and demand from UD ⁢ [ 1 , 100 ] UD 1 100 \text{UD}[1,100] UD [ 1 , 100 ] . Depots are assumed to take a random position on the grid in all cases. Using the above rules, eight classes of out-of-distribution MDVRP instances are generated, as shown in Table 8 .

Instance Class Customer Positioning Demand Distribution
D1 C UD[1,100]
D2 RC UD[1,100]
D3 R U
D4 R Q
D5 C U
D6 C Q
D7 RC U
D8 RC Q
Instance VROOM Average of 10 runs Best of 10 runs
Class Cost Time GANCP

Time

(sec) (sec) (%) (%)
D1 77202.13 389.16 79405.89 77606.12 39.69 0.62 77347.84 0.20
D2 80189.65 173.94 79777.80 80365.79 29.34 0.08 80178.28 -0.19
D3 82998.27 235.70 86458.29 83043.41 35.46 0.06 82900.18 -0.15
D4 85577.51 288.23 85711.91 85326.44 40.43 -0.26 85002.43 -0.69
D5 54606.09 106.43 59134.12 55126.99 22.23 1.28 54847.38 0.56
D6 63872.70 330.80 67575.29 64184.02 37.20 0.49 63903.97 0.06
D7 77442.77 98.38 81024.07 77722.09 26.95 0.49 77527.93 0.23
D8 67933.00 126.03 68419.02 68330.90 22.05 0.80 68060.73 0.17

\text{GANCP}^{+} GANCP start_POSTSUPERSCRIPT + end_POSTSUPERSCRIPT for the eight instance classes, each of different distributions, can be found in Table 9 . Given that our NN model is trained over uniform distributions, the results for out-of-distribution instances are satisfactory, despite the positive performance gap compared to the solver in some cases. A noticeable difference is observed for D4 instances, where customer positioning and demand distribution resemble our T instances with minor modifications. Here, the average result of our heuristics in 10 runs outperforms VROOM. Similarly, the maximum deviation from optimality is visible for instances in class D5 where customer locations are purely clustered, and all demands are assumed to be unitary. Moreover, we still retain a considerable computational advantage for out-of-distribution data, while maintaining the ability to achieve satisfactory results.

Instance NN Train Average of 10 runs Best of 10 runs
Class MAPE GANCP

(%) (%) (%)
D1 2.96 80415.02 77698.81 0.73 77321.69 0.06
D2 2.63 80811.31 80389.15 0.12 80122.80 -0.24
D3 2.33 85363.50 83021.85 0.05 82890.92 -0.17
D4 2.47 85458.10 85388.37 -0.20 85011.12 -0.73
D5 2.77 58020.85 55080.95 1.16 54793.58 0.46
D6 3.25 67482.74 64234.44 0.88 63825.88 -0.06
D7 2.39 79556.12 77695.78 0.44 77524.72 0.15
D8 2.65 67549.30 68311.91 0.71 68030.88 0.10

\text{GANCP}^{+} GANCP start_POSTSUPERSCRIPT + end_POSTSUPERSCRIPT decreases compared to Table 9 . However, fine-tuning the NN model with partial train data leads to improvements in multiple distribution classes. Remarkably, in all cases, the best cost from 10 experiments improved in comparison to the experiments with pretrained NN parameters. This underscores the potential to enhance the average performance accuracy of our solution framework by incorporating additional data, in the event of a change in data trend.

7 Conclusions

\text{GANCP}^{+} GANCP start_POSTSUPERSCRIPT + end_POSTSUPERSCRIPT . Numerical experiments show that our approach is effective in obtaining good solutions for large-scale data with significant computational advantages in comparison to VROOM, an effective open-source generic VRP solver. We further benchmark our results using simple decision rules that decompose the problem into CVRP subproblems and also discuss the drawbacks of employing such strategies in practice. The accuracy of the prediction model depends on the similarity of training and testing data distributions and problem size. Our experiments on out-of-distribution and out-of-range instances show that the results are satisfactory for practical execution with a significant advantage in computational time. A direct extension of this work is to consider route duration constraints for the MDVRP. Additionally, a supervised learning model can be developed to more accurately detect infeasible solutions, thereby enhancing the performance of our heuristic. The idea of subproblem cost prediction is a promising research direction and can be extended to multiple optimization problems such as the Orienteering Problem, Prize-Collecting VRP, Periodic VRP, VRP with time windows, Split Delivery VRP, and CLRP. We verify this idea by using the existing trained NN model to evaluate the CLRP benchmark datasets. It further validates the transferability of our heuristic and pre-trained NN model for a more generalized hierarchical VRP and still leads to satisfactory results.

Acknowledgement

This research was supported by the Korean government (MSIT) through the National Research Foundation of Korea (NRF) grant RS-2023-00259550. Disclaimer: Any opinions, findings, conclusions, or recommendations expressed in this material are those of the authors.

  • Akkerman and Mes (2022) Akkerman, F., M. Mes. 2022. Distance approximation to support customer selection in vehicle routing problems. Annals of Operations Research 1–29.
  • Akpunar and Akpinar (2021) Akpunar, Ö. Ş., Ş. Akpinar. 2021. A hybrid adaptive large neighbourhood search algorithm for the capacitated location routing problem. Expert Systems with Applications 168 114304.
  • Arnold et al. (2019) Arnold, F., M. Gendreau, K. Sörensen. 2019. Efficiently solving very large-scale routing problems. Computers & operations research 107 32–42.
  • Ba et al. (2016) Ba, J. L., J. R. Kiros, G. E. Hinton. 2016. Layer normalization. Advances in NIPS 2016 Deep Learning Symposium . (p. arXiv preprint arXiv:1607.06450).
  • Baldacci and Mingozzi (2009) Baldacci, R., A. Mingozzi. 2009. A unified exact method for solving different classes of vehicle routing problems. Mathematical Programming 120 347–380.
  • Baldacci et al. (2011) Baldacci, R., A. Mingozzi, R. Wolfler Calvo. 2011. An exact method for the capacitated location-routing problem. Operations Research 59 (5) 1284–1296.
  • Banerjee et al. (2022) Banerjee, D., A. L. Erera, A. Toriello. 2022. Fleet sizing and service region partitioning for same-day delivery systems. Transportation Science 56 (5) 1327–1347.
  • Barreto (2004) Barreto, S. d. S. 2004. Análise e modelização de problemas de localização-distribuição [analysis and modelling of location-routing problems]. Unpublished doctoral dissertation, University of Aveiro, Campus Universitário de Santiago 3810–193.
  • Belenguer et al. (2011) Belenguer, J.-M., E. Benavent, C. Prins, C. Prodhon, R. W. Calvo. 2011. A branch-and-cut method for the capacitated location-routing problem. Computers & Operations Research 38 (6) 931–941.
  • Bello et al. (2017) Bello, I., H. Pham, Q. V. Le, M. Norouzi, S. Bengio. 2017. Neural combinatorial optimization with reinforcement learning. 5th International Conference on Learning Representations, ICLR 2017, Toulon, France, April 24-26, 2017, Workshop Track Proceedings . OpenReview.net. URL https://openreview.net/forum?id=Bk9mxlSFx .
  • Bogyrbayeva et al. (2023) Bogyrbayeva, A., T. Yoon, H. Ko, S. Lim, H. Yun, C. Kwon. 2023. A deep reinforcement learning approach for solving the traveling salesman problem with drone. Transportation Research Part C: Emerging Technologies 148 103981.
  • CapGemini Research Institute (2019) CapGemini Research Institute. 2019. The last-mile delivery challenge. Tech. rep., CapGemini Research Institute.
  • Capital One Shopping Research (2019) Capital One Shopping Research. 2019. Amazon logistics statistics. Tech. rep., Capital One Shopping Research.
  • Cappart et al. (2023) Cappart, Q., D. Chételat, E. B. Khalil, A. Lodi, C. Morris, P. Veličković. 2023. Combinatorial optimization and reasoning with graph neural networks. Journal of Machine Learning Research 24 (130) 1–61.
  • Contardo et al. (2014) Contardo, C., J.-F. Cordeau, B. Gendron. 2014. An exact algorithm based on cut-and-column generation for the capacitated location-routing problem. INFORMS Journal on Computing 26 (1) 88–102.
  • Contardo and Martinelli (2014) Contardo, C., R. Martinelli. 2014. A new exact algorithm for the multi-depot vehicle routing problem under capacity and route length constraints. Discrete Optimization 12 129–146.
  • Cordeau et al. (1997) Cordeau, J.-F., M. Gendreau, G. Laporte. 1997. A tabu search heuristic for periodic and multi-depot vehicle routing problems. Networks: An International Journal 30 (2) 105–119.
  • Coupey et al. (2023) Coupey, J., J.-M. Nicod, C. Varnier. 2023. VROOM v1.13, Vehicle Routing Open-source Optimization Machine. http://vroom-project.org/ (Last Accessed: October 1, 2023).
  • de Oliveira et al. (2016) de Oliveira, F. B., R. Enayatifar, H. J. Sadaei, F. G. Guimarães, J.-Y. Potvin. 2016. A cooperative coevolutionary algorithm for the multi-depot vehicle routing problem. Expert Systems with Applications 43 117–130.
  • Ellegood et al. (2015) Ellegood, W. A., J. F. Campbell, J. North. 2015. Continuous approximation models for mixed load school bus routing. Transportation Research Part B: Methodological 77 182–198.
  • Figliozzi (2008) Figliozzi, M. A. 2008. Planning approximations to the average length of vehicle routing problems with varying customer demands and routing constraints. Transportation Research Record 2089 (1) 1–8.
  • Franceschetti et al. (2017) Franceschetti, A., O. Jabali, G. Laporte. 2017. Continuous approximation models in freight distribution management. Top 25 413–433.
  • Furian et al. (2021) Furian, N., M. O’Sullivan, C. Walker, E. Çela. 2021. A machine learning-based branch and price algorithm for a sampled vehicle routing problem. OR Spectrum 43 693–732.
  • Garn (2021) Garn, W. 2021. Balanced dynamic multiple travelling salesmen: Algorithms and continuous approximations. Computers & Operations Research 136 105509.
  • Ghaffarinasab et al. (2018) Ghaffarinasab, N., T. Van Woensel, S. Minner. 2018. A continuous approximation approach to the planar hub location-routing problem: Modeling and solution algorithms. Computers & Operations Research 100 140–154.
  • Golden et al. (1977) Golden, B. L., T. L. Magnanti, H. Q. Nguyen. 1977. Implementing vehicle routing algorithms. Networks 7 (2) 113–148.
  • He et al. (2016) He, K., X. Zhang, S. Ren, J. Sun. 2016. Deep residual learning for image recognition. Proceedings of the IEEE Conference on Computer Vision and Pattern Recognition . 770–778.
  • Kim et al. (2024) Kim, H., J. Park, C. Kwon. 2024. A neural separation algorithm for the rounded capacity inequalities. INFORMS Journal on Computing .
  • Kim et al. (2023) Kim, M., J. Park, J. Park. 2023. Learning to CROSS exchange to solve min-max vehicle routing problems. The Eleventh International Conference on Learning Representations . URL https://openreview.net/forum?id=ZcnzsHC10Y .
  • Kool et al. (2019) Kool, W., H. van Hoof, M. Welling. 2019. Attention, learn to solve routing problems! International Conference on Learning Representations . URL https://openreview.net/forum?id=ByxBFsRqYm .
  • Kou et al. (2023) Kou, S., B. Golden, L. Bertazzi. 2023. An improved model for estimating optimal vrp solution values. Optimization Letters 1–7.
  • Kwon et al. (2020) Kwon, Y.-D., J. Choo, B. Kim, I. Yoon, Y. Gwon, S. Min. 2020. Pomo: Policy optimization with multiple optima for reinforcement learning. Advances in Neural Information Processing Systems 33 21188–21198.
  • Laporte et al. (1986) Laporte, G., Y. Nobert, D. Arpin. 1986. An exact algorithm for solving a capacitated location-routing problem. Annals of Operations Research 6 291–310.
  • Laporte et al. (1988) Laporte, G., Y. Nobert, S. Taillefer. 1988. Solving a family of multi-depot vehicle routing and location-routing problems. Transportation Science 22 (3) 161–172.
  • Li et al. (2021) Li, S., Z. Yan, C. Wu. 2021. Learning to delegate for large-scale vehicle routing. Advances in Neural Information Processing Systems 34 26198–26211.
  • Lu et al. (2020) Lu, H., X. Zhang, S. Yang. 2020. A learning-based iterative method for solving vehicle routing problems. International Conference on Learning Representations . URL https://openreview.net/forum?id=BJe1334YDH .
  • Morabit et al. (2021) Morabit, M., G. Desaulniers, A. Lodi. 2021. Machine-learning–based column selection for column generation. Transportation Science 55 (4) 815–831.
  • Nazari et al. (2018) Nazari, M., A. Oroojlooy, L. Snyder, M. Takác. 2018. Reinforcement learning for solving the vehicle routing problem. Advances in Neural Information Processing Systems 31 .
  • Nicola et al. (2019) Nicola, D., R. Vetschera, A. Dragomir. 2019. Total distance approximations for routing solutions. Computers & Operations Research 102 67–74.
  • Park et al. (2023) Park, J., C. Kwon, J. Park. 2023. Learn to solve the min-max multiple traveling salesmen problem with reinforcement learning. Proceedings of the 2023 International Conference on Autonomous Agents and Multiagent Systems (AAMAS ‘23) . 878–886.
  • Qi et al. (2012) Qi, M., W.-H. Lin, N. Li, L. Miao. 2012. A spatiotemporal partitioning approach for large-scale vehicle routing problems with time windows. Transportation Research Part E: Logistics and Transportation Review 48 (1) 248–257.
  • Ramos et al. (2020) Ramos, T. R. P., M. I. Gomes, A. P. B. Póvoa. 2020. Multi-depot vehicle routing problem: a comparative study of alternative formulations. International Journal of Logistics Research and Applications 23 (2) 103–120.
  • Robust et al. (1990) Robust, F., C. F. Daganzo, R. R. Souleyrette II. 1990. Implementing vehicle routing models. Transportation Research Part B: Methodological 24 (4) 263–286.
  • Saberi and Verbas (2012) Saberi, M., İ. Ö. Verbas. 2012. Continuous approximation model for the vehicle routing problem for emissions minimization at the strategic level. Journal of Transportation Engineering 138 (11) 1368–1376.
  • Sadati et al. (2021) Sadati, M. E. H., B. Çatay, D. Aksen. 2021. An efficient variable neighborhood search with tabu shaking for a class of multi-depot vehicle routing problems. Computers & Operations Research 133 105269.
  • Schneider and Löffler (2019) Schneider, M., M. Löffler. 2019. Large composite neighborhoods for the capacitated location-routing problem. Transportation Science 53 (1) 301–318.
  • Solomon (1987) Solomon, M. M. 1987. Algorithms for the vehicle routing and scheduling problems with time window constraints. Operations Research 35 (2) 254–265.
  • Stroh et al. (2022) Stroh, A. M., A. L. Erera, A. Toriello. 2022. Tactical design of same-day delivery systems. Management Science 68 (5) 3444–3463.
  • Tuzun and Burke (1999) Tuzun, D., L. I. Burke. 1999. A two-phase tabu search approach to the location routing problem. European Journal of Operational Research 116 (1) 87–99.
  • Uchoa et al. (2017) Uchoa, E., D. Pecin, A. Pessoa, M. Poggi, T. Vidal, A. Subramanian. 2017. New benchmark instances for the capacitated vehicle routing problem. European Journal of Operational Research 257 (3) 845–858.
  • Varol et al. (2024) Varol, T., O. Ö. Özener, E. Albey. 2024. Neural network estimators for optimal tour lengths of traveling salesperson problem instances with arbitrary node distributions. Transportation Science 58 (1) 45–66.
  • Vaswani et al. (2017) Vaswani, A., N. Shazeer, N. Parmar, J. Uszkoreit, L. Jones, A. N. Gomez, Ł. Kaiser, I. Polosukhin. 2017. Attention is all you need. Advances in Neural Information Processing Systems 30 .
  • Vidal (2022) Vidal, T. 2022. Hybrid genetic search for the CVRP: Open-source implementation and SWAP* neighborhood. Computers & Operations Research 140 105643.
  • Vidal et al. (2012) Vidal, T., T. G. Crainic, M. Gendreau, N. Lahrichi, W. Rei. 2012. A hybrid genetic algorithm for multidepot and periodic vehicle routing problems. Operations Research 60 (3) 611–624.
  • Vidal et al. (2014) Vidal, T., T. G. Crainic, M. Gendreau, C. Prins. 2014. Implicit depot assignments and rotations in vehicle routing heuristics. European Journal of Operational Research 237 (1) 15–28.
  • Vinyals et al. (2015) Vinyals, O., M. Fortunato, N. Jaitly. 2015. Pointer networks. Proceedings of the 28th International Conference on Neural Information Processing Systems-Volume 2 . 2692–2700.
  • Wu et al. (2021) Wu, Y., W. Song, Z. Cao, J. Zhang, A. Lim. 2021. Learning improvement heuristics for solving routing problems. IEEE Transactions on Neural Networks and Learning Systems 33 (9) 5057–5069.
  • Xu et al. (2020) Xu, K., M. Zhang, J. Li, S. S. Du, K.-i. Kawarabayashi, S. Jegelka. 2020. How neural networks extrapolate: From feedforward to graph neural networks. arXiv preprint arXiv:2009.11848 .
  • Zhang et al. (2023) Zhang, K., X. Lin, M. Li. 2023. Graph attention reinforcement learning with flexible matching policies for multi-depot vehicle routing problems. Physica A: Statistical Mechanics and its Applications 128451.
  • Zong et al. (2022) Zong, Z., H. Wang, J. Wang, M. Zheng, Y. Li. 2022. Rbg: Hierarchically solving large-scale routing problems in logistic systems via reinforcement learning. Proceedings of the 28th ACM SIGKDD Conference on Knowledge Discovery and Data Mining . 4648–4658.
  • Zou et al. (2022) Zou, Y., H. Wu, Y. Yin, L. Dhamotharan, D. Chen, A. K. Tiwari. 2022. An improved transformer model with multi-head attention and attention to attention for low-carbon multi-depot vehicle routing problem. Annals of Operations Research 1–20.

Appendix A Capacitated Location Routing Problem

We choose the Capacitated Location Routing Problem (CLRP) as an example to demonstrate the applicability of our decomposition heuristic with subproblem cost predictions to other HVRPs. CLRP is a three-level hierarchical optimization problem. It involves opening depots, assigning customers to the opened depots, and making vehicle routing decisions to minimize overall operational costs while fulfilling all customer demands. One can refer to Laporte et al. ( 1986 ) for a mathematical formulation of the CLRP. We consider CLRP instances where the depots and vehicles are capacitated, and opening a depot or a route (vehicle) results in depot/vehicle opening costs. After selecting the depots to open from a set of candidate locations, the CLRP reduces into a variant of the MDVRP. Here, the main difference in the MDVRP variant from our T instances lies in the capacitated depots and the ability to add multiple routes (i.e., utilize more vehicles), each incurring an opening cost.

A.1 Solution Methodology

Since CLRP is a generalization of the MDVRP with additional upper level decisions, the solution methodology for MDVRP can be easily extended to this problem. Our GA for CLRP is extended by modifying the MDVRP chromosome representation to include both depot opening and customer assignment decisions, as shown in Figure 6 . A depot-opening chromosome is a binary vector that indicates whether a depot is open or closed, while the customer allocation chromosome represents the active depot assigned to a customer. For an allocation to be valid, the corresponding depot must be open. Following the GA methodology for MDVRP, each parent is selected using binary tournament selection, and crossovers are applied to the customer allocation chromosome. After determining all customer assignments for a child, only the depots corresponding to the allocated customers are opened on the depot chromosome. A similar approach can be utilized to determine the solution representation after mutations on the customer allocation chromosome. In each generation of the GA, a mutation of targeted customer assignments is performed to the nearest depot or the nearest neighbor’s nearest depot. This applies to approximately 10 % percent 10 10\% 10 % of the candidates determined by tournament selection from the population. Note that since this experiment is a proof of concept for our hierarchical problem-solving approach, we do not explore advanced location searches or mutations.

While the vehicles are capacitated in a CLRP, there is no restriction on the number of vehicles that can be utilized from a depot. Consequently, infeasibility in a CLRP solution may stem not from the vehicle capacity constraints but rather from depot capacity constraints. Similar to the MDVRP methodology, infeasible candidates are repaired with a probability by random assignment to available and open depots. During the initial population generation, we ensure that the open depots can satisfy the total demand for all customers. Each CLRP chromosome is identical to an MDVRP solution once the locations to be opened are determined. CVRP subproblems arising from the solution decompositions can be passed to the NN model in batches to obtain cost predictions. To assess the quality of a CLRP chromosome, subproblem routing costs and diversity scores alone are insufficient. Depot opening costs, an approximate estimate of vehicle opening costs (based on a lower bound on the number of vehicles), total routing cost, and solution diversity are all taken into consideration.

Our computational study focuses on the transferability of the GANCP framework and NN model from MDVRP to the CLRP. Therefore, we use the NN parameter weights from Phase III experiments in determining the solutions to CLRP benchmark datasets. This ensures that our approach is applicable to CLRP without requiring computational effort in training a problem-specific NN model.

A.2 Computational Study for CLRP

Our modified genetic algorithm for CLRP is carried out by using the same set of trained parameters θ 𝜃 \theta italic_θ as in the MDVRP experiments. Note that some local searches, such as random mutations, are not considered in this evaluation of CLRP benchmark datasets. We focus on our main goal of verifying the applicability of decomposition evaluation through cost prediction and therefore do not investigate additional solution improvement strategies for the CLRP. We analyze the performance of our modified heuristic in two popular CLRP benchmark sets, Tuzun and Burke ( 1999 ) and Barreto ( 2004 ) . Tuzun and Burke ( 1999 ) contains 36 36 36 36 instances with N ∈ { 100 , 150 , 200 } 𝑁 100 150 200 N\in\{100,150,200\} italic_N ∈ { 100 , 150 , 200 } customers, D ∈ { 10 , 20 } 𝐷 10 20 D\in\{10,20\} italic_D ∈ { 10 , 20 } uncapacitated depots and capacitated vehicles. Customers are distributed in varying densities, and the clusters are sized differently in these instances. Barreto ( 2004 ) instances are derived from the CVRP instances with up to 150 150 150 150 customers and 10 10 10 10 depots. The route (vehicle) opening costs are ignored in these instances.

\text{GANCP}^{+} GANCP start_POSTSUPERSCRIPT + end_POSTSUPERSCRIPT achieves proven optimal solutions in a few instances.

Appendix B Detailed Results of Computational Experiments

Instance NDA K-Means-10 VROOM VROOM- Average of 10 runs Best of 10 runs
Cost Cost Time Cost Time Cost Time NN Cost Time

Time

(sec) (sec) (sec) (sec) (sec) (%) (%) (%) (%) (%) (%)
T001-n100-d2 17390.51 17875.93 9.14 17166.17 0.50 17166.17 0.49 16553.16 1.01 17315.14 2.01 -0.43 -3.14 0.87 17183.24 -1.19 -3.87 0.10
T002-n113-d2 22632.60 22627.69 2.15 22489.61 0.78 22489.61 0.78 22450.15 1.04 22554.59 2.02 -0.34 -0.32 0.29 22500.46 -0.58 -0.56 0.05
T003-n127-d2 25580.35 25760.03 2.19 25330.31 0.86 25330.31 0.83 24704.28 1.18 25462.76 2.18 -0.46 -1.15 0.52 25138.36 -1.73 -2.41 -0.76
T004-n143-d2 35378.16 35552.89 2.19 35259.19 1.06 35259.19 1.13 34057.09 1.32 35250.60 2.32 -0.36 -0.85 -0.02 35023.34 -1.00 -1.49 -0.67
T005-n151-d3 24276.17 25466.05 3.19 24081.61 1.18 24081.61 1.18 23013.48 1.68 24175.28 3.18 -0.42 -5.07 0.39 24175.28 -0.42 -5.07 0.39
T006-n167-d2 33124.32 37978.55 2.28 32671.05 1.88 32671.05 1.84 32767.70 1.49 33020.89 2.49 -0.31 -13.05 1.07 32764.11 -1.09 -13.73 0.28
T007-n196-d3 28030.22 28734.19 3.24 27893.84 2.18 27893.84 2.17 28533.28 2.13 27711.31 3.63 -1.14 -3.56 -0.65 27557.16 -1.69 -4.10 -1.21
T008-n198-d2 41688.22 43524.77 2.25 41398.74 3.05 41398.74 2.78 41817.22 1.70 41229.69 2.70 -1.10 -5.27 -0.41 40981.93 -1.69 -5.84 -1.01
T009-n199-d3 37092.34 39305.30 3.25 36923.56 2.39 36923.56 2.42 35547.01 2.14 36965.70 3.64 -0.34 -5.95 0.11 36783.30 -0.83 -6.42 -0.38
T010-n200-d4 31564.57 39606.67 4.27 31775.31 2.74 31775.31 2.71 31727.54 2.80 31379.72 4.80 -0.59 -20.77 -1.24 31103.13 -1.46 -21.47 -2.12
T011-n203-d3 34416.78 39789.59 3.27 34305.96 2.72 34305.96 2.67 34627.33 2.31 34382.04 3.81 -0.10 -13.59 0.22 34278.09 -0.40 -13.85 -0.08
T012-n207-d4 31756.89 40040.09 4.27 32042.14 3.37 32042.14 3.34 30453.94 2.80 31588.49 4.80 -0.53 -21.11 -1.42 31369.87 -1.22 -21.65 -2.10
T013-n222-d3 43380.44 45195.20 3.29 43287.22 3.06 43287.22 3.02 42589.75 2.56 43565.26 4.06 0.43 -3.61 0.64 43125.71 -0.59 -4.58 -0.37
T014-n229-d2 38773.51 42790.45 20.35 38721.88 3.61 38721.88 3.57 38898.84 2.20 38405.54 12.20 -0.95 -10.25 -0.82 38320.44 -1.17 -10.45 -1.04
T015-n232-d4 40691.95 51674.20 4.31 40492.83 3.52 40492.83 3.50 40301.62 3.39 40525.98 5.39 -0.41 -21.57 0.08 40361.03 -0.81 -21.89 -0.33
T016-n235-d3 41946.43 42722.86 3.31 42086.26 3.60 42086.26 3.67 39610.28 2.69 41510.99 4.19 -1.04 -2.84 -1.37 41470.60 -1.13 -2.93 -1.46
T017-n269-d5 34511.06 55384.06 5.38 34389.10 3.85 34389.10 3.88 33460.22 4.45 34119.98 6.95 -1.13 -38.39 -0.78 33904.89 -1.76 -38.78 -1.41
T018-n276-d2 49689.65 49639.51 20.44 50123.49 5.27 50123.49 5.44 49908.66 2.78 49289.98 12.78 -0.80 -0.70 -1.66 49264.31 -0.86 -0.76 -1.71
T019-n288-d4 36014.90 52575.74 4.42 35927.17 5.68 35927.17 5.80 35480.95 4.34 35718.69 6.34 -0.82 -32.06 -0.58 35594.19 -1.17 -32.30 -0.93
T020-n297-d2 51182.15 51332.99 20.47 51370.55 6.21 51370.55 6.39 50630.73 3.06 50661.79 13.06 -1.02 -1.31 -1.38 50606.84 -1.12 -1.41 -1.49
T021-n298-d3 49392.30 60574.73 3.45 49143.33 9.51 49143.33 5.36 47792.38 3.55 49097.61 5.05 -0.60 -18.95 -0.09 48867.59 -1.06 -19.33 -0.56
T022-n306-d3 48467.74 51385.97 30.63 49115.12 6.03 49115.12 6.20 47299.71 3.83 48104.63 18.83 -0.75 -6.39 -2.06 48050.86 -0.86 -6.49 -2.17
T023-n308-d5 39204.51 48113.67 5.48 38860.82 7.80 38860.82 7.74 37624.79 5.20 38724.29 7.70 -1.22 -19.51 -0.35 38610.51 -1.52 -19.75 -0.64
T024-n314-d5 40656.96 55320.78 5.49 41139.55 10.47 41139.55 8.14 40865.76 5.25 40846.95 7.75 0.47 -26.16 -0.71 40488.90 -0.41 -26.81 -1.58
T025-n314-d6 36762.91 41728.19 6.08 36660.97 7.98 36660.97 8.23 36171.00 5.87 36555.12 8.87 -0.57 -12.40 -0.29 36399.74 -0.99 -12.77 -0.71
T026-n324-d5 43807.57 48936.01 5.51 43423.42 6.67 43423.42 6.29 43090.65 5.54 43958.06 8.05 0.34 -10.17 1.23 43407.38 -0.91 -11.30 -0.04
T027-n326-d2 55060.27 58568.28 20.59 54856.20 7.83 54856.20 8.03 55351.76 3.41 54587.47 13.42 -0.86 -6.80 -0.49 54387.16 -1.22 -7.14 -0.86
T028-n339-d4 47691.77 60661.02 4.51 47938.52 8.84 47938.52 7.43 47322.44 4.95 47783.97 6.96 0.19 -21.23 -0.32 47691.77 0.00 -21.38 -0.51
T029-n348-d3 65795.40 69809.67 30.67 66172.13 10.76 66172.13 11.14 65707.48 4.59 65270.16 19.60 -0.80 -6.50 -1.36 65202.67 -0.90 -6.60 -1.47
T030-n355-d6 51949.17 66414.83 6.55 51907.11 10.51 51907.11 9.89 50930.74 6.86 51699.52 9.87 -0.48 -22.16 -0.40 51414.50 -1.03 -22.59 -0.95
T031-n359-d7 42664.60 61396.23 7.55 42970.07 11.56 42970.07 11.06 43514.97 7.52 42584.23 11.02 -0.19 -30.64 -0.90 42536.33 -0.30 -30.72 -1.01
T032-n367-d5 42222.41 45707.94 5.58 42287.09 10.99 42287.09 9.30 41662.81 6.44 42003.96 8.94 -0.52 -8.10 -0.67 41787.08 -1.03 -8.58 -1.18
T033-n380-d2 69681.70 70630.94 20.62 70364.10 12.69 70364.10 13.06 69376.44 4.20 69304.32 14.21 -0.54 -1.88 -1.51 69300.57 -0.55 -1.88 -1.51
T034-n393-d6 50819.27 72061.98 6.61 51146.93 15.88 51146.93 12.33 49973.83 7.96 50433.09 10.96 -0.76 -30.01 -1.40 50370.40 -0.88 -30.10 -1.52
T035-n402-d6 49317.09 58030.77 6.62 49288.75 17.79 49288.75 12.11 47911.93 8.04 49138.92 11.04 -0.36 -15.32 -0.30 48942.76 -0.76 -15.66 -0.70
T036-n405-d3 74684.27 77122.76 30.80 74816.30 15.67 74816.30 16.79 73434.15 5.54 74281.41 20.55 -0.54 -3.68 -0.71 74023.18 -0.89 -4.02 -1.06
T037-n409-d5 67865.21 87791.54 5.62 67186.06 17.31 67186.06 10.59 65174.49 7.37 67507.97 9.87 -0.53 -23.10 0.48 67458.47 -0.60 -23.16 0.41
T038-n411-d8 40113.82 57118.94 8.64 40227.96 12.75 40227.96 12.81 38738.91 10.03 39926.70 14.03 -0.47 -30.10 -0.75 39775.92 -0.84 -30.36 -1.12
T039-n426-d5 54425.43 91576.55 5.65 54699.31 28.62 54949.84 11.25 52346.05 7.89 54223.79 10.39 -0.37 -40.79 -0.87 54014.84 -0.75 -41.02 -1.25
T040-n426-d3 58163.53 75335.11 30.17 59195.59 23.61 59195.59 21.96 58263.56 5.88 57771.13 20.88 -0.67 -23.31 -2.41 57732.58 -0.74 -23.37 -2.47
T041-n429-d7 43871.97 87083.69 7.68 44248.05 15.94 44248.05 13.48 43703.81 9.67 43703.07 13.17 -0.38 -49.81 -1.23 43638.52 -0.53 -49.89 -1.38
T042-n442-d5 58803.19 75029.90 5.67 58666.22 19.78 58666.22 11.31 58721.94 8.22 58947.15 10.73 0.24 -21.44 0.48 58374.75 -0.73 -22.20 -0.50
T043-n465-d2 86257.98 86916.76 40.86 86902.96 29.52 86902.96 26.90 84669.22 5.56 85760.01 25.57 -0.58 -1.33 -1.32 85587.25 -0.78 -1.53 -1.51
T044-n482-d8 49672.97 88810.84 8.78 49903.86 31.06 49903.86 18.51 49984.72 12.53 49725.66 16.53 0.11 -44.01 -0.36 49338.53 -0.67 -44.45 -1.13
T045-n489-d7 62067.18 81027.35 7.78 62185.92 29.83 62185.92 15.56 61848.66 11.18 61895.05 14.68 -0.28 -23.61 -0.47 61649.19 -0.67 -23.92 -0.86
T046-n497-d9 52491.02 79311.14 9.80 52600.17 25.49 52600.17 19.27 52002.07 13.06 52268.90 17.56 -0.42 -34.10 -0.63 52095.42 -0.75 -34.32 -0.96
T047-n503-d3 79409.73 94564.28 30.88 79547.74 56.52 79547.74 24.38 78054.59 7.19 79218.41 22.20 -0.24 -16.23 -0.41 79084.49 -0.41 -16.37 -0.58
T048-n505-d6 62084.64 97288.37 6.80 61555.26 36.23 61599.50 14.67 61682.04 10.54 61846.28 13.55 -0.38 -36.43 0.47 61539.96 -0.88 -36.74 -0.02
T049-n509-d8 50412.17 87618.22 8.88 50857.82 39.48 50857.82 19.36 51403.11 12.65 50581.33 16.65 0.34 -42.27 -0.54 50278.10 -0.27 -42.62 -1.14
T050-n510-d2 94647.21 111261.22 40.95 94044.04 35.12 94044.04 27.68 91931.96 6.21 93549.19 26.22 -1.16 -15.92 -0.53 93415.66 -1.30 -16.04 -0.67
T051-n524-d9 56200.20 93618.30 9.87 56508.63 32.84 56508.63 21.47 56690.04 14.22 56451.31 18.73 0.45 -39.70 -0.10 56169.36 -0.05 -40.00 -0.60
T052-n527-d2 102869.23 106426.77 40.99 103008.16 36.05 103008.16 28.16 100910.17 6.54 102462.67 26.55 -0.40 -3.72 -0.53 102302.44 -0.55 -3.88 -0.69
T053-n534-d10 56552.64 101266.82 10.89 56749.52 30.71 56749.52 22.91 55644.95 15.56 56424.74 20.56 -0.23 -44.28 -0.57 56179.64 -0.66 -44.52 -1.00
T054-n540-d7 81027.33 126267.45 7.87 80059.69 58.48 80059.69 17.80 79479.85 12.66 80678.59 16.17 -0.43 -36.10 0.77 80241.18 -0.97 -36.45 0.23
T055-n566-d5 80923.40 110025.74 51.10 80780.27 49.44 80780.27 38.63 80285.61 10.89 80317.27 35.89 -0.75 -27.00 -0.57 80053.77 -1.07 -27.24 -0.90
T056-n568-d4 82684.83 88524.93 41.05 83556.16 55.14 83556.16 33.19 84066.56 9.76 82199.30 29.77 -0.59 -7.15 -1.62 82164.22 -0.63 -7.19 -1.67
T057-n575-d8 - 105319.55 8.96 58955.89 42.22 58955.89 20.66 58550.50 14.56 58945.34 18.57 - -44.03 -0.02 58411.12 - -44.54 -0.92
T058-n579-d2 101252.23 114775.53 41.09 100663.45 95.22 100663.45 31.91 98187.65 7.63 100464.83 27.65 -0.78 -12.47 -0.20 100341.27 -0.90 -12.58 -0.32
T059-n581-d7 59934.75 112995.71 7.95 59614.05 60.39 59878.72 18.89 58646.00 13.69 59597.09 17.19 -0.56 -47.26 -0.03 59343.40 -0.99 -47.48 -0.45
T060-n593-d4 76299.85 83224.72 41.20 77060.51 51.47 77060.51 34.24 74849.83 10.39 75706.15 30.40 -0.78 -9.03 -1.76 75564.90 -0.96 -9.20 -1.94
T061-n603-d6 66488.01 97125.47 61.25 67270.27 76.83 67270.27 48.01 67309.91 13.34 66127.94 43.34 -0.54 -31.91 -1.70 66127.94 -0.54 -31.91 -1.70
T062-n618-d9 64066.29 109633.55 10.05 64736.50 98.16 64769.85 24.81 64666.71 17.15 63988.05 21.66 -0.12 -41.63 -1.16 63879.86 -0.29 -41.73 -1.32
T063-n623-d2 107002.90 124462.01 41.20 106461.77 81.29 106461.77 31.60 105633.81 9.04 106273.68 29.07 -0.68 -14.61 -0.18 106161.43 -0.79 -14.70 -0.28
T064-n634-d4 86214.49 103405.84 41.25 86214.91 92.96 86214.91 35.54 84698.75 11.15 85389.24 31.16 -0.96 -17.42 -0.96 85330.18 -1.03 -17.48 -1.03
T065-n647-d9 60550.63 124275.82 10.12 60554.03 99.27 60643.85 27.64 59704.36 18.64 60284.43 23.14 -0.44 -51.49 -0.45 60113.71 -0.72 -51.63 -0.73
T066-n680-d5 97843.23 142070.06 51.42 98220.53 95.60 98220.53 43.40 97205.11 13.94 97929.07 38.95 0.09 -31.07 -0.30 97429.40 -0.42 -31.42 -0.81
T067-n682-d8 66238.44 127140.62 9.17 66696.34 148.73 67153.71 25.44 64433.33 18.62 66402.29 22.62 0.25 -47.77 -0.44 66108.98 -0.20 -48.00 -0.88
T068-n684-d7 65138.19 140701.39 8.17 65503.01 100.25 65503.01 24.31 65112.27 17.30 65034.52 20.81 -0.16 -53.78 -0.72 64657.34 -0.74 -54.05 -1.29
T069-n686-d4 95913.03 117197.04 41.35 95966.77 106.22 95966.77 37.81 96952.69 12.91 96006.46 32.92 0.10 -18.08 0.04 95345.17 -0.59 -18.65 -0.65
T070-n692-d5 74831.66 108166.23 51.44 75246.33 127.60 75246.33 45.19 74350.61 14.27 74266.39 39.27 -0.76 -31.34 -1.30 74153.97 -0.91 -31.44 -1.45
T071-n723-d6 84499.83 108546.72 61.42 84919.30 158.59 84919.30 53.24 83098.01 17.29 83867.82 47.30 -0.75 -22.74 -1.24 83666.89 -0.99 -22.92 -1.47
T072-n734-d7 70010.00 124894.30 71.47 71147.92 122.61 71149.48 60.08 70067.65 18.67 70008.68 53.68 0.00 -43.95 -1.60 69739.71 -0.39 -44.16 -1.98
T073-n745-d3 108548.98 146425.96 61.50 108173.93 209.11 108173.93 50.41 106860.85 13.63 107700.60 43.66 -0.78 -26.45 -0.44 107371.74 -1.08 -26.67 -0.74
T074-n756-d5 90113.70 138413.00 51.44 90896.57 132.71 90998.02 47.77 89410.22 16.54 89528.35 41.55 -0.65 -35.32 -1.51 89235.37 -0.97 -35.53 -1.83
T075-n777-d10 72867.20 146345.77 11.35 72726.66 155.51 72831.48 35.81 72722.36 25.12 72682.64 30.12 -0.25 -50.33 -0.06 72368.54 -0.68 -50.55 -0.49
T076-n789-d2 156904.60 158265.26 41.48 155606.23 175.99 155606.23 38.07 149084.74 13.12 156080.91 33.15 -0.52 -1.38 0.31 155770.73 -0.72 -1.58 0.11
T077-n794-d3 126663.35 147492.92 61.67 125871.32 132.21 125871.32 49.37 126225.92 14.34 125702.85 44.36 -0.76 -14.77 -0.13 125465.72 -0.95 -14.93 -0.32
T078-n803-d4 100328.01 118669.56 81.80 101208.62 144.27 101208.62 64.10 103011.71 16.33 100427.03 56.35 0.10 -15.37 -0.77 100085.25 -0.24 -15.66 -1.11
T079-n810-d7 80685.95 117097.80 71.72 81726.09 204.31 81726.09 64.18 81296.17 21.82 80410.27 56.83 -0.34 -31.33 -1.61 80203.44 -0.60 -31.51 -1.86
T080-n816-d3 155441.13 164991.91 61.72 154897.13 179.42 154897.13 51.42 156839.64 15.36 154770.21 45.39 -0.43 -6.20 -0.08 154764.43 -0.44 -6.20 -0.09
T081-n817-d6 83826.65 133201.93 61.67 85041.40 196.15 85041.40 53.87 83751.60 20.21 83325.57 50.22 -0.60 -37.44 -2.02 83069.44 -0.90 -37.64 -2.32
T082-n820-d3 101738.18 106555.66 61.63 102080.29 284.99 102080.29 54.59 99951.16 15.55 101516.00 45.57 -0.22 -4.73 -0.55 101069.89 -0.66 -5.15 -0.99
T083-n831-d5 98898.56 124401.01 51.52 99617.52 182.24 99617.52 49.64 99536.21 19.01 98684.59 44.03 -0.22 -20.67 -0.94 98552.29 -0.35 -20.78 -1.07
T084-n858-d6 90386.05 141028.34 61.64 90993.54 206.43 90993.54 56.78 89026.06 21.33 89889.84 51.34 -0.55 -36.26 -1.21 89675.35 -0.79 -36.41 -1.45
T085-n859-d10 69950.42 161473.58 11.47 70188.38 209.29 70188.38 41.59 69405.42 28.70 69436.16 33.70 -0.74 -57.00 -1.07 69344.10 -0.87 -57.06 -1.20
T086-n864-d7 83041.32 153898.20 71.72 84005.31 264.38 84005.31 68.27 82035.41 24.06 82539.78 59.07 -0.60 -46.37 -1.74 82159.35 -1.06 -46.61 -2.20
T087-n867-d9 75836.82 160329.08 10.47 76007.36 241.15 76007.36 37.82 74842.08 27.90 75544.66 32.40 -0.39 -52.88 -0.61 75364.30 -0.62 -52.99 -0.85
T088-n871-d3 142289.22 154624.30 61.66 142381.34 334.79 142381.34 54.47 142145.70 17.35 142083.23 47.39 -0.14 -8.11 -0.21 141628.80 -0.46 -8.40 -0.53
T089-n871-d2 141044.53 143646.23 40.34 139854.93 242.94 139854.93 45.02 136886.35 16.62 140354.87 36.66 -0.49 -2.29 0.36 140028.42 -0.72 -2.52 0.12
T090-n899-d9 91503.11 145629.87 10.55 91636.03 276.72 91636.03 39.47 88188.19 28.85 91213.19 33.36 -0.32 -37.37 -0.46 91127.65 -0.41 -37.43 -0.55
T091-n903-d8 85114.43 167281.97 81.92 86309.03 320.92 86309.03 78.28 84326.78 27.62 84890.96 67.63 -0.26 -49.25 -1.64 84671.14 -0.52 -49.38 -1.90
T092-n905-d4 111976.88 117313.66 81.86 112733.10 204.08 112733.10 69.45 114491.36 20.06 111349.14 60.08 -0.56 -5.08 -1.23 111193.74 -0.70 -5.22 -1.37
T093-n923-d3 132592.11 145457.83 61.78 132114.76 307.15 132114.76 57.77 129804.53 18.68 131532.07 48.71 -0.80 -9.57 -0.44 131240.89 -1.02 -9.77 -0.66
T094-n924-d2 150269.43 166191.54 41.71 149215.21 423.03 149215.21 46.51 145856.23 18.75 149205.18 38.80 -0.71 -10.22 -0.01 149067.94 -0.80 -10.30 -0.10
T095-n935-d6 100078.50 165719.19 61.83 100293.22 275.39 100293.22 65.68 100996.00 24.73 99495.71 54.74 -0.58 -39.96 -0.80 99103.53 -0.97 -40.20 -1.19
T096-n943-d7 121106.11 151209.43 71.97 121321.95 290.12 121321.95 70.10 123099.97 27.29 120442.95 62.31 -0.55 -20.35 -0.72 120283.93 -0.68 -20.45 -0.86
T097-n955-d5 116720.71 175657.11 51.79 116720.57 459.82 116720.57 60.57 115041.59 23.22 116150.59 48.24 -0.49 -33.88 -0.49 115919.05 -0.69 -34.01 -0.69
T098-n972-d10 86690.58 202042.12 11.67 86493.27 395.53 86493.27 47.37 84943.55 34.17 86073.25 39.18 -0.71 -57.40 -0.49 85681.08 -1.16 -57.59 -0.94
T099-n985-d2 188266.44 196835.94 41.85 186937.02 301.76 186937.02 51.35 183225.95 21.31 187548.18 41.35 -0.38 -4.72 0.33 187523.11 -0.39 -4.73 0.31
T100-n1000-d9 87076.14 174917.30 92.06 88286.12 323.93 88313.91 91.69 87298.92 33.93 86746.11 78.94 -0.38 -50.41 -1.74 86496.79 -0.67 -50.55 -2.03
T101-n1012-d3 137802.77 149240.77 61.99 137128.19 503.12 137128.19 62.88 133386.18 22.05 136702.19 52.11 -0.80 -8.40 -0.31 136314.29 -1.08 -8.66 -0.59
T102-n1024-d6 112537.26 204901.42 61.99 113598.46 315.52 113598.46 66.28 113487.73 28.51 112393.72 58.52 -0.13 -45.15 -1.06 111780.84 -0.67 -45.45 -1.60
T103-n1036-d6 119310.64 182622.03 61.32 119230.85 485.94 119230.85 73.00 117209.51 28.91 118602.35 58.93 -0.59 -35.06 -0.53 118451.87 -0.72 -35.14 -0.65
T104-n1039-d10 80016.10 194124.97 101.62 81504.88 429.35 81504.88 103.47 80207.17 37.32 79751.46 87.33 -0.33 -58.92 -2.15 79493.12 -0.65 -59.05 -2.47
T105-n1047-d5 137436.44 178299.85 101.62 138035.45 433.75 138035.45 91.39 137159.93 27.44 137152.49 77.46 -0.21 -23.08 -0.64 136981.06 -0.33 -23.17 -0.76
T106-n1050-d4 136490.35 168681.82 81.46 136444.69 527.57 136444.69 78.68 135559.82 25.38 135818.12 65.42 -0.49 -19.48 -0.46 135707.54 -0.57 -19.55 -0.54
T107-n1069-d6 127168.23 195038.88 61.35 127508.99 388.72 127508.99 74.26 124901.44 30.40 126846.52 60.42 -0.25 -34.96 -0.52 126564.79 -0.47 -35.11 -0.74
T108-n1085-d10 99917.90 189179.61 101.67 101617.66 488.53 101617.66 107.28 101099.50 40.17 99856.78 90.18 -0.06 -47.22 -1.73 99734.51 -0.18 -47.28 -1.85
T109-n1086-d4 134097.17 170543.47 81.62 133504.62 487.10 133504.62 81.98 130250.51 27.36 133097.33 67.39 -0.75 -21.96 -0.31 132865.80 -0.92 -22.09 -0.48
T110-n1094-d3 171800.83 186818.96 61.53 169798.14 474.56 169798.14 65.97 170699.90 25.38 170519.61 55.42 -0.75 -8.72 0.42 170115.51 -0.98 -8.94 0.19
T111-n1109-d10 95850.21 167762.69 101.75 96864.21 609.60 96864.21 105.37 95929.01 41.56 95231.63 91.56 -0.65 -43.23 -1.69 95014.25 -0.87 -43.36 -1.91
T112-n1112-d7 117169.89 203788.88 71.49 118486.60 372.79 118486.60 84.96 115983.72 34.76 116944.58 69.77 -0.19 -42.61 -1.30 116613.98 -0.47 -42.78 -1.58
T113-n1160-d7 118252.48 188264.99 71.54 119276.20 577.77 119355.61 87.51 116637.40 37.24 117573.47 72.26 -0.57 -37.55 -1.43 117389.65 -0.73 -37.65 -1.58
T114-n1178-d3 158328.70 169541.46 61.55 157700.30 660.99 157700.30 71.18 156795.91 29.60 158060.74 59.64 -0.17 -6.77 0.23 157542.17 -0.50 -7.08 -0.10
T115-n1195-d7 128274.89 181008.25 71.61 128870.34 534.07 128870.34 86.26 127423.57 38.60 127799.41 73.62 -0.37 -29.40 -0.83 127699.43 -0.45 -29.45 -0.91
T116-n1209-d9 105482.14 227760.12 91.75 107213.41 627.89 107213.41 102.73 107538.47 45.51 105386.76 90.52 -0.09 -53.73 -1.70 105084.45 -0.38 -53.86 -1.99
T117-n1214-d8 114083.63 184995.27 81.76 114964.45 642.77 114964.45 100.03 117275.13 42.56 113478.04 82.57 -0.53 -38.66 -1.29 113112.51 -0.85 -38.86 -1.61
T118-n1229-d4 195134.15 217536.83 81.76 194196.06 720.72 194196.06 89.94 192079.47 35.13 194043.87 75.17 -0.56 -10.80 -0.08 193845.81 -0.66 -10.89 -0.18
T119-n1235-d9 100909.00 223928.07 91.76 102023.33 631.84 102041.54 116.60 98876.96 46.84 100185.33 91.85 -0.72 -55.26 -1.80 99717.09 -1.18 -55.47 -2.26
T120-n1248-d5 130580.81 188676.35 101.80 131277.99 551.14 131277.99 107.53 132321.77 37.07 130430.20 87.10 -0.12 -30.87 -0.65 129911.19 -0.51 -31.15 -1.04
T121-n1258-d3 178227.34 197941.30 61.72 177554.29 802.46 177554.29 83.57 175925.98 34.40 178135.55 64.45 -0.05 -10.01 0.33 177304.83 -0.52 -10.43 -0.14
T122-n1264-d8 114397.73 250532.07 81.87 115954.00 795.74 116461.00 101.22 118013.90 45.64 114004.01 85.66 -0.34 -54.50 -1.68 113728.75 -0.58 -54.61 -1.92
T123-n1266-d3 213691.26 220217.38 61.79 211823.19 773.96 211823.19 81.78 207573.56 35.34 213058.09 65.40 -0.30 -3.25 0.58 212950.25 -0.35 -3.30 0.53
T124-n1274-d8 121521.59 195432.21 81.92 121600.10 950.42 121600.10 110.27 119675.00 46.53 120763.31 86.55 -0.62 -38.21 -0.69 120452.11 -0.88 -38.37 -0.94
T125-n1282-d9 123235.57 238394.53 91.79 124118.94 971.50 124118.94 115.83 122120.43 49.45 122656.43 94.47 -0.47 -48.55 -1.18 122454.05 -0.63 -48.63 -1.34
T126-n1288-d7 155962.83 194746.34 71.71 155958.43 811.37 155958.43 103.53 156867.22 43.79 155381.08 78.82 -0.37 -20.21 -0.37 155178.70 -0.50 -20.32 -0.50
T127-n1307-d9 114473.83 224304.51 91.86 116019.74 850.76 116019.74 115.33 116588.78 50.13 114634.46 95.16 0.14 -48.89 -1.19 114010.67 -0.40 -49.17 -1.73
T128-n1313-d7 132399.37 220641.29 71.98 132031.36 1044.56 132031.36 103.08 132072.48 45.17 131515.32 80.19 -0.67 -40.39 -0.39 131308.79 -0.82 -40.49 -0.55
T129-n1316-d9 134839.86 231210.82 92.15 135909.98 990.16 135909.98 116.71 136132.01 50.90 134836.31 95.92 0.00 -41.68 -0.79 134379.45 -0.34 -41.88 -1.13
T130-n1330-d3 180450.65 191504.40 61.89 179318.58 975.40 179318.58 89.59 181088.57 38.59 180034.69 68.65 -0.23 -5.99 0.40 179657.59 -0.44 -6.19 0.19
T131-n1334-d8 151850.90 233685.11 81.92 151824.31 711.69 151824.31 105.73 153130.75 48.77 151301.40 88.80 -0.36 -35.25 -0.34 150873.28 -0.64 -35.44 -0.63
T132-n1357-d6 158737.60 262695.34 122.06 158945.23 881.19 158945.23 125.32 156737.67 45.29 158392.38 105.32 -0.22 -39.70 -0.35 157846.18 -0.56 -39.91 -0.69
T133-n1367-d3 195684.54 208106.77 62.00 193117.99 864.75 193117.99 86.08 194203.06 41.52 194883.83 71.57 -0.41 -6.35 0.91 194577.00 -0.57 -6.50 0.76
T134-n1369-d7 146806.17 270507.34 71.97 147257.34 1003.38 147257.34 102.89 147384.09 48.67 146367.35 83.70 -0.30 -45.89 -0.60 146037.32 -0.52 -46.01 -0.83
T135-n1377-d8 136392.39 296144.78 82.11 137546.06 943.86 137546.06 114.75 139862.53 52.21 135927.93 92.24 -0.34 -54.10 -1.18 135310.68 -0.79 -54.31 -1.63
T136-n1393-d9 114314.26 223730.98 92.27 115941.32 816.88 115941.32 120.41 116212.09 55.63 114523.54 100.65 0.18 -48.81 -1.22 113954.07 -0.32 -49.07 -1.71
T137-n1398-d4 249246.64 264527.20 82.20 247345.20 1079.25 247345.20 109.30 248386.57 43.74 248377.29 83.81 -0.35 -6.11 0.42 248156.34 -0.44 -6.19 0.33
T138-n1403-d3 199361.67 243927.55 62.02 197685.13 897.59 197685.13 89.96 197503.76 45.01 199168.47 75.11 -0.10 -18.35 0.75 198730.98 -0.32 -18.53 0.53
T139-n1412-d8 152566.48 291179.11 82.09 152670.38 824.18 152670.38 113.81 150265.32 54.36 152103.87 94.39 -0.30 -47.76 -0.37 151895.39 -0.44 -47.83 -0.51
T140-n1416-d6 143581.45 207459.48 122.34 143970.64 1091.44 143970.64 130.62 144065.03 48.45 143045.96 108.48 -0.37 -31.05 -0.64 142810.76 -0.54 -31.16 -0.81
T141-n1420-d4 220047.20 244400.44 81.99 218221.47 1026.84 218221.47 105.88 215670.00 44.22 218922.90 84.28 -0.51 -10.42 0.32 218922.74 -0.51 -10.42 0.32
T142-n1421-d5 154469.61 209928.60 102.32 154568.56 1047.29 154568.56 122.43 153487.60 47.75 153863.12 97.78 -0.39 -26.71 -0.46 153623.91 -0.55 -26.82 -0.61
T143-n1435-d7 161519.04 248277.39 142.60 161428.99 1121.70 161428.99 148.12 154796.83 52.90 160469.56 122.93 -0.65 -35.37 -0.59 160244.62 -0.79 -35.46 -0.73
T144-n1442-d4 174981.53 202146.35 82.12 174162.14 984.09 174162.14 101.50 172676.80 46.39 174375.78 86.43 -0.35 -13.74 0.12 173786.05 -0.68 -14.03 -0.22
T145-n1444-d6 145228.71 231758.44 122.33 145903.76 1065.12 145903.76 128.88 149798.24 50.71 144344.69 110.74 -0.61 -37.72 -1.07 144083.52 -0.79 -37.83 -1.25
T146-n1455-d5 164311.02 170957.27 102.37 164500.99 1061.19 164500.99 122.63 164432.19 47.88 163710.61 97.91 -0.37 -4.24 -0.48 163484.26 -0.50 -4.37 -0.62
T147-n1480-d10 122253.35 258673.62 102.38 123704.54 1257.74 123704.54 146.43 123792.98 65.23 122356.36 115.25 0.08 -52.70 -1.09 122006.96 -0.20 -52.83 -1.37
T148-n1486-d4 195969.88 230073.61 82.32 194583.28 1570.57 194583.28 114.61 197865.09 48.17 194736.76 88.23 -0.63 -15.36 0.08 194516.38 -0.74 -15.45 -0.03
T149-n1494-d3 192249.19 209840.52 62.22 190969.10 1301.99 190973.31 102.45 191113.36 48.48 191983.30 78.55 -0.14 -8.51 0.53 191283.05 -0.50 -8.84 0.16
T150-n1500-d5 152131.70 216392.03 102.43 152352.28 1191.00 152352.28 130.73 147890.13 51.51 151353.62 101.55 -0.51 -30.06 -0.66 150955.98 -0.77 -30.24 -0.92
Average Gap -0.42 -25.47 -0.59 -0.73 -25.70 -0.90
Instance VROOM Average of 10 runs Best of 10 runs
Cost Time GANCP Time

Time GANCP Time

Time
(sec) (sec) (sec) (%) (sec) (sec) (%)
O01-n55-d2 13046.00 0.11 13120.58 0.86 13119.74 1.87 0.57 13120.58 2.00 13119.74 3.02 0.57
O02-n58-d2 13396.68 0.11 13795.52 0.71 13484.07 1.70 0.65 13795.52 0.70 13484.07 1.70 0.65
O03-n64-d2 17634.26 0.14 18300.79 0.81 17721.86 1.82 0.50 18147.48 0.82 17719.46 1.82 0.48
O04-n70-d2 16284.54 0.14 15780.17 0.78 16344.11 1.78 0.37 15727.08 0.78 16287.42 1.78 0.02
O05-n72-d2 17668.66 0.18 17405.01 0.79 17764.05 1.79 0.54 17405.01 0.77 17764.05 1.77 0.54
O06-n74-d2 16798.60 0.20 16809.40 0.80 16888.19 1.81 0.53 17042.84 0.84 16835.85 1.84 0.22
O07-n75-d3 15020.76 0.27 15702.28 1.04 15327.23 2.55 2.04 15745.88 1.08 15323.38 2.58 2.01
O08-n78-d2 15062.94 0.18 15138.23 0.79 15306.72 1.79 1.62 15212.67 0.77 15108.62 1.77 0.30
O09-n84-d3 15709.18 0.30 15286.93 1.19 16523.24 2.69 5.18 14952.71 1.20 16429.19 2.70 4.58
O10-n86-d2 20149.83 0.25 19431.91 0.92 19869.00 1.94 -1.39 19431.91 0.96 19869.00 1.98 -1.39
O11-n91-d3 16141.22 0.36 15716.74 1.19 16072.47 2.66 -0.43 15630.49 1.15 15967.37 2.65 -1.08
O12-n96-d2 18838.04 0.33 18101.00 0.92 18632.94 1.92 -1.09 18209.62 0.91 18587.77 1.91 -1.33
O13-n100-d4 17075.42 0.44 17676.24 1.63 17615.75 3.64 3.16 18072.63 1.69 17587.82 3.69 3.00
O14-n104-d4 16872.50 0.45 17913.12 1.66 17019.30 3.66 0.87 17969.85 1.68 16971.73 3.68 0.59
O15-n108-d3 19933.67 0.47 20167.59 1.38 19924.81 2.88 -0.04 20192.75 1.32 19804.07 2.82 -0.65
O16-n116-d4 19602.32 0.62 20765.39 1.67 19583.03 3.67 -0.10 20742.90 1.65 19501.74 3.65 -0.51
O17-n120-d3 18541.54 0.57 18497.58 1.52 18603.24 3.02 0.33 18519.01 1.57 18590.85 3.07 0.27
O18-n124-d4 20533.69 0.80 19908.36 1.87 20424.76 3.87 -0.53 20094.64 1.85 20342.63 3.85 -0.93
O19-n126-d3 21866.33 0.68 21504.54 1.54 21953.41 3.04 0.40 21490.86 1.51 21798.10 3.01 -0.31
O20-n130-d5 20626.56 0.94 22075.37 2.25 20699.88 4.75 0.36 22029.50 2.25 20562.09 4.75 -0.31
O21-n135-d3 22076.84 0.72 21711.80 1.56 22000.31 3.06 -0.35 21472.18 1.54 21766.66 3.04 -1.41
O22-n136-d4 19619.87 0.86 18775.31 1.91 19441.89 3.91 -0.91 18777.62 1.83 19376.81 3.83 -1.24
O23-n141-d3 26515.67 1.01 26770.58 1.72 26739.24 3.22 0.84 26578.64 1.69 26621.22 3.19 0.40
O24-n144-d3 25254.53 0.96 24865.40 1.75 25117.44 3.25 -0.54 24781.35 1.73 25033.45 3.23 -0.88
O25-n145-d5 24631.29 1.14 24456.78 2.48 24405.61 4.99 -0.92 24643.78 2.46 24361.35 4.98 -1.10
O26-n148-d4 30452.34 1.19 28866.93 2.16 30446.77 4.16 -0.02 29058.94 2.08 30363.39 4.08 -0.29
O27-n152-d4 23002.27 1.26 23909.02 2.12 23190.83 4.12 0.82 24056.06 2.15 23012.42 4.15 0.04
O28-n155-d5 22448.21 1.32 22400.90 2.54 22763.71 5.04 1.41 22196.92 2.47 22531.04 4.97 0.37
O29-n156-d4 22491.65 1.21 22736.68 2.15 22881.73 4.15 1.73 22755.60 2.10 22699.61 4.10 0.92
O30-n160-d5 21172.66 1.48 20805.68 2.76 21369.47 5.26 0.93 20979.59 2.80 21146.07 5.30 -0.13
O31-n168-d4 24661.25 1.34 24409.43 2.36 24474.04 4.36 -0.76 24418.79 2.44 24470.40 4.44 -0.77
O32-n170-d5 22214.60 1.55 22899.82 2.76 22556.04 5.27 1.54 23034.71 2.73 22317.75 5.23 0.46
O33-n172-d4 24362.44 1.48 24233.68 2.40 24406.92 4.40 0.18 23942.73 2.39 24341.63 4.39 -0.09
O34-n180-d4 25596.32 1.58 25247.69 2.63 25349.07 4.63 -0.97 25160.86 2.70 25254.69 4.70 -1.33
O35-n185-d5 28925.29 2.37 29161.16 3.08 29247.41 5.58 1.11 29437.03 3.05 28917.75 5.55 -0.03
O36-n190-d5 27751.61 2.30 26621.62 3.10 27792.71 5.60 0.15 26639.15 3.05 27666.90 5.55 -0.31
O37-n192-d4 30592.23 2.17 29165.67 2.64 30495.66 4.64 -0.32 29165.67 2.62 30495.66 4.62 -0.32
O38-n200-d5 28719.89 2.76 28338.64 3.44 28696.39 5.95 -0.08 28110.76 3.41 28451.80 5.92 -0.93
O39-n235-d5 35792.40 3.98 35817.58 3.86 36125.30 6.37 0.93 35542.19 3.81 35877.75 6.32 0.24
O40-n245-d5 34921.58 4.44 34483.92 4.22 34367.88 6.72 -1.59 34358.67 4.18 34121.39 6.68 -2.29
O41-n1050-d2 218719.85 161.32 209828.27 24.20 219915.25 44.25 0.55 209003.43 24.13 219880.61 44.15 0.53
O42-n1159-d2 186810.50 225.55 185472.05 29.24 188159.35 49.28 0.72 185535.67 29.47 188034.38 49.50 0.66
O43-n1173-d2 199873.12 247.61 197200.95 30.84 200890.55 50.88 0.51 197214.94 30.42 200407.18 50.43 0.27
O44-n1225-d2 181660.32 266.65 182652.25 33.34 182235.49 53.38 0.32 183394.59 33.09 181895.12 53.14 0.13
O45-n1249-d2 242165.71 312.11 241640.56 35.01 245262.65 55.10 1.28 241108.19 35.42 245090.26 55.52 1.21
O46-n1318-d2 236007.13 354.64 235774.70 38.29 238350.86 58.32 0.99 236098.34 38.81 238220.44 58.85 0.94
O47-n1330-d2 256601.82 369.99 253226.30 39.17 259360.85 59.22 1.08 252670.20 39.20 258961.63 59.26 0.92
O48-n1364-d2 261990.53 413.47 253759.75 41.31 264193.95 61.35 0.84 253422.38 41.11 264086.01 61.13 0.80
O49-n1402-d2 221057.67 436.67 225343.35 46.29 222983.69 66.38 0.87 225343.35 45.79 222983.69 65.85 0.87
O50-n1500-d2 216945.08 519.33 215537.51 50.61 218073.88 70.66 0.52 215596.35 52.43 217920.27 72.47 0.45
Average 0.49 0.10
Instance VROOM Average of 10 runs Best of 10 runs
Cost Time GANCP Time

Time GANCP Time

Time
(sec) (sec) (sec) (%) (sec) (sec) (%)
D1-n105-d2 13966.79 0.37 13652.76 1.09 14272.19 2.09 2.19 13392.72 1.14 14156.04 2.14 1.35
D1-n237-d3 30436.81 3.28 30495.33 2.76 30905.40 4.27 1.54 30195.75 2.76 30667.79 4.26 0.76
D1-n255-d2 63931.22 3.21 64715.86 2.45 63669.52 12.45 -0.41 64715.86 2.46 63669.52 12.47 -0.41
D1-n350-d4 35926.04 9.40 35034.46 5.23 35814.37 7.23 -0.31 34863.14 5.28 35728.89 7.29 -0.55
D1-n416-d8 44776.34 46.41 49348.75 10.23 45202.43 13.25 0.95 49762.01 10.10 45063.58 13.40 0.64
D1-n517-d5 65750.49 46.38 65921.00 9.72 65612.61 34.72 -0.21 65597.96 9.99 65168.55 34.99 -0.89
D1-n1063-d8 74467.69 941.65 77777.84 35.07 74677.58 75.11 0.28 78612.17 35.25 74337.72 75.29 -0.17
D1-n1288-d3 135040.47 520.17 136794.39 37.30 136001.49 67.35 0.71 137565.85 36.99 135412.45 67.04 0.28
D1-n1333-d3 193723.42 580.32 206426.12 38.60 195036.98 68.77 0.68 207239.83 38.32 194840.14 68.60 0.58
D1-n1453-d6 114002.06 1740.45 113892.40 51.66 114868.64 111.70 0.76 115401.83 51.35 114433.72 111.41 0.38
D2-n225-d4 31800.99 2.91 31095.13 3.20 31880.11 5.20 0.25 31150.20 3.28 31776.15 5.28 -0.08
D2-n289-d2 46880.29 3.87 46363.22 2.98 46493.52 12.99 -0.83 46178.99 2.96 46358.03 12.97 -1.11
D2-n446-d6 50367.79 34.54 50755.17 9.33 50480.35 12.34 0.22 50906.04 9.32 50165.78 12.32 -0.40
D2-n467-d5 55440.76 29.59 54963.91 8.73 55623.93 11.25 0.33 54676.11 8.80 55492.69 11.32 0.09
D2-n469-d9 41824.28 77.51 41104.02 12.50 41361.43 17.01 -1.11 40891.80 12.49 41269.91 16.99 -1.33
D2-n577-d3 114942.42 33.47 113853.28 9.03 115449.78 24.04 0.44 113840.34 8.78 115426.24 23.81 0.42
D2-n598-d9 68269.87 176.12 68665.20 17.02 68490.47 21.53 0.32 68430.55 17.08 68341.38 21.61 0.10
D2-n905-d3 135398.79 162.65 135122.55 20.04 136227.53 50.06 0.61 135058.10 19.19 136074.30 49.23 0.50
D2-n1149-d3 121888.70 350.68 123002.16 28.34 122613.29 58.39 0.59 123829.23 28.31 122052.76 58.35 0.13
D2-n1356-d4 135082.62 868.01 132853.38 40.49 135037.44 80.54 -0.03 133021.32 39.44 134825.57 79.49 -0.19
D3-n154-d3 34138.57 0.85 34852.45 1.68 34185.13 3.19 0.14 34852.45 1.65 34185.13 3.15 0.14
D3-n177-d2 33998.02 1.01 35161.24 1.51 34182.21 2.52 0.54 35085.14 1.51 34038.42 2.51 0.12
D3-n247-d2 35903.84 1.73 37504.73 2.40 35779.37 12.40 -0.35 37325.24 2.40 35676.03 12.40 -0.63
D3-n294-d5 37730.98 5.42 39183.31 5.07 37810.46 7.57 0.21 39163.25 4.98 37666.86 7.48 -0.17
D3-n491-d6 54254.96 42.67 56483.44 10.14 54385.41 13.14 0.24 56069.70 10.05 54252.02 13.05 -0.01
D3-n521-d3 72339.41 15.95 74808.23 8.24 72132.17 23.27 -0.29 74648.36 8.16 71961.79 23.17 -0.52
D3-n1018-d5 121107.62 367.77 125479.87 27.04 121077.33 77.08 -0.03 125447.17 27.34 120871.55 77.39 -0.19
D3-n1085-d7 103426.63 672.55 108608.95 33.61 103098.52 68.63 -0.32 108683.84 33.86 102974.72 68.87 -0.44
D3-n1148-d3 166580.22 316.14 175989.32 28.27 167161.43 58.29 0.35 176851.84 28.05 166918.42 58.08 0.20
D3-n1489-d4 170502.44 932.90 176511.37 48.47 170622.06 88.52 0.07 176738.28 48.58 170456.85 88.63 -0.03
D4-n206-d4 25996.98 2.50 26096.64 2.86 26471.97 4.87 1.83 26051.33 2.91 26180.63 4.93 0.71
D4-n300-d4 37680.82 5.89 37745.89 4.43 37460.16 6.43 -0.59 37476.12 4.38 37226.41 6.38 -1.21
D4-n407-d3 57492.66 10.71 56943.06 5.64 56721.52 20.65 -1.34 56977.67 5.53 56559.23 20.53 -1.62
D4-n518-d6 54065.37 56.61 53465.05 10.60 53791.62 13.61 -0.51 53561.76 10.33 53671.92 13.33 -0.73
D4-n816-d5 86453.35 218.73 86506.00 18.79 85718.78 43.81 -0.85 86715.06 18.92 85246.81 43.93 -1.40
D4-n836-d10 72035.33 572.58 71956.42 28.38 71833.28 33.40 -0.28 71830.11 27.97 71701.61 32.99 -0.46
D4-n937-d4 108823.75 263.31 107324.33 20.81 107980.17 60.83 -0.78 107982.78 20.95 107536.19 60.97 -1.18
D4-n980-d3 141680.00 211.99 141768.74 21.74 141841.62 51.77 0.11 141460.04 21.88 141331.46 51.92 -0.25
D4-n1078-d5 114888.41 538.28 113446.01 28.80 114478.61 78.83 -0.36 113966.60 28.31 114101.77 78.34 -0.68
D4-n1325-d5 156658.45 1001.65 161866.91 40.08 156966.62 90.12 0.20 162311.84 39.57 156468.23 89.61 -0.12
D5-n173-d2 30294.37 1.13 31091.85 1.55 30348.10 2.55 0.18 31091.85 1.61 30348.10 2.61 0.18
D5-n188-d3 28977.50 1.26 30187.07 2.16 29099.04 3.32 0.42 30187.07 2.15 29099.04 3.45 0.42
D5-n327-d4 41956.74 6.85 46645.53 4.98 42675.38 6.98 1.71 47008.71 5.05 42232.63 7.05 0.66
D5-n338-d5 30268.50 7.97 33721.74 5.71 32107.76 7.96 6.08 34918.14 5.83 30788.70 8.14 1.72
D5-n376-d6 39652.81 15.35 41444.97 7.39 40189.35 9.30 1.35 41477.41 7.35 40020.94 9.26 0.93
D5-n451-d4 48405.36 18.44 51159.73 7.26 48535.83 27.27 0.27 51497.75 7.40 48424.48 27.40 0.04
D5-n481-d4 68770.32 26.24 78707.88 8.06 69258.96 28.07 0.71 80339.53 8.14 69056.36 28.14 0.42
D5-n483-d5 56995.74 30.42 62789.31 9.44 57947.16 11.10 1.67 63140.45 9.50 57624.69 11.31 1.10
D5-n774-d7 67316.12 261.44 71715.51 20.31 67433.17 55.33 0.17 71753.59 20.42 67396.33 55.45 0.12
D5-n1318-d4 133423.44 695.19 143877.62 39.36 133675.10 70.41 0.19 144482.17 39.52 133482.48 71.58 0.04
D6-n109-d2 19554.78 0.53 20783.63 1.08 19706.16 2.08 0.77 20622.64 0.99 19690.67 1.99 0.69
D6-n281-d2 45264.51 3.96 46393.95 2.95 44832.92 12.95 -0.95 46393.95 2.95 44832.92 12.95 -0.95
D6-n297-d3 40563.14 5.04 42311.02 3.51 41161.03 5.02 1.47 42917.10 3.47 40556.48 4.97 -0.02
D6-n353-d7 49174.16 21.19 54988.05 7.69 49054.30 9.91 -0.24 54988.05 7.86 49054.30 9.28 -0.24
D6-n505-d8 42845.09 86.89 42191.81 13.20 43221.34 15.46 0.88 42300.26 13.02 43167.09 16.02 0.75
D6-n603-d4 98238.34 64.31 106039.66 11.03 98178.77 31.05 -0.06 106039.66 10.94 98178.18 30.94 -0.06
D6-n827-d8 107572.54 419.15 113206.89 24.04 107848.32 64.06 0.26 115317.66 23.93 107424.27 63.96 -0.14
D6-n894-d8 63464.64 533.24 66027.29 27.12 64006.53 67.14 0.85 66402.99 26.76 63608.59 66.79 0.23
D6-n987-d9 76435.96 877.08 82446.11 32.81 76747.24 73.84 0.41 83217.42 32.77 75747.88 73.80 -0.90
D6-n1396-d5 95613.79 1296.63 101364.48 50.45 97083.59 90.52 1.54 102358.14 47.85 96779.28 87.93 1.22
D7-n100-d2 15463.60 0.25 15801.87 1.04 15618.27 2.05 1.00 15694.74 1.08 15564.03 2.08 0.65
D7-n260-d5 34545.92 3.93 35513.99 4.40 34800.25 6.91 0.74 35465.57 4.40 34721.21 6.90 0.51
D7-n287-d5 43532.64 5.40 44611.06 4.82 44037.15 7.33 1.16 44993.42 4.78 44001.48 7.30 1.08
D7-n386-d2 58990.09 6.18 62147.20 4.33 59020.77 14.34 0.05 62147.20 4.28 59020.77 14.28 0.05
D7-n399-d2 53163.55 6.46 55158.03 4.23 53328.72 14.23 0.31 55181.18 4.19 53242.81 14.19 0.15
D7-n494-d3 66277.93 13.60 69557.11 7.19 66198.80 22.20 -0.12 69547.35 7.06 66131.57 22.07 -0.22
D7-n633-d5 79355.94 88.76 83749.06 12.86 79825.00 37.88 0.59 84581.12 12.95 79491.06 37.98 0.17
D7-n661-d4 99725.00 70.69 103492.99 12.34 100014.76 32.35 0.29 103773.03 12.46 99640.68 32.48 -0.08
D7-n817-d4 96363.84 138.82 102620.72 17.17 97041.95 57.19 0.70 104288.37 17.09 96215.17 57.11 -0.15
D7-n1418-d3 227009.22 649.70 237588.70 45.00 227335.22 75.04 0.14 237577.83 44.82 227250.53 74.85 0.11
D8-n105-d2 18578.74 0.40 17076.87 1.06 19049.21 2.07 2.53 17250.87 1.11 18734.74 2.11 0.84
D8-n167-d3 32308.97 1.29 31806.10 1.88 32873.73 3.38 1.75 31981.01 1.89 32357.66 3.39 0.15
D8-n257-d3 37934.63 3.49 38107.55 2.97 38167.63 4.48 0.61 37831.69 2.92 38082.38 4.42 0.39
D8-n364-d4 48285.40 11.27 48540.66 5.58 48475.39 7.58 0.39 48388.79 5.59 48247.72 7.60 -0.08
D8-n373-d7 44323.28 23.58 44017.55 7.95 44804.36 11.47 1.09 43795.82 8.03 44554.37 11.57 0.52
D8-n593-d7 69517.24 118.01 69376.10 14.10 70043.28 17.60 0.76 69376.31 13.80 69859.77 17.31 0.49
D8-n608-d8 60892.67 161.43 61089.49 15.90 61294.88 19.90 0.66 62264.79 15.85 60812.51 19.86 -0.13
D8-n719-d7 67667.85 219.56 66864.91 18.11 67019.28 53.12 -0.96 67034.39 18.18 66704.40 53.19 -1.42
D8-n903-d6 166240.31 387.51 175076.48 24.02 167180.61 54.06 0.57 175076.48 24.19 167180.61 54.22 0.57
D8-n916-d5 133580.89 333.77 132234.50 21.76 134400.62 46.79 0.61 132684.31 21.55 134073.10 46.58 0.37
Instance NN Train Average of 10 runs Best Cost
MAPE GANCP

(%) (%) (%)
D1-n105-d2 2.96 13942.63 14241.61 1.97 14003.42 0.26
D1-n237-d3 30649.97 30842.83 1.33 30521.95 0.28
D1-n255-d2 65665.88 63684.04 -0.39 63669.52 -0.41
D1-n350-d4 35521.05 35840.60 -0.24 35703.83 -0.62
D1-n416-d8 49087.18 45652.11 1.96 45063.58 0.64
D1-n517-d5 67013.22 65652.96 -0.15 65401.51 -0.53
D1-n1063-d8 78596.76 74861.06 0.53 74337.72 -0.17
D1-n1288-d3 137359.67 136272.40 0.91 135630.55 0.44
D1-n1333-d3 210629.10 195164.53 0.74 194451.13 0.38
D1-n1453-d6 115684.75 114775.92 0.68 114433.72 0.38
D2-n225-d4 2.63 31632.60 31902.96 0.32 31867.84 0.21
D2-n289-d2 46918.01 46539.15 -0.73 46426.20 -0.97
D2-n446-d6 51089.17 50465.25 0.19 50042.01 -0.65
D2-n467-d5 55563.71 55582.34 0.26 55328.48 -0.20
D2-n469-d9 41395.62 41379.86 -1.06 41164.33 -1.58
D2-n577-d3 115646.56 115455.20 0.45 115435.88 0.43
D2-n598-d9 69659.15 68720.42 0.66 68440.77 0.25
D2-n905-d3 137167.72 136198.01 0.59 135807.24 0.30
D2-n1149-d3 124669.87 122514.31 0.51 122001.64 0.09
D2-n1356-d4 134370.72 135134.00 0.04 134713.64 -0.27
D3-n154-d3 2.33 34326.93 34240.61 0.30 34185.13 0.14
D3-n177-d2 34507.13 34208.26 0.62 34139.60 0.42
D3-n247-d2 36971.78 35781.39 -0.34 35676.03 -0.63
D3-n294-d5 38810.79 37803.97 0.19 37573.83 -0.42
D3-n491-d6 56045.25 54339.03 0.15 54193.35 -0.11
D3-n521-d3 73490.04 72077.84 -0.36 71902.43 -0.60
D3-n1018-d5 124072.60 121014.30 -0.08 120910.86 -0.16
D3-n1085-d7 107837.05 103095.26 -0.32 102974.72 -0.44
D3-n1148-d3 173215.51 167014.02 0.26 166896.44 0.19
D3-n1489-d4 174357.92 170643.84 0.08 170456.85 -0.03
D4-n206-d4 2.47 26076.19 26459.46 1.78 26125.37 0.49
D4-n300-d4 37663.94 37501.86 -0.47 37292.60 -1.03
D4-n407-d3 56813.04 56725.85 -1.33 56569.02 -1.61
D4-n518-d6 53436.93 53756.26 -0.57 53436.96 -1.16
D4-n816-d5 85951.59 85780.22 -0.78 85315.78 -1.32
D4-n836-d10 71953.59 71916.23 -0.17 71514.54 -0.72
D4-n937-d4 107010.06 108090.90 -0.67 107536.19 -1.18
D4-n980-d3 141237.94 141740.39 0.04 141536.29 -0.10
D4-n1078-d5 113502.69 114589.73 -0.26 114101.77 -0.68
D4-n1325-d5 160935.03 157322.77 0.42 156682.65 0.02
D5-n173-d2 2.77 30466.93 30348.10 0.18 30348.10 0.18
D5-n188-d3 29862.00 29099.04 0.42 29099.04 0.42
D5-n327-d4 46120.28 42588.49 1.51 42140.71 0.44
D5-n338-d5 33798.17 31910.36 5.42 30788.70 1.72
D5-n376-d6 40786.67 40192.50 1.36 39914.12 0.66
D5-n451-d4 50430.81 48562.02 0.32 48375.86 -0.06
D5-n481-d4 76393.72 69278.08 0.74 69056.36 0.42
D5-n483-d5 61121.39 57776.18 1.37 57487.98 0.86
D5-n774-d7 70421.67 67416.21 0.15 67249.18 -0.10
D5-n1318-d4 140806.85 133638.49 0.16 133475.78 0.04
D6-n109-d2 3.25 20522.49 20500.15 4.83 19694.74 0.72
D6-n281-d2 46500.92 44888.44 -0.83 44832.92 -0.95
D6-n297-d3 42291.72 41318.31 1.86 40556.48 -0.02
D6-n353-d7 54116.73 49054.30 -0.24 49054.30 -0.24
D6-n505-d8 42248.37 43172.96 0.77 42835.30 -0.02
D6-n603-d4 105251.59 98134.19 -0.11 97844.78 -0.40
D6-n827-d8 113372.67 107713.44 0.13 107424.27 -0.14
D6-n894-d8 66399.56 63904.05 0.69 63608.59 0.23
D6-n987-d9 82182.51 76633.12 0.26 75707.93 -0.95
D6-n1396-d5 101940.88 97025.46 1.48 96699.53 1.14
D7-n100-d2 2.39 15497.17 15615.76 0.98 15564.03 0.65
D7-n260-d5 35089.76 34765.85 0.64 34627.01 0.23
D7-n287-d5 44093.41 43983.45 1.04 43662.04 0.30
D7-n386-d2 60757.73 59020.77 0.05 59020.77 0.05
D7-n399-d2 54521.13 53279.57 0.22 53154.32 -0.02
D7-n494-d3 68174.61 66270.05 -0.01 66108.04 -0.26
D7-n633-d5 82975.44 79735.48 0.48 79305.34 -0.06
D7-n661-d4 101598.46 99972.92 0.25 99767.82 0.04
D7-n817-d4 101152.71 96973.41 0.63 96787.28 0.44
D7-n1418-d3 231700.74 227340.58 0.15 227250.53 0.11
D8-n105-d2 2.65 16886.40 19065.78 2.62 18734.74 0.84
D8-n167-d3 31202.18 32666.51 1.11 32233.83 -0.23
D8-n257-d3 37548.12 38078.19 0.38 37937.88 0.01
D8-n364-d4 48086.85 48516.30 0.48 48362.44 0.16
D8-n373-d7 43940.60 44746.32 0.95 44617.18 0.66
D8-n593-d7 68594.52 70044.67 0.76 69760.75 0.35
D8-n608-d8 60958.69 61195.09 0.50 60653.55 -0.39
D8-n719-d7 66615.03 67015.08 -0.96 66754.76 -1.35
D8-n903-d6 170608.56 167625.11 0.83 167180.61 0.57
D8-n916-d5 131052.01 134166.04 0.44 134073.10 0.37
Instance Best Hybrid ALNS

Known Best Cost Time Opened Gap
Cost Time Cost Time Cost Time Depots Vehicles
(sec) (sec) (sec) (sec) (%)
111112 1467.68 1467.68 22.80 1467.68 345.07 1467.68 1467.68 546 1500.42 3.35 2 11 2.23
111122 1448.37 1449.20 24.33 1448.37 369.68 1448.37 1448.37 451 1453.89 2.99 2 11 0.38
111212 1394.80 1396.46 22.36 1394.80 326.02 1394.80 1394.80 312 1427.53 4.01 2 11 2.35
111222 1432.29 1432.29 20.32 1432.29 336.30 1432.29 1432.29 297 1435.36 3.97 2 11 0.21
112112 1167.16 1167.16 20.73 1167.16 354.64 1167.16 1167.16 234 1217.84 3.35 2 12 4.34
112122 1102.24 1102.24 16.16 1102.24 354.15 1102.24 1102.24 764 1102.24 3.50 2 10 0.00
112212 791.66 791.82 16.32 791.66 324.80 791.66 791.66 342 791.66 3.21 2 11 0.00
112222 728.30 728.30 12.74 728.30 294.14 728.30 728.30 261 728.30 3.15 2 11 0.00
113112 1238.24 1238.67 19.99 1238.49 349.72 1238.49 1238.49 465 1285.25 3.30 2 11 3.80
113122 1245.30 1245.42 18.91 1245.31 390.48 1245.30 1245.42 597 1255.51 3.32 2 11 0.82
113212 902.26 902.33 15.61 902.26 311.39 902.26 902.26 176 902.26 3.28 3 12 0.00
113222 1018.29 1022.79 18.79 1018.29 331.00 1018.29 1018.29 247 1051.20 3.62 3 11 3.23
121112 2237.73 2267.30 125.30 2238.52 1773.32 2237.73 2238.37 1075 2307.14 3.34 3 22 3.10
121122 2137.45 2168.06 107.41 2138.38 1801.37 2137.45 2139.67 856 2264.78 4.87 3 22 5.96
121212 2195.17 2207.35 145.34 2206.69 2110.47 2195.17 2219.14 952 2257.68 3.69 3 22 2.85
121222 2214.86 2227.98 130.88 2214.86 2181.95 2214.86 2240.13 705 2276.02 3.77 4 22 2.76
122112 2070.43 2081.23 116.85 2072.92 1947.71 2070.43 2084.67 863 2096.32 3.25 2 21 1.25
122122 1685.52 1685.54 127.88 1685.65 1974.23 1685.52 1685.65 812 1712.32 4.34 3 21 1.59
122212 1449.93 1452.41 127.69 1449.93 1703.98 1449.93 1456.63 647 1469.70 3.08 2 20 1.36
122222 1082.46 1083.39 100.80 1082.46 1600.30 1082.46 1082.78 890 1160.78 3.51 3 22 7.24
123112 1942.23 1959.25 120.39 1943.10 1746.15 1942.23 1954.70 915 1974.40 4.36 4 22 1.66
123122 1910.08 1926.54 156.82 1910.66 1618.94 1910.08 1918.16 971 2102.32 4.58 4 22 10.06
123212 1760.84 1764.63 103.33 1763.20 1565.78 1761.11 1760.84 1100 1778.96 3.33 2 22 1.03
123222 1390.86 1391.87 90.97 1391.46 1484.46 1390.86 1395.48 712 1423.86 4.93 5 22 2.37
131112 1866.75 1904.28 55.77 1895.48 1104.45 1892.17 1900.68 462 1944.22 3.42 2 15 4.15
131122 1819.68 1824.47 60.61 1819.68 901.59 1819.68 1822.95 316 1880.57 4.64 3 16 3.35
131212 1960.02 1967.80 48.78 1960.02 989.23 1960.02 1969.40 751 2038.57 3.18 3 17 4.01
131222 1792.77 1798.38 63.18 1795.84 879.03 1792.77 1795.84 843 1879.86 3.45 2 16 4.86
132112 1443.32 1443.32 72.00 1443.32 776.99 1443.32 1445.68 912 1531.33 3.90 3 16 6.10
132122 1429.30 1441.86 72.50 1429.52 941.75 1429.30 1431.40 754 1446.68 3.54 2 16 1.22
132212 1204.42 1205.10 51.26 1204.62 880.01 1204.42 1204.42 822 1204.42 3.01 3 17 0.00
132222 924.68 925.95 54.82 925.05 738.18 924.68 924.68 945 998.24 3.05 2 17 7.96
133112 1694.18 1694.18 49.59 1694.18 828.29 1694.18 1694.18 931 1757.34 3.97 2 16 3.73
133122 1392.00 1404.21 55.81 1392.18 882.56 1392.01 1396.34 854 1408.83 4.01 4 16 1.21
133212 1197.95 1198.53 42.24 1198.08 877.64 1197.95 1198.31 860 1217.50 3.08 3 17 1.63
133222 1151.37 1152.49 57.89 1151.83 771.61 1151.37 1152.30 647 1214.22 3.99 4 17 5.46
Average 65.75 1004.65 674.64 3.65 2.84

Remarks for Table LABEL:tab:clrp_tuzun_detailed :

TBSA and Hybrid ALNS results (inclusive of run times) have been directly taken from Schneider and Löffler ( 2019 ) and Akpunar and Akpinar ( 2021 ) , respectively.

TBSA speed subscript TBSA speed \text{TBSA}_{\text{speed}} TBSA start_POSTSUBSCRIPT speed end_POSTSUBSCRIPT is the fast-variant and TBSA qual subscript TBSA qual \text{TBSA}_{\text{qual}} TBSA start_POSTSUBSCRIPT qual end_POSTSUBSCRIPT is the quality-oriented variant of TBSA.

TBSA ¯ ¯ TBSA \mkern 1.5mu\overline{\mkern-1.5mu\text{TBSA}\mkern-1.5mu}\mkern 1.5mu over¯ start_ARG TBSA end_ARG shows the best result from all four variants of TBSA proposed in Schneider and Löffler ( 2019 ) .

Instance Best Hybrid ALNS

Known Best Cost Time Opened Gap
Cost Time Cost Time Depots Vehicles
(sec) (sec) (sec) (%)
Christofides69-50x5 565.6 565.6 1.94 565.6 60 592.9 2.90 1 5 4.83
Christofides69-75x10 844.4 848.9 12.14 848.9 110 855.8 3.41 2 9 1.35
Christofides69-100x10 833.4 833.4 25.57 833.4 501 839.7 3.13 2 8 0.76
Daskin95-88x8 355.8 355.8 6.44 355.8 49 355.8 3.18 2 8 0.00
Daskin95-150x10 43919.9 43919.9 51.65 43919.9 563 47303.6 7.35 4 14 7.70
Gaskell67-21x5 424.9 424.9 0.65 424.9 2 429.6 3.37 2 5 1.11
Gaskell67-22x5 585.1 585.1 0.47 585.1 6 585.1 3.01 1 3 0.00
Gaskell67-29x5 512.1 512.1 0.76 512.1 11 512.1 2.57 2 4 0.00
Gaskell67-32x5-1 562.2 562.2 1.07 562.2 5 562.2 3.02 1 4 0.00
Gaskell67-32x5-2 504.3 504.3 0.78 504.3 25 504.3 2.70 1 3 0.00
Gaskell67-36x5 460.4 460.4 0.87 460.4 26 460.4 2.54 1 4 0.00
Min92-27x5 3062 3062 0.74 3062 15 3062 2.85 2 4 0.00
Min92-134x8 5709 5709 29.11 5709 285 6112 4.95 4 11 7.06
Average 10.17 127.54 3.46 1.75

Remarks for Table LABEL:tab:clrp_barreto_detailed :

TBSA speed subscript TBSA speed \text{TBSA}_{\text{speed}} TBSA start_POSTSUBSCRIPT speed end_POSTSUBSCRIPT is the fast-variant of the TBSA. The quality-oriented variant of TBSA, TBSA qual subscript TBSA qual \text{TBSA}_{\text{qual}} TBSA start_POSTSUBSCRIPT qual end_POSTSUBSCRIPT , produced the same results as TBSA speed subscript TBSA speed \text{TBSA}_{\text{speed}} TBSA start_POSTSUBSCRIPT speed end_POSTSUBSCRIPT with an average run time of 171.43 seconds for Barreto instances.

The Vehicle Routing Problem (VRP) Demystified: Solutions for Your Delivery Operations

  • By Rakesh Patel
  • Last Updated: September 13, 2024

vehicle-routing-problem

  • VRP is a complex real-life optimization problem.
  • Various types of VRPs exist, each with its own set of challenges and side constraints.
  • Route Optimization techniques and algorithms, like Upper Route Planner, can efficiently solve VRPs and reduce idle time.

The vehicle routing problem is a serious and one of the oldest challenges in the logistics and delivery business. George Dantzig and John Ramser first represented it in 1959, introducing it as a mixed-integer linear problem (MILP). This has been studied extensively, using commercial solvers to address the vehicle capacities of fleets and customer requests.

You must have heard about the Traveling Salesman Problem (TSP). It is a subset of VRP and the only difference between TSP, and VRP is that TSP is a single-route node-service-combination problem, while VRP manages multiple routes across various customers and service points.

TSP vs VRP

Source: Researchgate.com

In this blog, we will provide a detailed guide on vehicle routing problem (VRP), its types, and potential solutions.

So, let’s jump into it!

Forget Spaghetti Routes, Optimize Routes for Your Entire Team with Upper

Crew

Table of Contents

What is the Vehicle Routing Problem (VRP)?

Why you need to solve vehicle routing problem, perks of solving vehicle routing problems, what are the different ways to solve vehicle routing problem.

The vehicle routing problem is a classic problem in the logistics and delivery business. It causes inefficiencies like longer waiting times, inaccurate delivery routes, increased travel costs, wastage of time & resources, and customer dissatisfaction.

Vehicle routing problem is a problem that involves finding the most efficient way to carry out delivery operations and deliver goods with a multi-route using a fleet of vehicles.

The goal of this problem is to find the best possible set of routes for the vehicles to reach every stop while minimizing the total distance traveled, the number of vehicles used, delivery time, and fuel costs. This will also consider other factors, including the capacity of each vehicle, the time window, and the service time required per stop.

One way to solve this is the application of dynamic programming techniques and evolutionary algorithms to reach optimal solutions. These solutions help satisfy the maximum number of customer requests within specific constraints.

To get a detailed explanation of what is vehicle routing problem, do check the below-given video:

Types of Vehicle Routing Problems

There are several types of vehicle routing problems that affect logistics and operations to a great extent. Let’s talk about them one by one:

1. Traveling Salesman Problem (TSP)

Traveling Salesman Problem is a problem whose goal is to figure out the shortest possible route that can be assigned to a salesman. By following this route they can visit a set of cities and return to their starting location, covering each stop once. As the number of cities increases, finding the optimal solution becomes exponentially difficult. So, this problem is an NP-hard problem. Manufacturing, logistics, and transportation are some of the real-world applications of TSP. 

2. Capacitated Vehicle Routing Problem (CVRP)

Capacitated vehicle routing problem is a problem related to the capacity of delivery vehicles to carry parcels. The goal of CVRP is to figure out an optimal set of routes to reach each customer on time, considering the capacity of each delivery vehicle. This problem is known as mixed-integer linear problem (MILP) and is considered NP-hard. CVRP is widely studied and used as a benchmark problem for the development of new algorithms and heuristics. 

Solving complex delivery problems like Capacitated Vehicle Routing Problem (CVRP) just got easier with Upper.

Here is how:

  • Our advanced capacity optimization intelligently balances vehicle loads, enabling you to handle more deliveries in fewer trips.
  • By setting the exact load capacity for each vehicle in your fleet, based on its type, Upper ensures that every vehicle is utilized to its full potential.
  • This prevents overloading or underutilizing vehicles, helping you maximize your fleet’s potential.
  • With our system, you can reduce travel costs, ensure timely deliveries, and minimize operational complexities—all while keeping operations efficient and cost-effective.

crossline

3. Vehicle Routing Problem with Time Windows (VRPTWs)

In the vehicle routing problem with time windows (VRPTWs) , delivery drivers have to serve each customer in a specific time window, and the objective is to find the most efficient route for the drivers to serve each customer on time.

This problem is also formulated as MILP and is known to be NP-hard. This problem is more complex as time constraints should also be considered along with capacity constraints. VRPTWs have many real-world applications and use cases.

4. Pickup and Delivery Problem (PDP)

In the pickup and delivery problem (PDP) , the delivery drivers must visit both pick-up and delivery locations. The objective of this problem is to determine an optimal set of routes for the vehicles to visit pick-up and delivery locations while considering capacity and time constraints. 

Just like other VRPs, PDP is also formulated as MILP and is known to be NP-hard. PDVRP has many real-life applications, and one of them is transporting goods from factories to warehouses.

5. Vehicle Routing Problem with Profits (VRPP)

In a delivery business, each customer is associated with a known profit and the objective of the vehicle routing problem with profits is to determine the optimal set of routes to reach customers while maximizing the total profit. Again, this problem is NP-hard and formulated as MILP. Delivering high-value goods or services is one of the applications of VRPP.

6. Vehicle Routing Problem with Multiple Trips (VRPMT)

In Vehicle Routing Problems with Multiple Trips (VRPMT), delivery drivers are required to cover multiple routes to complete deliveries to a set of customers. The goal of this VRP is to find an optimal solution for planned routes considering capacity and time constraints along with managing multiple trips. This VRP is considered NP-hard and is formulated as a MILP. As an example, we can consider the distribution of goods to retailers or grocery stores.

7. ​​Open Vehicle Routing Problem (OVRP)

In the open vehicle routing problem (OVRP) , there is no fixed starting point for the delivery vehicles. The main objective of this vehicle routing problem is to figure out a set of optimal routes for the delivery vehicles to reach to maximum customer while minimizing the travel time, distance, or any other routing constraints. One of the real-world applications of this NP-hard problem is the transportation of goods or services in urban areas.

8. Periodic Vehicle Routing Problem (PVRP)

Periodic Vehicle Routing Problem (PVRP) involves determining an optimal set of routes for delivery drivers to serve a set of customers who require periodic deliveries or pickups. Unlike VRP, in PVRP, allows delivery drivers to visit customers multiple times, according to a predefined schedule. Real-world applications of PVRP includes waste management where garbage is collected on a fixed day of every week or month.

9. Stochastic Vehicle Routing Problem (SVRP)

When there is uncertainty in customer demands and travel times, Stochastic Vehicle Routing Problem (SVRP) arises. Solution of this type of VRP is probability-based. This is a significant problem in the industries where demand and travel time varies a lot like delivering groceries and food during the rush hour traffic or rainy day. 

To solve this problem, you need special methods that can handle uncertain situations and give convincing results even when things don’t go as planned.

10. Multi-Depot Vehicle Routing Problem (MDVRP)

The goal of the Multi-Depot Vehicle Routing Problem (MDVRP) is to find the best set of routes for a fleet of vehicles to follow in order to serve a group of clients from multiple depots. The MDVRP aims to reduce the overall distance traveled by all vehicles for serving all customers. 

Each depot has a set of vehicles and a set of customers that it can serve. The MDVRP is applicable to sectors like trash management or public transportation where there are numerous depots that need to be maintained and each station has its own set of clients and vehicles to serve. 

11. The Green Vehicle Routing Problem (GVRP)

The Green Vehicle Routing Problem (GVRP) , another variation of vehicle routing problem (VRP), considers factors such as fuel consumption, traffic congestion, and vehicle emissions to minimize the environmental impact of transportation. Industries looking to reduce their carbon footprint and comply with environmental regulations implement GVRP. 

GVRP requires the use of optimization that helps minimize transportation costs and minimizing environmental impact by involving the use of alternative fuels or vehicle technologies. Also developing environmental friendly routing strategies comprises the goal of solving the Green Vehicle Routing Problem (GVRP).

12. Dynamic Vehicle Routing Problem (DVRP)

In the Dynamic Vehicle Routing Problem (DVRP) ,  the problem parameters such as customer demand, travel time, and even adjustments to existing routes are not fixed but rather evolve over time in response to changes in the environment or customer behavior. DVRP is relevant to industries such as on-demand delivery, where the demand varies on factors such as time of day or location. 

The goal of DVRP is to create and implement routing strategies that can dynamically respond to changes in the environment and customer demand, while also adjusting existing routes as needed to minimize transportation costs throughout the journey.

13. Pickup and Delivery Problem with Time Windows (PDPTW)

In the Pickup and Delivery Problem with Time Windows (PDPTW) , the goal is to find the best possible routes for a fleet of vehicles to travel in order to pick up goods from a set of locations and deliver them to a different set of locations while keeping in mind time window restrictions. Each location has a particular time window during which the pickup or delivery operation can take place. 

The PDPTW aims to reduce the overall distance traveled by all vehicles while completing all pickup and delivery requests within each location’s designated time window.  In some cases, pickups and deliveries are required to be handled by the same vehicle, ensuring that goods are transported directly without unnecessary transfers.

The PDPTW applies to sectors like package delivery or freight transportation where timely and effective scheduling of pickup and delivery operations is necessary to meet client demands.

14. Inventory Routing Problem (IRP)

Identifying the best inventory and delivery schedule for a business that wants to restock its inventory at several locations while reducing transportation costs is the goal of the Inventory Routing Problem (IRP) . The objective of the IRP is to identify the optimum delivery frequency, amount, and route for each location in order to maintain inventory levels within a specific range and reduce transportation costs. 

The IRP is relevant for industries such as fuel delivery or waste management, where inventory levels need to be maintained at multiple locations and transportation costs can be a significant part of the overall operation cost. 

15. Traveling Salesman Problem with Time Windows (TSPTW)

Finding the most efficient route a salesperson can take to visit a group of customers within their designated time windows is the goal of the Traveling Salesman Problem with Time Windows (TSPTW). It is a version of the traditional Traveling Salesman Problem (TSP).

The objective of the TSPTW is to discover the shortest route that visits all customers during their respective time windows. Each client has a defined time window during which they can be visited. The TSPTW is applicable to sectors like transportation where meeting deadlines is necessary to assure timely delivery or service.

16. Vehicle Routing Problem with Simultaneous Pickup and Delivery (VRPSPD)

Finding the most efficient set of routes for a fleet of vehicles to simultaneously pick up goods from some sites and deliver items to other places is the goal of the Vehicle Routing Problem with Simultaneous Pickup and Delivery (VRPSPD) . 

The VRPSPD aims to reduce the overall distance traveled by all vehicles while ensuring that each vehicle’s capacity is not exceeded. Each stop can either be a pickup or a delivery point. The VRPSPD is applicable to sectors like waste collection or postal delivery where simultaneous pickup and delivery tasks must be coordinated well to reduce transportation costs. 

17. Dial-a-Ride Problem (DARP)

This variation of the Vehicle Routing Problem (VRP) involves finding optimal routes for a fleet of vehicles to handle a variety of pick-up and drop-off requests from different passengers. The objective of the Dial-a-Ride Problem (DARP) is to reduce the overall travel distance and time for all cars while completing all pickup and drop-off requests. Each passenger specifies their pickup and drop-off locations as well as the desired pickup and drop-off timings.

The DARP applies to sectors like healthcare or public transit when it is necessary to transport people quickly and efficiently from one place to another. Advanced optimization algorithms are required by the DARP because they can handle the complexity of each passenger request and produce workable solutions that reduce transportation costs while maintaining high levels of service quality.

18. School Bus Routing Problem (SBRP)

To determine the best set of routes for a fleet of school buses to convey students from their homes to schools and back, one must solve the “ School Bus Routing Problem ” (SBRP), another vehicle routing problem. Each student in the SBRP has a specified pick-up and drop-off location and time.

The objective is to reduce the overall travel distance and time for all buses while making sure that no student is missed or comes late. The SBRP is relevant for school transportation systems, where safety, efficiency, and reliability are critical for the transportation of students.

19. Arc Routing Problem (ARP)

The goal of the Arc Routing Problem (ARP) is to find the best set of routes for a fleet of vehicles to service several arcs or edges in a transportation network. The objective of the ARP is to reduce the overall distance traveled by all vehicles while ensuring that each arc or edge is serviced within a predetermined time window.

Each arc or edge in the network must be visited by one or more vehicles. The ARP is relevant for sectors like trash pickup or mail delivery where it’s important to effectively service network edges to save on transportation expenses.

20. Chinese Postman Problem (CPP)

The Chinese Postman Problem (CPP) is a graph theory problem that aims to determine the quickest way for a postman to visit each street in a particular city or region at least once before heading back to the starting point. Being an NP-hard problem, it is difficult to develop an effective algorithm that can tackle this problem for huge graphs. The CPP has uses in circuit design, DNA sequencing, and transportation planning. Many methods, most notably Hierholzer’s algorithm, which effectively finds the best solution for Eulerian graphs, have been developed to solve the CPP.

21. Zone Routing Problem (ZRP)

The Zone Routing problem (ZRP) is a transportation optimization problem that entails choosing the most optimal set of routes for a fleet of cars to travel to cover several zones or locations. The objective of the ZRP is to reduce the overall distance traveled by all vehicles while ensuring that each zone or region is serviced within a predetermined time window.

Each zone or region must be served by one or more vehicles. The ZRP is applicable to sectors like public transportation or healthcare where it is important to efficiently serve particular zones or areas to minimize the cost of transportation.

22. Pickup and Delivery Problem with Profits (PDPP)

The goal of the Pickup and Delivery Problem with Profits (PDPP) is to choose the best set of routes for a fleet of vehicles to pick up and deliver goods to clients while maximizing the profit made from each delivery. 

The objective of the PDPP is to maximize the total profit made by the vehicle fleet while guaranteeing that all pickups and deliveries are done within a predetermined time window. Each customer has a designated pickup and delivery location and time. The PDPP is applicable to sectors like e-commerce or logistics, where businesses strive to optimize earnings by streamlining their delivery processes.

23. Capacitated Arc Routing Problem (CARP)

The Capacitated Arc Routing Problem (CARP) is a transportation optimization problem in which the goal is to find the best possible set of routes for a fleet of vehicles to travel to service a collection of edges or arcs in a graph, each of which has a limited capacity. Every arc has a demand in the CARP, and the objective is to reduce the overall distance traveled by all cars while making sure that no vehicle’s capacity is exceeded.

The CARP applies to sectors like trash management or products transportation where the objective is to reduce transportation costs while maintaining high levels of service quality. Each vehicle has a restricted carrying capacity in these sectors.

Facing Any of the Vehicle Routing Problems Listed Above?

Worry no more! Get Upper to overcome your routing problems with a fully automated process and increase your efficiency.

All businesses that include the process of planning and optimizing delivery routes face vehicle routing problems. Solving VRP using commercial solvers or route optimization software will reduce logistics expenses, improve fleet utilization, and lower travel costs. 

This includes determining the cost-efficient, effective, and time-saving way to deliver goods or services to several customers while considering various factors like time, distance, fuel cost, profitability, vehicle capacity, time windows, and traffic conditions.

By leveraging tools that optimize based on business requirements, companies can drastically cut down on unnecessary fuel usage and minimize idle time. Solving VRPs also ensures that customer requests are met promptly, which improves overall satisfaction.

There are several benefits of solving vehicle routing problems:

1. Reduce logistics expenses

The main goal of adopting an effective route planning solution is to cut down on logistics expenses significantly. By optimizing delivery routes and minimizing the number of vehicles required for transportation, you can notice a huge impact on your daily logistics expenses. Also, by minimizing fuel costs and other operational costs you can improve your business’s profitability.

2. Achievable sustainable growth

By reducing carbon footprints, an effective VRP solution can help your business achieve sustainable growth. You can reduce the environmental impact and contribute to a more sustainable future by optimizing your delivery routes and reducing the number of vehicles on the road.

3. Save time and increase customer satisfaction

By optimizing delivery routes and saving delivery times using an effective VRP solution can save a lot of time spent in manual route planning. Hence you can utilize this time in other productive tasks of your business and ultimately leading to higher customer satisfaction. This increases the chances of customer retention and repeat business.

Solve Your VRPs, Streamline Your Logistics & Increase Profitability With Upper

Optimize your transportation operations and get rid of all sorts of vehicle routing problems your business faces.

As we are on the same page, knowing what exactly is vehicle routing problem and its types. Now it is the time to look at its solution. Vehicle Routing problems have several solutions. They are:

1. Manually using two-index flow formulation

In the two-index VF formulation, we define the binary decision variable xij that assumes value 1 if and only if there is a route that goes from customer i to j directly, for i, j ∈ N . In addition, yj is a continuous decision variable corresponding to the cumulated demand on the route that visits node j ∈ N up to this visit. With these parameters and decision variables, the two-index flow formulation of the CVRP if given by:

Manually using two index flow formulation

Source: arxiv.org

Do you know?

Time required to solve a VRP with 10 nodes is 3 milliseconds, while a VRP with 20 nodes will take 77 years to get solved!

The VRP has an order of n! potential solutions, where n is the number of network nodes (locations the vehicle must travel to). In the brute-force method, factors like factorial growth rate of possible solutions and assuming that the machine used for enumeration can execute one billion operations per second are considered. The total time required to solve the VRP using this brute-force method for various numbers of nodes is shown in the below-given table:

problem size vs solution time

2. Using Google OR-tools

Google OR tools provide a set of powerful optimization tools for solving various types of vehicle routing problems. Here are the steps to use Google OR-tools VRP solver:

Step-1: Define the problem

Firstly, you have to define the problem by specifying the number of vehicles, set of customers, delivery stops, and other delivery constraints such as vehicle capacity and time windows.

Step-2: Formulate the problem

Then, the next step is to formulate the problem as a mathematical model using the OR-tools routing library. The model should consider the objective function, the constraints, and the decision variables.

Step-3: Define the search strategy

To find the optimal solution, the third step is to define the search strategy. The Google OR tools provide several strategies, such as local search, guided local search, and simulated annealing.

Step-4: Solve the problem

The last step is to solve the problem using the OR-tools solver. Here is the general code outline to solve a VRP using Google OR tools:

from ortools.constraint_solver import routing_enums_pb2

from ortools.constraint_solver import pywrapcp

# Define the problem

num_vehicles = …

demands = …

depot = …

time_windows = …

# Create the routing model

routing = pywrapcp.RoutingModel(num_vehicles, …)

# Define the cost function

def cost_function(i, j):

  …

# Define the capacity constraint

def demand_callback(i):

 …

# Define the time window constraint

def time_callback(i, j):

# Define the search strategy

search_parameters = pywrapcp.DefaultRoutingSearchParameters()

search_parameters.first_solution_strategy = routing_enums_pb2.FirstSolutionStrategy.PARALLEL_CHEAPEST_INSERTION

# Solve the problem

assignment = routing.SolveWithParameters(search_parameters)

# Print the solution

if assignment:

    print_solution()

3. Using route optimization software

The problem with solving vehicle routing problems using the above 2 methods is the two-index flow formulation method is too lengthy and humanly complex to perform, while Google OR tools require a lot of programming knowledge and proficiency in coding. Also, the chances of generating errors are and once an error is generated, you are back to square one.

Now the question arises, isn’t there a single method that is easy to perform and quick to get solutions to various vehicle routing problems?

The answer is yes, By using proficient route optimization software, you can get quick and easy solutions to VRPs.

What is Route Optimization?

Route optimization is a process of finding the most optimal route including all the stops you need to cover considering factors like number of stops, number of delivery vehicles & drivers, time window, break time, and priority.

Upper Route Planner is one such route optimization software that helps you solve VRPs with its powerful route planning and optimization methods.

Using Upper Route Planner, all you need is a list of stops and available drivers, feed the data in the software, wait for a few seconds, and your optimized routes will be ready in a blink of an eye.

The goal of VRP is to minimize the total cost of delivering goods to customers considering various constraints like distance, time, capacity, fuel costs, profitability, and time window. Route optimization on the other hand does the same task automatically. Algorithms of route optimization software helps you find the optimal routes that minimize overall delivery costs.

Using Upper will lead you to reduced fuel costs, reduced vehicle maintenance costs, and improved customer satisfaction. So, you worry less about customer retention and focus more on serving new customers as well.

Another benefit of using route optimization software like Upper is that you get many other features along with route planning and optimization like:

  • Route Scheduling
  • Smart Excel Import
  • One-click Dispatch
  • Proof of Delivery
  • Reports and Analytics
  • API Integration
  • Business-Specific Customization

VRP With Just 20 Nodes and 77 Years to Solve it? ?

Get your business, Upper Route Planner and solve VRP with hundreds of stops in just a few minutes.

Consider a company that employs a fleet of delivery trucks to deliver goods to its customers. Each of the company’s customers has particular delivery needs, including the quantity of goods to be delivered and the delivery time. By reducing the trucks’ travel distance while still meeting all delivery standards, the company aims to maximize its delivery operations.

This problem is known as a vehicle routing problem (VRP), the goal of which is to determine the best routes for the delivery vehicles in order to satisfy all delivery requirements while minimizing the overall distance traveled.

Vehicle Routing Problem (VRP) is a commonly known problem among the delivery businesses. It can be difficult to solve especially for large instances. The difficulty of solving VRP depends on various factors like the number of customers, the number of vehicles, the vehicle capacity, the routing constraints, and the objective function.

There are several manual methods and Google OR-Tools available but they make it more complex to solve. The easiest way is to use a route optimization software that solves the VRP in a matter of a few seconds.

For such problems, one solution is to use a rolling horizon optimization algorithm, which solves the problem for a fixed time horizon and then updates the solution as new demand arises. Another solution is to use a stochastic optimization algorithm that considers the uncertainty in the demand and generates solutions that are robust to changes in demand.

Vehicle Routing Problem is a complex optimization problem that arises in real-life where the challenge is to find an optimized and efficient route for delivery operations considering various constraints and minimizing delivery costs.

There are several types of VRP, each having unique challenges and solutions. Solving VRPs can result in a reduction in operational costs and a positive graph can be seen in sales. There are various methods to solve VRP ranging from exact methods such as branch and bound and dynamic programming to heuristic and metaheuristic algorithms such as Clarke and Wright savings algorithm, sweep algorithm, and genetic algorithms.

But with increasing automation, these methods have gone outdated and are considered time consuming & complex. So, if you are a delivery business owner facing Vehicle Routing Problems (VRPs), it is the right time to switch to a robust route optimization software that is fast, reliable, and easy-to-use.

When looking for such software, Upper Route Planner is the best choice available in the market. Before trying anything else, get your business the 7 days FREE TRIAL of Upper and start exploring its features.

Rakesh Patel

Rakesh Patel, author of two defining books on reverse geotagging, is a trusted authority in routing and logistics. His innovative solutions at Upper Route Planner have simplified logistics for businesses across the board. A thought leader in the field, Rakesh's insights are shaping the future of modern-day logistics, making him your go-to expert for all things route optimization. Read more.

Arrow

Related Posts

How to do efficient delivery route planning

Efficient Route Planning: 9-Steps to Optimizing Delivery Operations

Save money and time using route planning software

How to Save Money and Time Using Route Planning Software

What is route optimization engine

What is a Route Optimization Engine? 5 Benefits of a Routing Engine for Business

Shortest vs fastest route

Shortest vs Fastest Route: What’s Best for Your Delivery Business?

How to solve a dynamic vehicle routing problem

Dynamic Vehicle Routing Problem: How Do You Solve It?

What is route planning and route optimization software

What is Route Planning & Route Optimization Software? – Everything You Need to Know

Sign Up with Upper Route Planner and automate your daily business process route planning, scheduling, and optimizing!

Grab a FREE Trial of Upper

  • Plan routes with hundreds of stops in a minute
  • Schedule routes months in advance
  • Collect reliable proof of delivery
  • Track drivers live for real-time updates
  • Experience unparalleled customer support

Grab a FREE Trial of Upper TODAY!

  • Schedule routes in advance for weeks
  • Collect proof of delivery to maintain accountability
  • Experience 24/7 customer support
  • Smart reporting to get real-time insights

Google OR-Tools

  • Google OR-Tools
  • Español – América Latina
  • Português – Brasil
  • Tiếng Việt

Vehicle Routing

One of the most common optimization tasks is vehicle routing , in which the goal is to find the best routes for a fleet of vehicles visiting a set of locations. Usually, "best" means routes with the least total distance or cost. Here are a few examples of routing problems:

  • A package delivery company wants to assign routes for drivers to make deliveries.
  • A cable TV company wants to assign routes for technicians to make residential service calls.
  • A ride-sharing company wants to assign routes for drivers to pick up and drop off passengers.

The most famous routing problem is the Traveling Salesperson Problem (TSP): find the shortest route for a salesperson who needs to visit customers at different locations and return to the starting point. A TSP can be represented by a graph, in which the nodes correspond to the locations, and the edges (or arcs) denote direct travel between locations. For example, the graph below shows a TSP with just four locations, labeled A, B, C, and D. The distance between any two locations is given by the number next to the edge joining them.

By calculating the distances of all possible routes, you can see that the shortest route is ACDBA, for which the total distance is 35 + 30 + 15 + 10 = 90 .

The problem gets harder when there are more locations. In the example above, there are just six routes. But if there are ten locations (not counting the starting point), the number of routes is 362880. For 20 locations, the number jumps to 2432902008176640000. An exhaustive search of all possible routes would be guaranteed to find the shortest, but this is computationally intractable for all but small sets of locations. For larger problems, optimization techniques are needed to intelligently search the solution space and find an optimal (or near-optimal) solution.

A more general version of the TSP is the vehicle routing problem (VRP), in which there are multiple vehicles. In most cases, VRPs have constraints: for example, vehicles might have capacities for the maximum weight or volume of items they can carry, or drivers might be required to visit locations during specified time windows requested by customers.

OR-Tools can solve many types of VRPs, including the following:

  • Traveling Salesperson Problem , the classic routing problem in which there is just one vehicle.
  • Vehicle routing problem , a generalisation of the TSP with multiple vehicles.
  • VRP with capacity constraints , in which vehicles have maximum capacities for the items they can carry.
  • VRP with time windows , where the vehicles must visit the locations in specified time intervals.
  • VRP with resource constraints , such as space or personnel to load and unload vehicles at the depot (the starting point for the routes).
  • VRP with dropped visits , where the vehicles aren't required to visit all locations, but must pay a penalty for each visit that is dropped.

Limitations on solving vehicle routing problems

Vehicle routing problems are inherently intractable: the length of time it takes to solve them grows exponentially with the size of the problem. For sufficiently large problems, it could take OR-Tools (or any other routing software) years to find the optimal solution. As a result, OR-Tools sometimes returns solutions that are good, but not optimal. To find a better solution, change the search options for the solver. See Changing the search strategy for an example.

Except as otherwise noted, the content of this page is licensed under the Creative Commons Attribution 4.0 License , and code samples are licensed under the Apache 2.0 License . For details, see the Google Developers Site Policies . Java is a registered trademark of Oracle and/or its affiliates.

Last updated 2024-08-28 UTC.

Optimization Online

Data-driven Stochastic Vehicle Routing Problems with Deadlines

  • Shanshan Wang
  • Erick Delage
  • Leandro Coelho

Vehicle routing problems (VRPs) with deadlines have received significant attention around the world. Moti vated by a real-world food delivery problem, we assume that the travel time depends on the routing decisions, and study a data-driven stochastic VRP with deadlines and endogenous uncertainty. We use the non- parametric approaches, including k-nearest neighbor (kNN) and kernel density estimation (KDE), to estimate the decision-dependent probability distribution of travel time. To solve the resulting problem efficiently, we employ a logic-based Benders decomposition (LBBD) algorithm with several algorithmic enhancements. In particular, we propose a novel family of optimality cuts that includes the expected delay for all the sub routes. Moreover, we solve a total travel cost minimization problem to warm-start the algorithm. We also use a local search procedure to improve the current routing decision and propose a machine learning-based lower bound heuristic to efficiently solve problems of realistic size. A practical case study for a food delivery routing problem using real-world data is conducted to show the efficiency of the proposed techniques and the advantage of the data-driven stochastic VRP in reducing the expected delay. In our case study, we show that incorporating routing decisions in a non-parametric model outperforms a state-of-the-art data-driven parametric model by 23% on average in terms of the expected delay, and the order-assignment decisions obtained from a robust model with travel-time predictors by 26% on average. Moreover, compared with the drivers’ actual routes, our suggested routes can significantly improve the on-time performance of delivery services. We also quantify the value of the proposed routes with different service deadlines.

View Data-driven Stochastic Vehicle Routing Problems with Deadlines

  • Quantitative Management
  • Analytical Solutions
  • Applied Mathematics
  • Mathematics
  • Vehicle Routing Problem

Constructive heuristics for Capacitated Vehicle Routing Problem: a comparative study

  • September 2019
  • Proceedings of the Institute for System Programming of RAS 31(3):145-156
  • 31(3):145-156

Sergey Avdoshin at National Research University Higher School of Economics

  • National Research University Higher School of Economics

E.N. Beresneva at National Research University Higher School of Economics

Abstract and Figures

Solution quality of constructive heuristics, set B

Discover the world's research

  • 25+ million members
  • 160+ million publication pages
  • 2.3+ billion citations
  • Haneen Algethami

Ghada Talat Alhothali

  • DISCRETE DYN NAT SOC
  • Yeping Chen

Jian Li

  • Dongqing Zhang
  • Pedro Leonardo Rodríguez Quintana
  • Lucía Argüelles Cortés

Gonzalo Palencia

  • J CLEAN PROD

Chefi Triki

  • COMPUT IND ENG

Kris Braekers

  • Gilbert Laporte

Frédéric Semet

  • Edward Wasil
  • T. J. Gaskell

Thibaut Vidal

  • Yves Nobert
  • Martin Desrochers
  • P. C. Yellow
  • Recruit researchers
  • Join for free
  • Login Email Tip: Most researchers use their institutional email address as their ResearchGate login Password Forgot password? Keep me logged in Log in or Continue with Google Welcome back! Please log in. Email · Hint Tip: Most researchers use their institutional email address as their ResearchGate login Password Forgot password? Keep me logged in Log in or Continue with Google No account? Sign up

Multi-objective multi-compartment vehicle routing problem of fresh products with the promised latest delivery time

  • Original Research
  • Published: 09 September 2024

Cite this article

vehicle routing assignment problem

  • Xiufeng Li   ORCID: orcid.org/0000-0002-6072-652X 1  

61 Accesses

Explore all metrics

In light of the growing consumer emphasis on delivery speed and corporate environmental responsibility, it becomes paramount to simultaneously address the alignment of customer expectations with corporate objectives. We undertake a comprehensive examination of delivery delays and carbon emissions stemming from e-commerce logistics, leading us to formulate a delivery delay penalty function informed by customer behavior traits such as loss aversion. Concurrently, we analyze various factors influencing customer satisfaction and integrate them into our model. Similarly, we incorporate multiple determinants impacting vehicle emissions, devising a logistics cost-minimization model encompassing carbon emissions and cooling expenses. By amalgamating considerations of customer satisfaction, logistics expenses, and environmental concerns, we devise a dual-objective optimization model. To tackle this complex challenge, we introduce a multi-objective Artificial Bee Colony algorithm based on MOEA/D principles, substantiating its efficacy through extensive numerical experiments. Our findings demonstrate the algorithm's ability to intelligently optimize logistics routes, thus reducing vehicle utilization. Finally, we present a Pareto front, illustrating how mitigating customer satisfaction can alleviate logistics and carbon emission costs.

This is a preview of subscription content, log in via an institution to check access.

Access this article

Subscribe and save.

  • Get 10 units per month
  • Download Article/Chapter or eBook
  • 1 Unit = 1 Article or 1 Chapter
  • Cancel anytime

Price includes VAT (Russian Federation)

Instant access to the full article PDF.

Rent this article via DeepDyve

Institutional subscriptions

vehicle routing assignment problem

Similar content being viewed by others

vehicle routing assignment problem

Multi-objective artificial bee colony algorithm for reducing carbon emission and maximizing profit in a circular supply chain network

vehicle routing assignment problem

An Overview of Optimization Models and Technological Trends of Logistics in the Retail Sector

vehicle routing assignment problem

Formulation of a multi-period multi-echelon location-inventory-routing problem comparing different nature-inspired algorithms

Explore related subjects.

  • Artificial Intelligence

Data availability

The datasets used and/or analysed during the current study are available from the corresponding author on reasonable request.

Afshar-Bakeshloo, M., Mehrabi, A., Safari, H., Maleki, M., & Jolai, F. (2016). A green vehicle routing problem with customer satisfaction criteria. Journal of Industrial Engineering International, 12 (4), 529–544. https://doi.org/10.1007/S40092-016-0163-9

Article   Google Scholar  

Akkaya, D., Bimpikis, K., & Lee, H. (2021). Government interventions to promote agricultural innovation. Manufacturing and Service Operations Management, 23 (2), 437–452. https://doi.org/10.1287/MSOM.2019.0834

Baldacci, R., Toth, P., & Vigo, D. (2010). Exact algorithms for routing problems under vehicle capacity constraints. Annals of Operations Research, 175 , 213–245. https://doi.org/10.1007/s10479-009-0650-0

Barkaoui, M., Berger, J., & Boukhtouta, A. (2015). Customer satisfaction in dynamic vehicle routing problem with time windows. Applied Soft Computing Journal, 35 , 423–432. https://doi.org/10.1016/J.ASOC.2015.06.035

Bektaş, T., & Laporte, G. (2011). The pollution-routing problem. Transportation Research Part b: Methodological, 45 (8), 1232–1250. https://doi.org/10.1016/J.TRB.2011.02.004

Brandão, J. (2020). A memory-based iterated local search algorithm for the multi-depot open vehicle routing problem. European Journal of Operational Research, 284 , 559–571. https://doi.org/10.1016/j.ejor.2020.01.008

Brandstätter, C. (2021). A metaheuristic algorithm and structured analysis for the line-haul feeder vehicle routing problem with time windows. Central European Journal of Operations Research, 29 (1), 247–289. https://doi.org/10.1007/S10100-019-00625-0

Cai, L., Lv, W., Xiao, L., & Xu, Z. (2021). Total carbon emissions minimization in connected and automated vehicle routing problem with speed variables. Expert Systems with Applications, 165 , 113910. https://doi.org/10.1016/j.eswa.2020.113910

Cavero, S., Pardo, E. G., & Duarte, A. (2022). A general variable neighborhood search for the cyclic antibandwidth problem. Computational Optimization and Applications, 81 (2), 657–687. https://doi.org/10.1007/S10589-021-00334-Y

Chen, G., Wahab, M. I. M., & Fang, L. (2022). Optimal replenishment strategy for a single-manufacturer multi-retailer cold chain considering multi-stage quality degradation. Applied Mathematical Modelling, 104 , 96–113. https://doi.org/10.1016/J.APM.2021.11.019

Chen, J., Liao, W., & Yu, C. (2021). Route optimization for cold chain logistics of front warehouses based on traffic congestion and carbon emission. Computers and Industrial Engineering, 161 (August), 107663. https://doi.org/10.1016/j.cie.2021.107663

Cordeau, J. F., Laporte, G., & Mercier, A. (2004). Improved tabu search algorithm for the handling of route duration constraints in vehicle routing problems with time windows. Journal of the Operational Research Society, 55 (5), 542–546. https://doi.org/10.1057/PALGRAVE.JORS.2601707

Dayarian, I., Savelsbergh, M., & Clarke, J. P. (2020). Same-day delivery with drone resupply. Transportation Science, 54 (1), 229–249. https://doi.org/10.1287/trsc.2019.0944

Deb, K., Pratap, A., Agarwal, S., & Meyarivan, T. (2002). A fast and elitist multiobjective genetic algorithm: NSGA-II. IEEE Transactions on Evolutionary Computation, 6 (2), 182–197. https://doi.org/10.1109/4235.996017

Desfontaines, L., & Desaulniers, G. (2018). Multiple depot vehicle scheduling with controlled trip shifting. Transportation Research Part b: Methodological, 113 , 34–53. https://doi.org/10.1016/J.TRB.2018.05.011

Du, J., Wang, X., Wu, X., et al. (2023). Multi-objective optimization for two-echelon joint delivery location routing problem considering carbon emission under online shopping. Transportation Letters, 15 (8), 907–925.

Ferreira, K. M., de Queiroz, T. A., Munari, P., & Toledo, F. M. B. (2024). A variable neighborhood search for the green vehicle routing problem with two-dimensional loading constraints and split delivery. European Journal of Operational Research, 316 (2), 597–616. https://doi.org/10.1016/j.ejor.2024.01.049

Grey10Media. (2018) https://coldchainmanagement.org/2018/03/12/refrigerated-vehicles-and-cold-chain-management/

Guo, N., Qian, B., Na, J., Hu, R., & Mao, J. L. (2022a). A three-dimensional ant colony optimization algorithm for multi-compartment vehicle routing problem considering carbon emissions. Applied Soft Computing, 127 , 109326. https://doi.org/10.1016/j.asoc.2022.109326

Guo, X., Zhang, W., & Liu, B. (2022b). Low-carbon routing for cold-chain logistics considering the time-dependent effects of traffic congestion. Transportation Research Part D: Transport and Environment, 113 , 103502. https://doi.org/10.1016/j.trd.2022.103502

Huang, X., Tan, T., & Toktay, L. B. (2021). Carbon Leakage: The Impact of Asymmetric Regulation on Carbon-Emitting Production. Production and Operations Management, 30 (6), 1886–1903. https://doi.org/10.1111/poms.13181

Jabir, E., Panicker, V. V., & Sridharan, R. (2017). Design and development of a hybrid ant colony-variable neighbourhood search algorithm for a multi-depot green vehicle routing problem. Transportation Research Part d: Transport and Environment, 57 , 422–457. https://doi.org/10.1016/J.TRD.2017.09.003

Junior, A. J., Klotzle, M. C., Brandão, L. E. T., & Pinto, A. C. F. (2021). Prospect theory and narrow framing bias: Evidence from emerging markets. The Quarterly Review of Economics and Finance, 80 , 90–101. https://doi.org/10.1016/J.QREF.2021.01.016

Kara, I., Kara, B. Y., Yetis, M. K. (2007). Energy minimizing vehicle routing problem. In International Conference on Combinatorial Optimization and Applications (pp. 62–71) Springer-Verlag.

Klapp, M. A., Erera, A. L., & Toriello, A. (2018). The dynamic dispatch waves problem for same-day delivery. European Journal of Operational Research, 271 (2), 519–534. https://doi.org/10.1016/J.EJOR.2018.05.032

Larraín, S., Pradenas, L., Pulkkinen, I., & Santander, F. (2020). Multiobjective optimization of a continuous kraft pulp digester using SPEA2. Computers & Chemical Engineering, 143 , 107086. https://doi.org/10.1016/J.COMPCHEMENG.2020.107086

Lei, Y., Jasin, S., & Sinha, A. (2018). Joint dynamic pricing and order fulfillment for E-commerce retailers. Manufacturing and Service Operations Management, 20 (2), 269–284. https://doi.org/10.1287/MSOM.2017.0641

Li, X. (2023). Multi-objective vaccine delivery problem considering low carbon and customer loss aversion. Expert Systems with Applications, 223 , 119870. https://doi.org/10.1016/j.eswa.2023.119870

Li, Y., Soleimani, H., & Zohal, M. (2019). An improved ant colony optimization algorithm for the multi-depot green vehicle routing problem with multiple objectives. Journal of Cleaner Production, 227 , 1161–1172. https://doi.org/10.1016/j.jclepro.2019.03.185

Liu, C., Lv, J., Hou, P., & Lu, D. (2022). Disclosing products’ freshness level as a non-contractible quality: Optimal logistics service contracts in the fresh products supply chain. European Journal of Operational Research . https://doi.org/10.1016/J.EJOR.2022.09.024

Luo, J., & Chen, M. R. (2014). Multi-phase modified shuffled frog leaping algorithm with extremal optimization for the MDVRP and the MDVRPTW. Computers and Industrial Engineering, 72 (1), 84–97. https://doi.org/10.1016/j.cie.2014.03.004

Marrekchi, E., Besbes, W., Dhouib, D., et al. (2021). A review of recent advances in the operations research literature on the green routing problem and its variants. Annals of Operations Research, 304 , 529–574. https://doi.org/10.1007/s10479-021-04046-8

McNabb, M. E., Weir, J. D., Hill, R. R., & Hall, S. N. (2015). Testing local search move operators on the vehicle routing problem with split deliveries and time windows. Computers & Operations Research, 56 , 93–109. https://doi.org/10.1016/J.COR.2014.11.007

Moonsri, K., Sethanan, K., & Worasan, K. (2022). A novel enhanced differential evolution algorithm for outbound logistics of the poultry industry in Thailand. Journal of Open Innovation: Technology, Market, and Complexity, 8 (1), 15. https://doi.org/10.3390/joitmc8010015

Mor, A., & Speranza, M. G. (2022). Vehicle routing problems over time: A survey. Annals of Operations Research, 314 , 255–275. https://doi.org/10.1007/s10479-021-04488-0

Qian, J., & Eglese, R. (2016). Fuel emissions optimization in vehicle routing problems with time-varying speeds. European Journal of Operational Research, 248 (3), 840–848. https://doi.org/10.1016/J.EJOR.2015.09.009

Qin, S., Pi, D., Shao, Z., & Xu, Y. (2022). Hybrid collaborative multi-objective Artificial Bee Colony algorithm for scheduling workflow in cloud environment. Swarm and Evolutionary Computation, 68 , 101008. https://doi.org/10.1016/J.SWEVO.2021.101008

Qureshi, A. G., Taniguchi, E., & Yamada, T. (2009). An exact solution approach for vehicle routing and scheduling problems with soft time windows. Transportation Research Part e: Logistics and Transportation Review, 45 (6), 960–977. https://doi.org/10.1016/J.TRE.2009.04.007

Ray, S., Soeanu, A., Berger, J., & Debbabi, M. (2014). The multi-depot split-delivery vehicle routing problem: Model and solution algorithm. Knowledge-Based Systems, 71 , 238–265. https://doi.org/10.1016/J.KNOSYS.2014.08.006

Shao, L., Wang, D., & Wu, X. (2022). Competitive trading in forward and spot markets under yield uncertainty. Production and Operations Management, 31 (9), 3400–3418. https://doi.org/10.1111/poms.13769

Soriano, A., Gansterer, M., & Hartl, R. F. (2023). The multi-depot vehicle routing problem with profit fairness. International Journal of Production Economics, 255 , 108669. https://doi.org/10.1016/j.ijpe.2022.108669

Stodola, P., & Kutěj, L. (2024). Multi-depot vehicle routing problem with drones: Mathematical formulation, solution algorithm and experiments. Expert Systems with Applications . https://doi.org/10.1016/j.eswa.2023.122483

Sun, J., & Wang, X. (2015). Study on the e-commerce logistics distribution modes of fresh agricultural products. Applied Mechanics and Materials, 744–746 , 1895–1901. https://doi.org/10.4028/WWW.SCIENTIFIC.NET/AMM.744-746.1895

Sun, P., Veelenturf, L. P., Hewitt, M., & Van Woensel, T. (2018). The time-dependent pickup and delivery problem with time windows. Transportation Research Part b: Methodological, 116 , 1–24. https://doi.org/10.1016/J.TRB.2018.07.002

Sun, Q., Polman, E., & Zhang, H. (2021). On prospect theory, making choices for others, and the affective psychology of risk. Journal of Experimental Social Psychology, 96 , 104177. https://doi.org/10.1016/j.jesp.2021.104177

Suzuki, Y. (2016). A dual-objective metaheuristic approach to solve practical pollution routing problem. International Journal of Production Economics, 176 , 143–153.

Tavares, G., Zsigraiova, Z., Semiao, V., & Carvalho, M. D. G. (2008). A case study of fuel savings through optimisation of MSW transportation routes. Management of Environmental Quality: An International Journal, 19 (4), 444–454.

Wang, Y., Wang, L., Peng, Z., Chen, G., Cai, Z., & Xing, L. (2019). A multi ant system based hybrid heuristic algorithm for vehicle routing problem with service time customization. Swarm and Evolutionary Computation, 50 , 100563. https://doi.org/10.1016/j.swevo.2019.100563

Xiao, Y., & Konak, A. (2015). A simulating annealing algorithm to solve the green vehicle routing & scheduling problem with hierarchical objectives and weighted tardiness. Applied Soft Computing, 34 , 372–388.

Xiao, Y., Zhao, Q., Kaku, I., & Xu, Y. (2012). Development of a fuel consumption optimization model for the capacitated vehicle routing problem. Computers & Operations Research, 39 (7), 1419–1431.

Yang, Y., Chen, H., Li, S., Heidari, A. A., & Wang, M. (2020). Orthogonal learning harmonizing mutation-based Artificial Bee Colony-inspired optimizers. Applied Mathematical Modelling, 86 , 368–383.

Yesodha, R., & Amudha, T. (2022). A bio-inspired approach: Firefly algorithm for multi-depot vehicle routing problem with time windows. Computer Communications, 190 , 48–56. https://doi.org/10.1016/j.comcom.2022.04.005

Zhang, Q., & Li, H. (2007). MOEA/D: A multiobjective evolutionary algorithm based on decomposition. IEEE Transactions on Evolutionary Computation, 11 (6), 712–731.

Zitzler, E. (1999). Evolutionary algorithms for multiobjective optimization: Methods and applications. Doctoral dissertation ETH 13398, Swiss Federal Institute of Technology (ETH), Zurich, Switzerland.

Zou, Y., Wu, H., Yin, Y., Dhamotharan, L., Chen, D., & Tiwari, A. K. (2024). An improved transformer model with multi-head attention and attention to attention for low-carbon multi-depot vehicle routing problem. Annals of Operations Research, 339 (1), 517–536. https://doi.org/10.1007/S10479-022-04788-Z/METRICS

Download references

Author information

Authors and affiliations.

College of Management and Economics, Tianjin University, Tianjin, 300072, China

You can also search for this author in PubMed   Google Scholar

Contributions

Xiufeng Li conceived this study, conducted and drafted the context, and designed the algorithm. All authors read and approved the final manuscript.

Corresponding author

Correspondence to Xiufeng Li .

Ethics declarations

Conflict of interest.

The authors declare that they have no known competing financial interests or personal relationships that could have appeared to influence the work reported in this paper.

Ethical approval

Not applicable.

Consent to participate

Consent to publish, additional information, publisher's note.

Springer Nature remains neutral with regard to jurisdictional claims in published maps and institutional affiliations.

Rights and permissions

Springer Nature or its licensor (e.g. a society or other partner) holds exclusive rights to this article under a publishing agreement with the author(s) or other rightsholder(s); author self-archiving of the accepted manuscript version of this article is solely governed by the terms of such publishing agreement and applicable law.

Reprints and permissions

About this article

Li, X. Multi-objective multi-compartment vehicle routing problem of fresh products with the promised latest delivery time. Ann Oper Res (2024). https://doi.org/10.1007/s10479-024-06254-4

Download citation

Received : 15 May 2023

Accepted : 29 August 2024

Published : 09 September 2024

DOI : https://doi.org/10.1007/s10479-024-06254-4

Share this article

Anyone you share the following link with will be able to read this content:

Sorry, a shareable link is not currently available for this article.

Provided by the Springer Nature SharedIt content-sharing initiative

  • Fresh foods
  • Time windows
  • Multi-compartment
  • Multi-objective
  • Artificial bee colony algorithm
  • Find a journal
  • Publish with us
  • Track your research

Information

  • Author Services

Initiatives

You are accessing a machine-readable page. In order to be human-readable, please install an RSS reader.

All articles published by MDPI are made immediately available worldwide under an open access license. No special permission is required to reuse all or part of the article published by MDPI, including figures and tables. For articles published under an open access Creative Common CC BY license, any part of the article may be reused without permission provided that the original article is clearly cited. For more information, please refer to https://www.mdpi.com/openaccess .

Feature papers represent the most advanced research with significant potential for high impact in the field. A Feature Paper should be a substantial original Article that involves several techniques or approaches, provides an outlook for future research directions and describes possible research applications.

Feature papers are submitted upon individual invitation or recommendation by the scientific editors and must receive positive feedback from the reviewers.

Editor’s Choice articles are based on recommendations by the scientific editors of MDPI journals from around the world. Editors select a small number of articles recently published in the journal that they believe will be particularly interesting to readers, or important in the respective research area. The aim is to provide a snapshot of some of the most exciting work published in the various research areas of the journal.

Original Submission Date Received: .

  • Active Journals
  • Find a Journal
  • Proceedings Series
  • For Authors
  • For Reviewers
  • For Editors
  • For Librarians
  • For Publishers
  • For Societies
  • For Conference Organizers
  • Open Access Policy
  • Institutional Open Access Program
  • Special Issues Guidelines
  • Editorial Process
  • Research and Publication Ethics
  • Article Processing Charges
  • Testimonials
  • Preprints.org
  • SciProfiles
  • Encyclopedia

electronics-logo

Article Menu

vehicle routing assignment problem

  • Subscribe SciFeed
  • Recommended Articles
  • Author Biographies
  • Google Scholar
  • on Google Scholar
  • Table of Contents

Find support for a specific problem in the support section of our website.

Please let us know what you think of our products and services.

Visit our dedicated information section to learn more about MDPI.

JSmol Viewer

Vehicle-uav integrated routing optimization problem for emergency delivery of medical supplies.

vehicle routing assignment problem

1. Introduction

2. literature review, 2.1. current research status of emergency material distribution, 2.2. current research status of vehicle-uav integrated delivery path optimization, 2.3. current research on solving methods for routing optimization problem, 3. research methodology, 3.1. modeling ideas, 3.1.1. objective functions, 3.1.2. constraints, 4. results and discussions, 4.1. data preprocessing for uav delivery to customers, 4.1.1. uav customer clustering processing based on dbscan algorithm.

  • If “No”, then a is marked as noise, and the process loops back to select another random point.
  • If “Yes”, then a new class C will be created, and a will be added to this class.
  • If “No”, then the current points are added to the class C.
  • If “Yes”, then the delivery points are added to the set N , all midpoints are marked, and the classes are outputted.

4.1.2. Dual Objective Processing in the Model

4.1.3. artificial bee colony algorithm initialization and neighborhood search strategy, 4.1.4. decoding strategy for uav path, 4.1.5. adaptive probability design for following bees, 4.2. a demonstrative case, 4.3. discussions, 5. improved artificial bee colony, results analysis, 6. conclusions and discussion, 6.1. conclusions, 6.2. future work, author contributions, data availability statement, conflicts of interest.

  • Li, X.F. Analysis of Implementing the National Emergency System Plan for the 14th Five Year Plan. Labor Prot. 2022 , 4 , 30–32. [ Google Scholar ]
  • Lee, M.; Choi, M.; Yang, T.; Kim, J.; Kim, J.; Kwon, O.; Cho, N. A Study on the Advancement of Intelligent Military UAVs: Focusing on Reconnaissance Operations. IEEE Access 2024 , 12 , 55964–55975. [ Google Scholar ] [ CrossRef ]
  • Deng, T.; Xu, X.; Zou, Z.; Liu, W.; Wang, D.; Hu, M. Multi-Drone Parcel Delivery via Public Vehicles: A Joint Optimization Approach. IEEE Internet Things J. 2023 , 11 , 9312–9323. [ Google Scholar ] [ CrossRef ]
  • Jasim, A.N.; Fourati, L.C. Guided Genetic Algorithm for Solving Capacitated Vehicle Routing Problem with Unmanned-Aerial-Vehicles. IEEE Access 2024 , 12 , 106333–106358. [ Google Scholar ] [ CrossRef ]
  • Elghitani, F. Dynamic UAV routing for multi-access edge computing. IEEE Trans. Veh. Technol. 2024 , 73 , 8878–8888. [ Google Scholar ] [ CrossRef ]
  • Maharana, A.; Amutorine, M.; Sengeh, M.D.; Nsoesie, E.O. COVID-19 and beyond: Use of digital technology for pandemic response in Africa. Sci. Afr. 2021 , 14 , e01041. [ Google Scholar ] [ CrossRef ]
  • Sudbury, A.W.; Hutchinson, E.B. A cost analysis of amazon prime air (UAV delivery). J. Econ. Educ. 2016 , 16 , 1–12. [ Google Scholar ]
  • UPS Pressroom. UPS Flight Forward, CVS To Launch Residential UAV Delivery Service in Florida Retirement Community to Assist in Coronavirus Response ; UPS Pressroom: Atlanta, GA, USA, 2020. [ Google Scholar ]
  • Bamburry, D. UAVs: Designed for product delivery. Des. Manag. Rev. 2015 , 26 , 40–48. [ Google Scholar ]
  • Ahmed, J.U.; Islam, Q.T.; Islam, S.; Ahmed, A.; Mim, K.P. Last-Mile UAV Delivery: Is Wing’s Business Model Sustainable? SAGE Business Cases Originals ; SAGE Publications: Thousand Oaks, CA, USA, 2022. [ Google Scholar ]
  • Demuyakor, J. Ghana go digital Agenda: The impact of zipline UAV technology on digital emergency health delivery in Ghana. Humanities 2020 , 8 , 242–253. [ Google Scholar ] [ CrossRef ]
  • Reddy, T.B.S.; Teja, P.H.; Teja, R.P.; Praneeth, T. Adaptive Autonomous Technology in Unmanned Aerial Vehicles for Parcel Delivery. Int. J. Sci. Res. Comput. Sci. Eng. Inf. Technol. 2019 , 5 , 7–12. [ Google Scholar ] [ CrossRef ]
  • Liu, Y.; Liu, Z.; Shi, J.; Wu, G.; Pedrycz, W. Two-echelon routing problem for parcel delivery by cooperated truck and UAV. IEEE Trans. Syst. Man Cybern. Syst. 2020 , 51 , 7450–7465. [ Google Scholar ] [ CrossRef ]
  • Supriya, M.; Jeevitha, S.; Pranathi, S.K.; Lokeshkumar, M. The Wings of Wellness: Autonomous Medical Delivery Drones Enhancing Patient Care and Accessibility. In Proceedings of the 2024 Ninth International Conference on Science Technology Engineering and Mathematics (ICONSTEM), Chennai, India, 4–5 April 2024; pp. 1–6. [ Google Scholar ]
  • Jung, H.; Kim, J. UAV scheduling model for delivering small parcels to remote islands considering wind direction and speed. Comput. Ind. Eng. 2022 , 163 , 107784. [ Google Scholar ] [ CrossRef ]
  • Tian, Z.; Haas, Z.J.; Shinde, S. Routing in solar-powered UAV delivery system. Drones 2022 , 6 , 282. [ Google Scholar ] [ CrossRef ]
  • Zhu, L.; Cao, J.; Gu, J.; Zheng, Y. Dynamic emergency supply distribution considering fair mitigation of victim suffering. Syst. Eng. Theory Pract. 2020 , 40 , 2427–2437. [ Google Scholar ]
  • Qin, J.; Xu, J.L. Research on Optimization of Emergency Support Service Network for Livelihood Materials under the Background of Sudden Infectious Disease Epidemic. J. Railw. Sci. Eng. 2023 , 20 , 1–11. [ Google Scholar ]
  • Yu, G.H. Research on the Occupying Path Problem of Unconventional Disaster Emergency Logistics Based on Complex Networks. Master’s Thesis, South China University of Technology, Guangzhou, China, 2012. [ Google Scholar ]
  • Wu, T.; He, L.; Yu, H. Online traveling salesman problem with time cost and non-zealous server. J. Comb. Optim. 2022 , 44 , 2143–2166. [ Google Scholar ] [ CrossRef ]
  • Su, B.; Geng, X.Y. Research on the unpredictable emergency material distribution path selection for service requests at demand points. Chin. Manag. Sci. 2022 , 30 , 1–10. [ Google Scholar ]
  • Wohlsen, M. The Next Big Thing You Missed: Amazon’s Delivery UAVs Could Work—They Just Need Trucks ; Wired Business: London, UK, 2014. [ Google Scholar ]
  • Yurek, E.E.; Ozmutlu, H.C. A decomposition-based iterative optimization algorithm for traveling salesman problem with UAV. Transp. Res. Part C Emerg. Technol. 2018 , 91 , 249–262. [ Google Scholar ] [ CrossRef ]
  • de Freitas, J.C.; Penna, P.H.V. A randomized variable neighborhood descent heuristic to solve the flying sidekick traveling salesman problem. Electron. Notes Discret. Math. 2018 , 66 , 95–102. [ Google Scholar ] [ CrossRef ]
  • Sacramento, D.; Pisinger, D.; Ropke, S. An adaptive large neighborhood search metaheuristic for the vehicle routing problem with UAVs. Transp. Res. Part C Emerg. Technol. 2019 , 102 , 289–315. [ Google Scholar ] [ CrossRef ]
  • Cavani, S.; Iori, M.; Roberti, R. Exact methods for the traveling salesman problem with multiple UAVs. Transp. Res. Part C Emerg. Technol. 2021 , 130 , 103280. [ Google Scholar ] [ CrossRef ]
  • Murray, C.C.; Raj, R. The multiple flying sidekicks traveling salesman problem: Parcel delivery with multiple UAVs. Transp. Res. Part C Emerg. Technol. 2020 , 110 , 368–398. [ Google Scholar ] [ CrossRef ]
  • Liu, W.S.; Li, W.; Zhou, Q. Optimization model and algorithm for “UAV-vehicle” delivery path. Dep. Transp. Unified Eng. Inf. 2021 , 21 , 176–186. [ Google Scholar ]
  • Yang, L.; Zhou, J. Research on Distribution Path Problem of Truck Combined with UAV in Restricted Area. J. Comput. Eng. Appl. 2023 , 59 , 326. [ Google Scholar ]
  • Tamke, F.; Buscher, U. The vehicle routing problem with UAVs and UAV speed selection. Comput. Oper. Res. 2023 , 152 , 106112. [ Google Scholar ] [ CrossRef ]
  • Moshref-Javadi, M.; Hemmati, A.; Winkenbach, M. A truck and UAVs model for last-mile delivery: A mathematical model and heuristic approach. Appl. Math. Model. 2020 , 80 , 290–318. [ Google Scholar ] [ CrossRef ]
  • Lin, M.; Lyu, J.Y.; Gao, J.J.; Li, L.Y. Model and hybrid algorithm of integrated distribution system with multiple UAVs and a truck. Sci. Program. 2020 , 2020 , 8887057. [ Google Scholar ]
  • Zhang, R.; Dou, L.; Xin, B.; Chen, C.; Deng, F.; Chen, J. A review on the truck and UAV cooperative delivery problem. Unmanned Syst. 2023 , 12 , 823–847. [ Google Scholar ] [ CrossRef ]
  • Boyd, S.; Mattingley, J. Branch and Bound Methods ; Notes for EE364b; Stanford University: Stanford, CA, USA, 2007. [ Google Scholar ]
  • Li, Y.; Li, J. Dynamic programming heuristic algorithm for solving time-varying vehicle scheduling problems. Syst. Eng. Theory Pract. 2012 , 32 , 1712–1718. [ Google Scholar ]
  • Ou, W.; Jiao, L. A dynamic planning algorithm for the vehicle path problem under Ping contingency. Comput. Simul. 2011 , 28 , 354–358. [ Google Scholar ]
  • Lin, S.; Kernighan, B.W. An effective heuristic algorithm for the traveling-salesman problem. Oper. Res. 1973 , 21 , 498–516. [ Google Scholar ] [ CrossRef ]
  • Solomon, M.M. Algorithms for the vehicle routing and scheduling problems with time window constraints. Oper. Res. 1987 , 35 , 254–265. [ Google Scholar ] [ CrossRef ]
  • Lin, S. Computer solutions of the traveling salesman problem. Bell Syst. Tech. J. 1965 , 44 , 2245–2269. [ Google Scholar ]
  • Vigo, D. A heuristic algorithm for the asymmetric capacitated vehicle routing problem. Eur. J. Oper. Res. 1996 , 89 , 108–126. [ Google Scholar ]
  • Ester, M.; Kriegel, H.P.; Sander, J.; Xu, X. Density-based spatial clustering of applications with noise. In Proceedings of the International Conference on Knowledge Discovery and Data Mining, Portland, OR, USA, 2–4 August 1996; Volume 240. [ Google Scholar ]
  • Zhang, J.H. Application of simulated annealing ant colony algorithm in optimal path selection. J. Huaihua Coll. 2022 , 41 , 68–75. [ Google Scholar ]
  • Kirkpatrick, S.; Vecchi, M.P. Optimization by simulated annealing. In Readings in Computer Vision: Issues, Problems, Principles, and Paradigms ; Morgan Kaufmann Publishers Inc.: Burlington, MA, USA, 1987; pp. 339–348. [ Google Scholar ]
  • Wang, Y.; Shin, H.-k.; Zhou, Y.; Lin, H. An artificial bee colony algorithm for solving the shortest path problem in transportation networks. J. Jilin Univ. 2021 , 59 , 1144–1150. [ Google Scholar ]
  • Jiang, M.Y.; Yuan, D.F. Artificial Bee Colony Algorithm and Its Application ; Science Press: Beijing, China, 2014. [ Google Scholar ]
  • Metropolis, N.; Rosenbluth, A.W.; Rosenbluth, M.N. Equation of State Calculations by Fast Computing Machines. J. Chem. Phys. 2004 , 21 , 1087–1092. [ Google Scholar ] [ CrossRef ]

Click here to enlarge figure

Basic ParametersBasic Parameters
Number of vehicles used (vehicle)1Number of available UAVs4
Customer point service time (min)20Maximum capacity of UAV (kg)200
Bee colonies40Farthest-flying distance of UAVs (km)50
Exchange probability0.15UAV violation of capacity constraint penalty coefficient10
Insertion probability0.35Average vehicle speed (km/h)80
Reverse order probability0.5UAV flying speed (km/h)150
Delivery PointX Axis
Coordinates
Y Axis
Coordinates
Demand (kg)Earliest Expected Arrival Time (min)Latest Expected Arrival Time (min)
Vehicles can get to passengers
05025001000
11571254070140
235539070180720
310033560300500
414818820100360
Vehicles can get to passengers540503080180
61651475090200
735037590300500
830030030260560
927522560190700
1025020020200480
1124519035290750
1219817040190640
1326526550240590
1420045070220800
1525040090560950
1610039865630900
Delivery UAVs1456810912967
2457030825870
342661065146
4426810727782
54265101567
6406920621702
7386820255324
8387010534605
9356910448505
10208040384429
1118752099148
12157520179254
13158010278345
143050101073
15285220812883
1625501065144
17255240169224
181354810812867
191856030525570
201661261085126
211208010327442
2214010920531602
2320013410534605
242356010348405
2518812940284329
2612510220179254
271058310278345
2823579104093
291608120712803
301381201035104
311004240119164
3238531840812867
3338037130525570
3438534510612657
3539730240525570
3634540020225246
3735040950387432
3838837810415467
3938934920301382
4040038030185224
4137837650428505
4234940130254289
439030020554689
4415020010855867
Genetic AlgorithmArtificial Bee Colony (ABC)
Iterations15001500
Total delivery time of vehicles and UAVs (h)5821.8
Algorithm average running time (s)2414
Algorithm NamePath SequencePath Time (h)IterationsAlgorithm Time (s)
Artificial bee colony0 1 6 12 13 8 11 10 9 2 7 15 14 16 3 4 517.8631150010.5
Improved bee colony0 5 4 3 16 14 15 2 7 8 13 9 10 11 12 6 115.16018006.3
IndexArtificial Bee Colony AlgorithmImproved Bee Colony Algorithm
Vehicle-UAV total path time (h)21.818.2
Iterations1500800
Algorithm average running time (s)147.5
The statements, opinions and data contained in all publications are solely those of the individual author(s) and contributor(s) and not of MDPI and/or the editor(s). MDPI and/or the editor(s) disclaim responsibility for any injury to people or property resulting from any ideas, methods, instructions or products referred to in the content.

Share and Cite

Ghaffar, M.A.; Peng, L.; Aslam, M.U.; Adeel, M.; Dassari, S. Vehicle-UAV Integrated Routing Optimization Problem for Emergency Delivery of Medical Supplies. Electronics 2024 , 13 , 3650. https://doi.org/10.3390/electronics13183650

Ghaffar MA, Peng L, Aslam MU, Adeel M, Dassari S. Vehicle-UAV Integrated Routing Optimization Problem for Emergency Delivery of Medical Supplies. Electronics . 2024; 13(18):3650. https://doi.org/10.3390/electronics13183650

Ghaffar, Muhammad Arslan, Lei Peng, Muhammad Umer Aslam, Muhammad Adeel, and Salim Dassari. 2024. "Vehicle-UAV Integrated Routing Optimization Problem for Emergency Delivery of Medical Supplies" Electronics 13, no. 18: 3650. https://doi.org/10.3390/electronics13183650

Article Metrics

Article access statistics, further information, mdpi initiatives, follow mdpi.

MDPI

Subscribe to receive issue release notifications and newsletters from MDPI journals

IMAGES

  1. Vehicle Routing Problem: Meaning and Solutions (In-depth Guide)

    vehicle routing assignment problem

  2. Example of a basic Vehicle Routing Problem (VRP)

    vehicle routing assignment problem

  3. Vehicle routing problem with mathematical solvers

    vehicle routing assignment problem

  4. [Solved] The vehicle routing assignment consists o

    vehicle routing assignment problem

  5. How to Solve Dynamic Vehicle Routing Problems (Ultimate Guide 2024)

    vehicle routing assignment problem

  6. (PDF) The Time Window Assignment Vehicle Routing Problem

    vehicle routing assignment problem

VIDEO

  1. Genetic Algoritm for Vehicle Routing Problem (GA for VRP)

  2. Vehicle Routing Problem (VRP) using Simulated Annealing (SA) free matlab code videos download

  3. VRP

  4. Solving a Routing Planning Problem using LogVRP

  5. Tax for Assignment vehicle in Rajasthan/अन्य राज्य के वाहन का असाइनमेंट पर टैक्स

  6. Explicacion Capacitated Vehicle Routing Problem(CVRP) mediante GRASP

COMMENTS

  1. The Vehicle Routing Problem: Exact and Heuristic Solutions

    Objective function of CVRP. (Image by the author). We also need to include constraints that ensure: Each customer i is visited once, therefore has one active arc which starts from it and one that arrives on it.; If any arc variable indexed by vehicle k goes into one node i or out of it, the demand q of this node is assigned to vehicle k.; The total demand assigned to a vehicle must not exceed ...

  2. Determining the problem type

    "The vehicle routing problem with time windows and temporal dependencies" (Dohn et al., 2011) DOI ... Every Vehicle routing is an assignment problem. Basically, VRP is about the combination of assignments and knapsack. You assign a vehicle for each node and a knapsack for the commodity carried for each vehicle. From your description, besides ...

  3. Vehicle Routing Problem

    One answer is the routes with the least total distance. However, if there are no other constraints, the optimal solution is to assign just one vehicle to visit all locations, and find the shortest route for that vehicle. This is essentially the same problem as the TSP. A better way to define optimal routes is to minimize the length of the ...

  4. The discrete time window assignment vehicle routing problem

    The time window assignment vehicle routing problem (TWAVRP) was recently introduced by Spliet and Gabor (2014). This problem can be viewed as a two-stage stochastic optimization problem (see Birge and Louveaux, 1997). Given a set of customers to be visited on the same day regularly during some period of time (e.g., a season), the TWAVRP ...

  5. Vehicle Routing Problem with Time Windows

    Many vehicle routing problems involve scheduling visits to customers who are only available during specific time windows. ... // Solve the problem. Assignment solution = routing.solveWithParameters(searchParameters); // Print solution on console. printSolution(data, routing, manager, solution); } } // [END_program_part1] ...

  6. PDF The Robust Vehicle Routing Problem with Time Window Assignments

    The vehicle routing problem with time window assignments (VRP-TWA) and uncertain demand was introduced by Spliet and Gabor (2014). In this prob-lem, time windows have to be assigned to customers before demand is known. When the demand is revealed, a vehicle routing schedule has to be generated that satis es the assigned time windows.

  7. The Time Window Assignment Vehicle Routing Problem

    Abstract. In this paper we introduce the time window assignment vehicle routing problem (TWAVRP). In this problem, time windows have to be assigned before demand is known. Next, a realization of demand is revealed, and a vehicle routing schedule is made that satisfies the assigned time windows. The objective is to minimize the expected ...

  8. A generalized formulation for vehicle routing problems

    In addition, vehicle routing problems lead to challenging formulations that require the development of sophisticate solution strategies and motivates the design of clever heuristics and meta-heuristics [2, 18]. Vehicle routing problems are typically modeled using two di erent types of formulations. The rst type, known as vehicle

  9. Vehicle assignment in site-dependent vehicle routing problems with

    In this paper we consider the problem of vehicle assignment in heterogeneous fleet site-dependent Vehicle Routing Problems (VRP) with split deliveries. In such VRP problems vehicles can have ...

  10. The Time Window Assignment Vehicle Routing Problem with ...

    In this paper, we introduce the time window assignment vehicle routing problem (TWAVRP) with time-dependent travel times. It is the problem of assigning time windows to customers before their demand is known and creating vehicle routes adhering to these time windows after demand becomes known. The goal is to assign the time windows in such a ...

  11. Using Group Role Assignment to Solve Dynamic Vehicle Routing Problem

    The Dynamic Vehicle Routing Problem (DVRP) is more complex than the traditional Vehicle Routing Problem (VRP) in the combinatorial optimization of operations research. With more degrees of freedom, DVRP introduces new challenges while judging the merit of a given route plan. This paper utilizes the time slice strategy to solve dynamic and deterministic routing problems. Based on Group Role ...

  12. Vehicle assignment in site-dependent vehicle routing problems with

    In this paper we consider the problem of vehicle assignment in heterogeneous fleet site-dependent Vehicle Routing Problems (VRP) with split deliveries. In such VRP problems vehicles can have different capacities, fixed and travel costs, and site-dependency constraints limit for every customer a set of vehicles, which can serve it. The Vehicle Assignment Problem (VAP) arises in heuristic and ...

  13. A generalized assignment heuristic for vehicle routing

    We present a heuristic for this problem in which an assignment of customers to vehicles is obtained by solving a generalized assignment problem with an objective function that approximates delivery cost. This heuristic has many attractive features. It has outperformed the best existing heuristics on a sample of standard test problems.

  14. Vehicle routing problems with soft time windows: an optimization based

    In that matter, many researchers tried to improve bus systems by solving different problems like Vehicle Routing Problem (VRP) [6,10,[25] [26] [27], Vehicle Assignment Problem (VAP) [2 ...

  15. Genetic Algorithms with Neural Cost Predictor for Solving Hierarchical

    Vehicle Routing Problems (VRPs) have garnered significant attention over the past few decades due to their crucial role in last-mile delivery logistics. ... The fitness cost of an assignment is calculated from the sum of subproblems' cost predictions and the distinctiveness of the decomposition compared to other assignments. We refer to this ...

  16. PDF Data-driven Stochastic Vehicle Routing Problems with Deadlines

    Wang, Delage & Coelho Data-driven Stochastic Vehicle Routing Problems with Deadline Assignment 3 order. If this estimated completion time is close to or larger than the deadline, the driver will speed up, leading to a shorter travel time. In last-mile delivery, drivers can use electric bikes, which are

  17. New Assignment Algorithms for the Multi-Depot Vehicle Routing Problem

    Keywords: multi-depot vehicle routing problem; clustering; assignment Introduction A key element of many distribution systems is the routing and scheduling of vehicles through a set of customers requiring service. The vehicle routing problem (VRP) involves the design of a set of minimum-cost vehicle

  18. Approaches to Solve the Vehicle Routing Problem in the Valuables

    Abstract and Figures. The various extensions of the vehicle routing problem with time windows (VRPTW) are considered. In addition to the VRPTW, the authors present a method to solve the SDVRPTW ...

  19. Vehicle Routing Problem: Meaning and Solutions (In-depth Guide)

    The vehicle routing problem is a serious and one of the oldest challenges in the logistics and delivery business. George Dantzig and John Ramser first represented it in 1959 in their paper. In which they wrote the first mathematical approach and later applied it to the petrol deliveries. You must have heard about the Traveling Salesman Problem ...

  20. Vehicle Routing

    OR-Tools can solve many types of VRPs, including the following: Traveling Salesperson Problem, the classic routing problem in which there is just one vehicle. Vehicle routing problem, a generalisation of the TSP with multiple vehicles. VRP with capacity constraints, in which vehicles have maximum capacities for the items they can carry.

  21. Data-driven Stochastic Vehicle Routing Problems with Deadlines

    Vehicle routing problems (VRPs) with deadlines have received significant attention around the world. Motivated by a real-world food delivery problem, we assume that the travel time depends on the routing decisions, and study a data-driven stochastic VRP with deadlines and endogenous uncertainty. We use the non-parametric approaches, including k ...

  22. A new ILP-based refinement heuristic for Vehicle Routing Problems

    In this paper we address the Distance-Constrained Capacitated Vehicle Routing Problem (DCVRP), where k minimum-cost routes through a central depot have to be constructed so as to cover all customers while satisfying, for each route, both a capacity and a total-distance-travelled limit.Our starting point is the following refinement procedure proposed in 1981 by Sarvanov and Doroshko for the ...

  23. Genetic Algorithms with Neural Cost Predictor for Solving Hierarchical

    Examples are the multi-depot vehicle routing problem (MDVRP), where customers are assigned to depots before delivery, and the capacitated location routing problem (CLRP), where the locations of depots should be determined first. A simple and straightforward approach for such hierarchical problems would be to separate the higher-level decisions ...

  24. Constructive heuristics for Capacitated Vehicle Routing Problem: a

    Vehicle Routing Problem (VRP) is concerned with the optimal design of routes to be used by a fleet. of vehicles to serve a set of customers. In this study we analyze con structive heuristics for a ...

  25. Multi-objective multi-compartment vehicle routing problem of fresh

    3.1 Problem description. The multi-depot multi-compartment vehicle routing problem can be described as follows: a fresh goods e-commerce platform has a series of fresh product depots \(\mathop D\nolimits_{i} (i = 1,2, \cdots ,m)\), where orders are received from \(n\) customers. To maintain the freshness level of fresh products and prevent decay, e-commerce companies need to use refrigerator ...

  26. Vehicle-UAV Integrated Routing Optimization Problem for Emergency

    In recent years, the delivery of medical supplies has faced significant challenges due to natural disasters and recurrent public health emergencies. Addressing the need for improved logistics operations during such crises, this article presents an innovative approach, namely integrating vehicle and unmanned aerial vehicle (UAV) logistics to enhance the efficiency and resilience of medical ...